BEC All Chapters Flashcards

1
Q

The difference between standard hours at standard wage rates and actual hours at standard wage rates is referred to as which of the following types of variances?

A.
Labor rate

Correct B.
Labor usage

C.
Direct labor spending

D.
Indirect labor spending

A

SR(SH-AH)

What is inside the parenthesis HRS used so you calculate labor usage.

How well did you know this?
1
Not at all
2
3
4
5
Perfectly
2
Q

Which of the following is the best way to identify and manage risk?

A.
Know the impact on the project

B.
Have experts on the team

C.
Control costs

Incorrect D.
Know the risks

A

The best way to identify and manage risk is to have experts in the area of the project on the team. These experts will have had experience in the aspects of the project and can help identify possible risks as well as manage the risks without busting the bank.

To help identify risks, the project manager may perform brainstorming sessions. One example of brainstorming is the Delphi method, a technique for decision making and problem solving. The Delphi method attempts to develop a forecast through a group consensus. Individual experts are asked to respond to an initial questionnaire followed by a second one prepared using the information and opinions gathered through the first questionnaire. In this process, each expert is asked to reconsider and revise his or her initial answers to the various questions. This process continues until some type (or range) of consensus is reached.

How well did you know this?
1
Not at all
2
3
4
5
Perfectly
3
Q

Which of the following assumptions is associated with the economic order quantity formula?

A.
The carrying cost per unit will vary with quantity ordered.

B.
The cost of placing an order will vary with quantity ordered.

Correct C.
Periodic demand is known.

D.
The purchase cost per unit will vary based on quantity discounts.

A

Assumptions of economic order quantity analysis include the following:

Periodic demand for the good is known.
Total carrying costs vary with quantity ordered.
Costs of placing an order are unaffected by quantity ordered.
Purchase costs per unit are not affected by quantity discounts.

How well did you know this?
1
Not at all
2
3
4
5
Perfectly
4
Q

Everything else being equal, a noncallable bond will be priced in comparison to a callable bond so that the noncallable bond will provide:

Incorrect A.
a higher yield.

B.
a lower yield.

C.
the same yield.

D.
a yield 1% less.

A

Callable bonds reduce issuer risk by allowing the bonds to be called in if interest rates decline. The holder of callable bonds, however, is exposed to greater risk (i.e., loss of relatively high interest in a declining interest rate period).

In contrast, a noncallable bond is less risky for a bondholder, so it should sell at a lower yield.

How well did you know this?
1
Not at all
2
3
4
5
Perfectly
5
Q

A normal profit is:

Incorrect A.
the same as an economic profit.

B.
the same as a pure profit.

C.
an explicit or out-of-pocket cost.

D.
a cost of resources from an economic perspective.

A

Normal profit is the level of return required to keep resources in their current use (i.e., the cost of capital). Normal profits are included by economists (but not businessmen) in total costs.

Economic or pure profit is simply the difference between revenue and costs, including opportunity costs of all resources including capital.

How well did you know this?
1
Not at all
2
3
4
5
Perfectly
6
Q

The Sarbanes-Oxley Act changed the way financial reports are treated. What section of the act requires the CEO to review the financial statements?

A.
Section 202

Correct B.
Section 302

C.
Section 102

D.
Section 402

A

Correct B.

Section 302

How well did you know this?
1
Not at all
2
3
4
5
Perfectly
7
Q

The federal budget deficit is the:

A.
total accumulation of the federal government’s surpluses and deficits.

B.
excess of state, local, and federal spending over their revenues.

Correct C.
amount by which the federal government’s expenditures exceed its revenues in a given year.

D.
amount by which liabilities exceed assets on the federal government’s balance sheet.

A

The federal budget deficit is the amount by which the federal government’s expenditures exceed its revenues in a given year.

Total accumulation of the federal government’s surpluses and deficits over time is the national debt.
Excess of state, local, and federal spending over their revenues would represent aggregate government budget deficit.
The amount by which liabilities exceed assets on the federal government’s balance sheet represents a negative fund balance.

How well did you know this?
1
Not at all
2
3
4
5
Perfectly
8
Q

Which one of the following statements concerning cash flow determination for capital budgeting purposes is incorrect?

A.
Tax depreciation must be considered since it affects cash payments for taxes.

Correct B.
Book depreciation is relevant since it affects net income.

C.
Sunk costs are not incremental flows and should not be included.

D.
Net working capital changes should be included in cash flow forecasts.

A

Depreciation itself is not a cash outflow. Depreciation is relevant for capital budgeting purposes only because it affects the income upon which taxes must be assessed. Taxes are a cash outflow. Therefore, only tax depreciation is relevant. Book depreciation is not relevant despite its effect on net income.

How well did you know this?
1
Not at all
2
3
4
5
Perfectly
9
Q

Which of the following is a technique for assessing the potential effect of risk in a capital budgeting project?

A.
Sensitivity analysis

B.
Adjusting required rate of return

C.
Adjusting estimated future cash inflows

Correct D.
All of the answer choices are correct.

A

Sensitivity analysis involves testing the effects of various assumptions. Adjusting the required rate of return involves increasing the rate for more risky projects. Similarly, adjusting estimated future cash flows is to make them more conservative for more risky projects. Thus, all of these are techniques for dealing with risk in capital budgeting projects.

How well did you know this?
1
Not at all
2
3
4
5
Perfectly
10
Q

Lin Co. is buying machinery it expects will increase average annual operating income by $40,000. The initial increase in the required investment is $60,000, and the average increase in required investment is $30,000. To compute the accrual accounting rate of return, what amount should be used as the numerator in the ratio?

A.
$20,000

B.
$30,000

Correct C.
$40,000

D.
$60,000

A

Accounting rate of return = Increase in income ÷ Required investment

Increase in income (numerator) = $40,000

How well did you know this?
1
Not at all
2
3
4
5
Perfectly
11
Q

Enert Inc.’s current capital structure is shown as follows. This structure is optimal, and the company wishes to maintain it.

 Debt                       25%
 Preferred equity            5%
 Common equity              70% Enert's management is planning to build a $75 million facility that will be financed according to this desired capital structure. There is currently $15 million of cash that is available for capital expansion. The percentage of the $75 million that will come from a new issue of common stock is:

A.
52.50%.

B.
56.25%.

C.
70.00%.

Correct D.
56.00%.

A

To find the percentage that will come from a new stock issue, first assume that the $15 million available for capital expansion will be used. Then apply the 70% rate to the remaining $60 million. That results in $42 million, or 56% of the $75 million.

How well did you know this?
1
Not at all
2
3
4
5
Perfectly
12
Q

Which of the following types of bonds is most likely to maintain a constant market value?

Incorrect A.
Zero coupon

B.
Floating rate

C.
Callable

D.
Convertible

A

B.
Floating rate
With most bonds, interest rates are fixed, resulting in an identical interest payment for each payment period over the life of the bonds. As current market interest rates rise, the market value of the bonds will go down since that value is based upon the present value of the future cash flows related to the current market rate; therefore, most bonds expose the holder to interest rate risk.

Floating-rate bonds eliminate interest rate risk since the interest rate paid for a given payment period is based upon the prevailing rates in the current bond market; therefore, if interest rates rise, the interest payment will also increase. Since the market value of bonds is based upon the present value of future cash flows, the market value of floating-rate bonds will remain relatively constant.

How well did you know this?
1
Not at all
2
3
4
5
Perfectly
13
Q

A company purchased property that it expects to sell for $14,000 next year. The net present value of the investment is $1,000. The company is guaranteed an interest rate of 12% by the bank. What amount did the company pay for the property?

A.
$11,500

B.
$12,500

Incorrect C.
$13,000

D.
$13,500

A

The present value of $14,000 in one year discounted at 12% is 14,000/1.12 = $12,500. The net present value of $1,000 is the difference between this $12,500 present value and the cost of the investment therefore the cost of the investment must be $11,500.

How well did you know this?
1
Not at all
2
3
4
5
Perfectly
14
Q

A firm’s dividend policy may treat dividends either as the residual part of a financing decision or as an active policy strategy.

Treating dividends as the residual part of a financing decision assumes that:

Correct A.
earnings should be retained and reinvested as long as profitable projects are available.

B.
dividends are important to shareholders, and any earnings left over after paying dividends should be invested in high-return assets.

C.
dividends are relevant to a financing decision.

D.
dividends are costly, and the firm should retain earnings and issue stock dividends.

A

Treating dividends as the residual part of a financing decision assumes that earnings should be retained and reinvested as long as profitable projects are available.

Dividends do not need to be a focus of company strategy, since in a world with taxes, dividends can be problematic for both the company (dividends are not tax deductible) and the stockholder (dividends are taxable).

In addition, the transaction and other costs of financing by selling additional stock (as discussed by Rozeff) lead a company to prefer using internal funds (i.e., retained earnings) for expansion.

Masulis and Trueman suggest that firms will use internal funds to finance all investments that have high returns—younger firms will invest more and older firms will pay more dividends since profitable investment opportunities for older firms will be smaller relative to funds available.

How well did you know this?
1
Not at all
2
3
4
5
Perfectly
15
Q

According to COSO, which of the following serves as a starting point for control monitoring?

Documentation of the system.

A baseline understanding of the system.

An assessment of the system.

An understanding of the flow of transactions through the system.

A

n all situations, a baseline understanding of internal control system’s effectiveness in an area serves as the starting point for monitoring. This baseline understanding allows organizations to design ongoing and separate monitoring procedures. Monitoring may be considered as consisting of the following sequence of activities (monitoring-for-change control continuum):

1. Control baseline—establishing a starting point that includes a supported understanding of the existing internal control system.
2. Change identification—identifying through monitoring changes in internal control that are either necessary because of changes in the operating environment or have already taken place.
3. Change management—evaluating the design and implementation of the changes, and establishing a new baseline.
4. Control revalidation/update—periodically revalidating control operation when no known changes have occurred.

How well did you know this?
1
Not at all
2
3
4
5
Perfectly
16
Q

Kanban is:

Correct A.
a technique for managing a just-in-time (JIT) inventory system developed by the Japanese.

B.
a method of evaluating alternative credit policies developed by the Japanese.

C.
a method of determining the economic order quantity expressed in mathematical terms.

D.
a method of developing a relationship between sales and inventory used in forecasting.

A

Kanban is a technique for managing a just-in-time inventory system. The kanban is a tag attached to the storage container where component parts are kept. As a component is used, a kanban is placed in a box. Managers determine the number of kanbans needed to be in the box of the component part before a reorder of that part is necessary.

How well did you know this?
1
Not at all
2
3
4
5
Perfectly
17
Q

The following information data pertains to a manufacturing company:

Total sales $80,000
Total variable costs 20,000
Total fixed costs 30,000
What is the breakeven level in sales dollars?

A.
$30,000

Correct B.
$40,000

C.
$50,000

D.
$80,000

A

The contribution margin ratio is the contribution margin (sales of $80,000 less variable costs of $20,000, or $60,000) divided by the sales revenue of $80,000, or 0.75.

Breakeven revenue is found by dividing the fixed costs of $30,000 by the contribution margin ratio of 0.75, for breakeven sales revenue of $40,000.

How well did you know this?
1
Not at all
2
3
4
5
Perfectly
18
Q

Which of the following would be most impacted by the use of the percentage of sales forecasting method for budgeting purposes?

Correct A.
Accounts payable

B.
Mortgages payable

C.
Bonds payable

D.
Common stock

A

When forecasting using the percentage of sales method, a company would first determine a new sales figure. Other financial statement accounts would then be forecasted as a percentage of sales. Of the answers given, only accounts payable would fluctuate in proportion to sales (as sales increase, the trade accounts payable necessary to support inventory and overhead costs would increase also). Mortgages payable and bonds payable would decrease as principal is paid; any increase would be linked to a management decision not directly proportional to sales. Common stock, likewise, would increase or decrease according to management decisions to increase cash flow (selling stock) or to lessen outside control of the company (purchase treasury stock). Equity financing decisions are not necessarily proportional to sales.

How well did you know this?
1
Not at all
2
3
4
5
Perfectly
19
Q

The purpose of the TDRA (top-down risk assessment) is for the company to analyze the internal controls currently in place and to assess the effectiveness of those controls so as to avoid material misstatement in the company’s financial reporting. As part of that assessment process, which of the following items would be done?

Incorrect A.
The TDRA will focus on the identification and analysis of pertinent risks related to the achievement of the company’s objectives. The starting point for this assessment will be the lowest-level control, such as a process-level control (separation of duties and steps involved in a single task, such as issuing a credit for returned merchandise).

B.
As a starting point of the TDRA, management must conclude whether the danger of an internal control failure is high, medium, or low. After that decision is made, the focus then shifts to the areas suspected of being vulnerable.

C.
The focus of the TDRA assessment of internal controls is to ferret out all possible areas of error within the financial reporting process.

D.
As part of the TDRA, management will develop a list related to a particular account that would have a reasonable likelihood of material misstatement, focusing on problems that have been encountered in the past and the solutions that were developed to avoid such errors in the future.

A

The purpose of the TDRA is for the company to analyze the internal controls currently in place and to assess the effectiveness of those controls to avoid material misstatement in the firm’s financial reporting.

The focus of the assessment of internal controls will deal with significant (material) accounts.
The TDRA will focus on the identification and analysis of pertinent risks related to the achievement of the company’s objectives. The higher levels are examined first in the assessment process.
Based upon the identification and analysis of risks and the associated internal control to mitigate those risks, management needs to conclude whether the danger of an internal control failure is low, medium, or high. This step is taken after the internal controls in place have been assessed.

D.
As part of the TDRA, management will develop a list related to a particular account that would have a reasonable likelihood of material misstatement, focusing on problems that have been encountered in the past and the solutions that were developed to avoid such errors in the future.

How well did you know this?
1
Not at all
2
3
4
5
Perfectly
20
Q

Dartmoor Company’s budgeted sales for the coming year are $40,500,000, of which 80% are expected to be credit sales at terms of n/30. Dartmoor estimates that a proposed relaxation of credit standards would increase credit sales by 20% and increase the average collection period from 30 days to 40 days. Based on a 360-day year, the proposed relaxation of credit standards would result in an expected increase in the average accounts receivable balance of:

A.
$540,000.

B.
$2,700,000.

Correct C.
$1,620,000.

D.
$945,000.

A

The firm wants to relax credit standards and increase the average collection period. Previous sales are $40,500,000 and 80% of these, or $32,400,000, are credit sales. The new policies will increase these credit sales by 20%, to $38,880,000 ($32,400,000 × 1.20). Under the old system, the average day’s credit sales were $32,400,000 ÷ 360 or $90,000. Each day’s credit sales remained in accounts payable for 30 days, so the total amount in accounts payable under the old system was $90,000 × 30 days = $2,700,000. Under the new system, average day’s credit sales will be $38,880,000 ÷ 360, or $108,000. These daily sales will remain in accounts payable for 40 days, so the new accounts receivable balance will be $108,000 × 40 = $4,320,000. The expected increase in accounts receivable will be ($4,320,000 - $2,700,000) = $1,620,000.

How well did you know this?
1
Not at all
2
3
4
5
Perfectly
21
Q

Processing data through the use of simulated files provides an auditor with information about the operat­ing effectiveness of control policies and procedures. One of the techniques involved in this approach makes use of:

A.
controlled reprocessing.

Correct B.
an integrated test facility.

C.
input validation.

D.
program code checking.

A

An integrated test facility allows an auditor to introduce test data (simulated files) into an actual processing run to test the processing of that data. This provides evidence about operating effectiveness of the software.

“Controlled reprocessing” is incorrect because reprocessing the same data again with the same software provides no new information. “Input validation” is incorrect because input validation is a control that improves the accuracy of data entry, but does not provide information about control effectiveness. “Program code checking” is incorrect because manual program code checking in a complex system is a difficult task, sometimes impossible, which is more efficiently done by using test data in an integrated test facility.

How well did you know this?
1
Not at all
2
3
4
5
Perfectly
22
Q

A computer input control is designed to ensure that
Machine processing is accurate.
Only authorized personnel have access to the computer area.
Data received for processing are properly authorized and converted to machine-readable form.
Computer processing has been performed as intended for the particular application.

A

This answer is correct because input controls are designed to provide reasonable assurance that data received for processing by computer have been properly authorized, converted into machine sensible form and identified, and have not been lost, suppressed, added, duplicated, or otherwise improperly changed.

How well did you know this?
1
Not at all
2
3
4
5
Perfectly
23
Q
When considering disaster recovery, what type of backup facility involves an agreement between two organizations to aid each other in the event of disaster?
Cold site.
Hot site.
Reciprocal agreement.
Rollback.
A

Close
Online databases and master files systems

(1) Checkpoint—Similar to grandfather-father-son, but at certain points, “checkpoints,” the system makes a copy of the database and this “checkpoint” file is stored on a separate disk or tape. If a problem occurs the system is restarted at the last checkpoint and updated with subsequent transactions.
(2) Rollback—As a part of recovery, to undo changes made to a database to a point at which it was functioning properly.
(3) Backup facilities

a] Reciprocal agreement—An agreement between two or more organizations (with compatible computer facilities) to aid each other with their data processing needs in the event of a disaster. This is sometimes referred to as a mutual aid pact.
b] Hot site—A commercial disaster recovery service that allows a business to continue computer operations in the event of computer disaster. For example, if a company’s data processing center becomes inoperable, that enterprise can move all processing to a hot site that has all the equipment needed to continue operation. This is also referred to as a recovery operations center (ROC) approach.
c] Cold site—Similar to a hot site, but the customer provides and installs the equipment needed to continue operations. A cold site is less expensive, but takes longer to get in full operation after a disaster. This is sometimes referred to as an “empty shell” in that the “shell” is available and ready to receive whatever hardware the temporary user needs.
d] Internal site—Large organizations with multiple data processing centers sometimes rely upon their own sites for backup in the event of a disaster.

NOTE: Be aware that most approaches to control for catastrophic failures rely upon backup of the entire system in one form or another. Also, various combinations of the above approaches may be used.

How well did you know this?
1
Not at all
2
3
4
5
Perfectly
24
Q

What does integrated planning accomplish?

A.
Participation of stakeholders and affected departments

Incorrect B.
The creation of strategic planning

C.
Electronic commerce

D.
Business process design

A

Integrated planning provides for the participation of stakeholders with affected departments within an organization. This participation helps the organization to examine costs and benefits of a plan of action.

How well did you know this?
1
Not at all
2
3
4
5
Perfectly
25
Q

Which of the following structures refers to the collection of data for all vendors in a relational data base?

A.
Record

B.
Field

Correct C.
File

D.
Byte

A

A byte is a part of a field. A field is a part of a record. A record is a set of logically related data items that describes specific attributes of an entity, such as all payroll data relating to a single employee. Multiple records make up a file, so a collection of data from all vendors would be a file.

How well did you know this?
1
Not at all
2
3
4
5
Perfectly
26
Q

A validation check used to determine if a quantity ordered field contains only numbers is an example of:

Correct A.
an input control.

B.
an audit trail control.

C.
a processing control.

D.
a data security control.

A

A validation used to assure that a quantity ordered field allows input of only numerical data is an example of an input control.

Audit trail controls seek to make sure that a record of all relevant events and transactions has been recorded chronologically.
Processing controls relate to completeness and accuracy of data during processing (i.e., updating).
Data security controls restrict unauthorized individuals from access to and use of systems.

How well did you know this?
1
Not at all
2
3
4
5
Perfectly
27
Q

According to the Sarbanes-Oxley Act of 2002, which of the following statements is correct regarding an issuer’s audit committee financial expert?

A.
The issuer’s current outside CPA firm’s audit partner must be the audit committee financial expert.

Correct B.
If an issuer does not have an audit committee financial expert, the issuer must disclose the reason why the role is not filled.

C.
The issuer must fill the role with an individual who has experience in the issuer’s industry.

D.
The audit committee financial expert must be the issuer’s audit committee chairperson to enhance internal control.

A

The Sarbanes-Oxley Act of 2002 (SOX), also known as the Public Company Accounting Reform and Investor Protection Act, was enacted to develop new or enhanced standards for all U.S. public company boards, management, and public accounting firms.

One of its requirements is that an issuer should have an audit committee, and at least one of its members should be an individual with significant financial reporting expertise. If the firm does not have an audit committee financial expert, the issuer must disclose the reason why the role is not filled.

How well did you know this?
1
Not at all
2
3
4
5
Perfectly
28
Q

In general, it is more expensive for a company to finance with equity capital than with debt capital because:

A.
investors are exposed to greater risk with equity capital.

B.
the interest on debt is a legal obligation.

C.
equity capital is in greater demand than debt capital.

Incorrect D.
dividends fluctuate to a greater extent than interest rates.

A

Stockholders are the last to be paid, making their risk in the company greater. The greater degree of risk requires a greater reward. Bondholders, on the other hand, can expect a fixed return, known in advance, and therefore have a lower degree of risk.

How well did you know this?
1
Not at all
2
3
4
5
Perfectly
29
Q

The Sarbanes-Oxley Act of 2002 (SOX) requires that all publicly traded firms establish internal controls related to financial reporting that are documented, tested, and maintained for the purpose of preventing fraud. Per SOX, a company needs to do all of the following except:

A.
develop documentation of existing internal controls and procedures associated with financial reporting.

B.
test the effectiveness of the existing internal controls and procedures.

C.
provide information on deficiencies in the controls and/or documentation of those controls.

Correct D.
include all areas of potential risk to the misstatement of the financial statements in this documentation, testing, and reporting process.

A

Section 404 of the Sarbanes-Oxley Act of 2002 (SOX) requires that all publicly traded firms establish internal controls related to financial reporting that are documented, tested, and maintained. The purpose of these controls is to reduce the probabilities of corporate fraud. In order to be in compliance with SOX 404, a company needs to:

develop documentation of existing internal controls and procedures associated with financial reporting,
test the effectiveness of those controls and procedures, and
provide details on any deficiencies in the controls and/or documentation.
Although the initial response to SOX was to document, test, and report on both high- and low-risk areas related to financial reporting, the Public Company Accounting Oversight Board (PCAOB) developed Auditing Standard 5 (An Audit of Internal Control Over Financial Reporting That Is Integrated with an Audit of Financial Statements), approved by the SEC, that limits such activities. This standard takes a risk-based approach related to internal control documentation and testing, thus limiting the compliance measures necessary under SOX. SOX was never meant to create unnecessary compliance burdens on a company, but rather to protect shareholders’ investment from fraud.

How well did you know this?
1
Not at all
2
3
4
5
Perfectly
30
Q

Assume that each day a company writes and receives checks totaling $10,000. If it takes five days for the checks to clear and be deducted from the company’s account, and only four days for the deposits to clear, what is the float?

Correct A.
$10,000

B.
$(10,000)

C.
$50,000

D.
$25,000

A

The delay for the checks and deposits to clear the company’s account is called the “float.” During the float, the company has the use of the cash equal to the amount of checks written but does not have the use of deposits until they clear its bank.

Since it takes five days for the checks to clear (i.e., be deducted from the company’s account), the company has the use of $10,000 for five days or what is essentially a “loan” of $50,000. However, for the four days required for the deposits to clear, the company is losing the use of $10,000 × 4 or $40,000. Thus, the company enjoys a net 1-day float on the checks it writes of $10,000.

Thus, skillful management of the float requires the company to delay the coverage of checks as long as possible and to deposit checks received as quickly as possible.

How well did you know this?
1
Not at all
2
3
4
5
Perfectly
31
Q

The method for allocating service department costs that best recognizes the mutual services rendered to other service departments is the:

A.
dual-rate allocation method.

B.
direct allocation method.

C.
step-down allocation method.

Correct	D.	 	
linear algebra (reciprocal) allocation method.
A

The linear algebra or reciprocal allocation method recognizes reciprocity among service department by explicitly including the mutual services rendered among support departments.

The dual-rate allocation method is really a refinement of either the direct or step-down methods, depending upon how it is applied. In the dual-rate method, variable and fixed costs are allocated to departments in a two-step process, variable costs on current use and fixed costs on a long-term, maximum capacity basis. This method may not recognize any reciprocity of services among service departments.
The direct allocation method allocates the cost of service departments directly to the production departments without any intermediate allocations to other service departments. Thus, this method does not recognize any reciprocity of services among service departments.
The step-down allocation method allocates service department costs to other service departments and production departments usually starting with the service department that provides the most service to other service departments. This method allows for partial recognition of reciprocity of services among service departments.

How well did you know this?
1
Not at all
2
3
4
5
Perfectly
32
Q

Which of the following changes would result in the highest present value?

A.
A $100 decrease in taxes each year for four years

B.
A $100 decrease in the cash outflow each year for three years

C.
A $100 increase in disposal value at the end of four years

Incorrect D.
A $100 increase in cash inflow each year for three years

A

A.
A $100 decrease in taxes each year for four years

Two general rules can be developed related to present value calculations:

Increases in cash inflows (decreases in cash outflows) will result in higher present values, all else being equal.
The earlier the cash inflows (the later the cash outflows) the higher the present value, all else being equal.
Increases in cash flows can be the result of increased revenues, increases in other cash inflows, or decreases in cash outflows. Of the answer choices available, only one (a $100 decrease in taxes each year for four years) results in increased net cash inflows for each of four years, thus resulting in the highest present value.

How well did you know this?
1
Not at all
2
3
4
5
Perfectly
33
Q

Which of the following is a characteristic of a flexible budget?

Correct A.
Provides budgeted numbers for various activity levels

B.
Allows for modification during the budgeted period

C.
Isolates the impact of variable costs on the overall budget

D.
Can be utilized by several product divisions

A

A flexible budget is developed using the static budget standards for any activity level within the relevant range. Amounts are calculated using the actual output level based on the standard cost and usage per unit for variable items, and the identical dollar amounts for the fixed costs in the static budget are used in the flexible budget. A flexible budget prepared outside the relevant range would not necessarily be accurate.

How well did you know this?
1
Not at all
2
3
4
5
Perfectly
34
Q

Limitations of an activity-based costing system include which of the following?

A.
Control of overhead costs is enhanced.

B.
Activity-based costing systems are less reliable.

Correct C.
The expense of obtaining cost data is relatively high.

D.
It eliminates arbitrary assignment of overhead costs

A

Activity-based costing (ABC) accumulates costs in cost pools related to separately identified activities that are allocated based on cost drivers. One limitation of an ABC system is that the cost of determining the pools and drivers is higher than a traditional allocation system based on one base, such as direct labor costs.

Enhanced control of overhead costs and elimination of arbitrary assignment of overhead costs are incorrect because they are advantages of ABC, not limitations. ABC systems are usually more reliable than traditional cost accounting systems because overhead costs are allocated to products in a more meaningful manner.

How well did you know this?
1
Not at all
2
3
4
5
Perfectly
35
Q

Spear Corp. had sales of $2,000,000, a profit margin of 11%, and assets of $2,500,000. Spear decided to reduce its debt ratio to 0.40 from 0.50 by selling new common stock and using the proceeds to repay principal on some outstanding long-term debt. After the refinancing, what is Spear’s return on equity?

A.
3.5%

Incorrect B.
5.3%

C.
14.7%

D.
22.9%

A

C.
14.7%

Assets total $2.500,000. The sum of liabilities and equity also equals $2,500,000 (Assets=Liab+OE)2.5=1M+1.5 The company will have debt equal to 40% of the $2,500,000 or $1,000,000. That leaves equity of $1,500,000. Since the profit is 11% of the $2,000,000 sales, the company has a profit of $220,000. Return on equity is net income divided by equity and profit of $220,000 divided by equity of $1,500,000 gives return on equity of 14.7%.

How well did you know this?
1
Not at all
2
3
4
5
Perfectly
36
Q

Augusta, Inc., expects manufacturing and sales of 70,000 units of product Maggie, its only product, to occur evenly over a 10-week period. Augusta pays for materials in the week following use. The balance of accounts payable for materials at the beginning of the 10-week period is $40,000. There are no beginning inventories. The fol­lowing information pertains to product Maggie for the 10-week period:

Sales price $11 per unit
Materials $3 per unit
Manufacturing conversion costs—Fixed $210,000
Variable $2 per unit
Selling and administrative costs—Fixed $45,000
Variable $1 per unit

Using absorption costing, what is Augusta’s budgeted income for the period?

Correct A.
$95,000

B.
$140,000

C.
$305,000

D.
$350,000

A

Absorption costing is a method of costing in which manufacturing fixed costs are treated as product costs and assigned to the units produced. Fixed costs follow the units through work-in-process and finished goods as inventoriable costs and are expensed through cost of goods sold when the units are sold.

Dividing 70,000 units to be produced into $210,000 of manufacturing fixed costs gives $3 per unit produced. Production costs include materials of $3, other variable costs of $2, and fixed costs of $3, for a total of $8 per unit.
Selling 70,000 units at $11 each results in revenue of $770,000.
Cost of sales is 70,000 units at $8, or $560,000.
This leaves a gross profit of $210,000 ($770,000 − $560,000).
Selling and administrative costs include the variable costs of 70,000 units at $1 each, or $70,000, plus fixed costs of $45,000, for a total of $115,000.
Subtracting $115,000 of selling and administrative costs from gross profit of $210,000 leaves a net profit of $95,000.

How well did you know this?
1
Not at all
2
3
4
5
Perfectly
37
Q

Which of the following is responsible for making sure that the information system operates efficiently and effectively?

A.
Network manager

B.
Security management

Correct C.
Systems administrator

D.
Change management

A

The systems administrators make sure the information system operates efficiently and effectively. The network managers make sure the networks operate continuously and properly and that all applicable devices are linked to the organization’s internal and external networks. Security management makes sure all system components are secure and protected from any and all internal as well as external threats.

How well did you know this?
1
Not at all
2
3
4
5
Perfectly
38
Q

Which of the following tasks would be included in a document flowchart for processing cash receipts?

Correct A.
Compare control and remittance totals

B.
Record returns and allowances

C.
Authorize and generate an invoice

D.
Authorize and generate a voucher

A

One of the key tasks in processing cash receipts is to compare control totals and remittance totals. Such a task may be represented by a box on a flowchart describing the process. Tasks such as recording returns and allowances, authorizing and generating invoices, and authorizing and generating vouchers are not part of processing cash receipts.

How well did you know this?
1
Not at all
2
3
4
5
Perfectly
39
Q

The following information pertains to Quest Co.’s Gold Division:

  Sales                       $311,000
  Variable cost                250,000
  Traceable fixed costs         50,000
  Average invested capital      40,000
  Imputed interest rate            10%

Quest’s return on investment was:

A.
10.00%.

B.
13.33%.

Correct C.
27.50%.

D.
30.00%.

A

Division net income equals $311,000 sales less variable costs of $250,000 and traceable fixed costs of $50,000, or $11,000.

Return on investment is net income ($11,000) divided by investment ($40,000), or 27.5%.

How well did you know this?
1
Not at all
2
3
4
5
Perfectly
40
Q

The capital budgeting model that is generally considered the best model for long-range decision making is the:

A.
payback model.

B.
accounting rate of return model.

C.
unadjusted rate of return model.

Correct D.
discounted cash flow model.

A

The capital budgeting model that is generally considered the best model for long-range decision making is the discounted cash flow model because the time value of money (present and future values) is considered.

The payback model and accounting rate of return model are unadjusted rate of return models which do not consider the time value of money.

How well did you know this?
1
Not at all
2
3
4
5
Perfectly
41
Q

A distributed processing environment would be most beneficial in which of the following situations?

Correct A.
Large volumes of data are generated at many locations and fast access is required.

B.
Large volumes of data are generated centrally and fast access is not required.

C.
Small volumes of data are generated at many locations, fast access is required, and summaries of the data are needed promptly at a central site.

D.
Small volumes of data are generated centrally, fast access is required, and summaries are needed monthly at many locations.

A

A distributed/decentralized processing environment works best when significant volumes of data are generated at many remote locations and the user requires near-immediate access to the data. This type of processing environment will allow for quick access to the data as opposed to having that information generated at many locations and processed at a centralized location.

How well did you know this?
1
Not at all
2
3
4
5
Perfectly
42
Q

Which of the following security controls might prevent unauthorized access to sensitive data via an unattended data terminal directly connected to a mainframe?

A.
Use of a screen saver

Incorrect B.
Use of passwords to identify users

C.
Encryption of data files

D.
Automatic log-off of inactive users

A

Automatic log-off of inactive users would help to prevent unauthorized access to sensitive data displayed on an unattended terminal. This could create a problem, however, if the time interval for non-use log-off was set too short.

Using a screen saver is not a solution. Simply moving the mouse or keyboard would restore the sensitive data.
The use of a password would not prevent viewing of sensitive data already displayed on the monitor.
Encryption of data files would not prevent viewing of data displayed on a monitor.

How well did you know this?
1
Not at all
2
3
4
5
Perfectly
43
Q

Which of the following is not one of the five components of internal control identified in SAS 78 as originally included in the COSO Report?

A.
Control environment

Correct B.
Accounting system

C.
Control activities

D.
Information and communication

A

Accounting system is not one of the five components of internal control identified in SAS 78. This was one of the elements of internal control identified in SAS 55, but this part of SAS 55 has been superseded by SAS 78. The five components are control environment, risk assessment, control activities, information and communication, and monitoring activities.

How well did you know this?
1
Not at all
2
3
4
5
Perfectly
44
Q

The use of activity-based costing normally results in:

A.
substantially greater unit costs for low-volume products than is reported by traditional product costing.

Incorrect B.
substantially lower unit costs for low-volume products than is reported by traditional product costing.

C.
decreased set-up costs being charged to low-volume products.

D.
equalizing set-up costs for all product lines.

A

In the past, conventional costing techniques assigned indirect manufacturing costs to a single (or few) cost pool(s) and allocated those costs based on a single (or few) allocation base(s). The result was that both high and low volume products received the same unit “dosage” of indirect costs.

Activity-based costing (ABC) utilizes multiple cost pools for accumulating indirect costs. These costs are then allocated to products in proportion to the respective products’ consumption of those resources. Thus, low volume products which consume more resources (such as set-up costs) receive substantially greater unit costs than they would receive under traditional costing.

How well did you know this?
1
Not at all
2
3
4
5
Perfectly
45
Q

A disk storage unit is preferred over a magnetic tape drive because the disk storage unit:

A.
has nine tracks.

B.
offers sequential access to data files.

C.
offers random access to data files.

Incorrect D.
is a cheaper medium for data storage.

A

Access to data takes less time with disk storage than with magnetic tape storage.

Consider how data is stored on magnetic tape. Blocks of data files are arranged linearly along the entire length of the tape. In order to move from a read location at or near the beginning of the tape to a read location near the end of the tape, it is necessary to travel over all tape between the two read locations.

On the other hand, if disk storage is used, it is possible to jump directly from one read location to another. This is possible because disk storage offers random access to data files.

How well did you know this?
1
Not at all
2
3
4
5
Perfectly
46
Q

Yola Co. manufactures one product with a standard direct labor cost of four hours at $12.00 per hour. During June, 1,000 units were produced using 4,100 hours at $12.20 per hour. The unfavorable direct labor efficiency variance was:

A.
$1,220.

Correct B.
$1,200.

C.
$820.

D.
$400.

A

LEV= SR (SR-AH)
12(4,000-4,100)
=1,200

The labor efficiency (usage) variance is the difference between standard cost of actual hours and the standard cost of the budgeted labor hours.

Standard cost (at $12 per hour) of the actual hours of 4,100 hours was $49,200.
Standard cost (at $12 per hour) of the standard hours (4 hours × 1,000 units) was $12 × 4,000 hours, or $48,000.
Since the actual cost was more than the budgeted cost ($49,200 − $48,000), the labor efficiency (usage) variance was $1,200 unfavorable.
How well did you know this?
1
Not at all
2
3
4
5
Perfectly
47
Q

The following information is available on Crain Co.’s two product lines:

                       Chairs      Tables
                      ---------   --------- Sales                     $180,000    $ 48,000 Variable costs             (96,000)    (30,000)
                      ---------   --------- Contribution margin       $ 84,000    $ 18,000 Fixed costs:   Avoidable                (36,000)    (12,000)   Unavoidable              (18,000)    (10,800)
                      ---------   --------- Operating income (loss)   $ 30,000    $ (4,800)
                      =========   ========= Assuming that the table line is discontinued, and the factory space previously used to make tables is rented for $24,000 per year, operating income will increase by what amount?
A

When looking at business segment issues, relevant items are those that are avoidable. In other words, revenue, variable costs, and avoidable fixed costs are relevant. The allocated common costs are not relevant since they will continue even if the table line is discontinued. If the segment margin is positive, this means that the segment is making a contribution toward the common costs of the organization. Reordering the information given in the problem, it can be seen that the table segment has a positive segment margin.

                        Total      Chairs      Tables
                      ---------   ---------   --------- Sales                     $228,000    $180,000    $ 48,000 Variable costs            (126,000)    (96,000)    (30,000)
                      ---------   ---------   --------- Contribution margin       $102,000    $ 84,000    $ 18,000 Avoidable fixed costs      (48,000)    (36,000)    (12,000)
                      ---------   ---------   --------- Segment margin            $ 54,000    $ 48,000    $  6,000
                                  =========   ========= Unavoidable fixed costs    (28,800)
                      ---------  Operating income          $ 25,200
                      ========= What items are relevant if the table line is discontinued? The table-line segment margin would be eliminated; however, factory space that would be freed up could then be rented for $24,000 per year (opportunity cost).

Solution:

Lost contribution to overhead $(6,000)
Rental opportunity 24,000
——–
Increase in operating income $18,000

How well did you know this?
1
Not at all
2
3
4
5
Perfectly
48
Q

FASB ASC 820 discusses the disclosures that need to be presented in financial statements related to fair value measurements. A basic goal of these disclosures is to provide:

Correct A.
enough information to the user of the financial statements so that the inputs used in the fair value measurement can be assessed.

B.
evidence that the fair value measurement is accurate.

C.
a reconciliation of the beginning and ending balances of the various fair value measurements for all nonrecurring fair value measurements.

D.
information on the effect of the fair value measurements on the retained earnings balance.

A

Per FASB ASC 820, the disclosures related to fair value measurements contained in the financial statements need to:

allow for enough information to be provided so that the user of that information can do a reasonable assessment of the inputs used to develop the fair value.
provide for enough information so that the use of significant unobservable inputs (Level 3 inputs) can be assessed.
Although a reconciliation of the beginning and ending balances of balance sheet items using a fair value measurement needs to be provided for all recurring measurements, a reconciliation only needs to be done for nonrecurring fair value measurements when significant unobservable inputs are used (Level 3).

How well did you know this?
1
Not at all
2
3
4
5
Perfectly
49
Q

Madi and Molly, Inc., produces lumber, with an average pine tree having 61% of its trunk producing construction grade lumber. Of the remaining 39%, 22% is used for producing wood products for a local furniture manufacturer, and 17% goes to waste. Therefore, Madi and Molly’s lumber usage rate is 83%. In 20X1 the local furniture manufacturer closed, leaving no viable prospects for Madi and Molly to sell the 22% of the wood allocated to the furniture manufacturer. What improvement initiative should Madi and Molly implement to have its lumber usage rate rise back above 80%? You can assume that an increase in the lumber usage rate will replace 100% of the revenue lost from the furniture manufacturer.

A.
Lean production

B.
Business excellence framework

C.
Six Sigma

Incorrect D.
Business process re-engineering

A

Lean manufacturing, which is often known simply as “lean,” is a production practice and methodology that focuses on reduction of the seven wastes (overproduction, waiting time, transportation, processing, inventory, motion, and scrap) in manufacturing products.

Upon the loss of sales to the furniture manufacturer, Madi and Molly were looking at waste of 39% (100% - 61% for lumber = 39%) as well as a loss in revenue. By implementing lean production tools to achieve 80% conversion of the raw materials (pine trees) into lumber, Madi and Molly would see its revenue stream remain even, while its waste was reduced from 39% to 20%. While it is true that waste was originally 17% before the loss of the furniture client, Madi and Molly’s decision to implement lean production tools must be based upon the current production data, which showed a 39% waste factor.

How well did you know this?
1
Not at all
2
3
4
5
Perfectly
50
Q

In a large organization, the biggest risk in not having an adequately staffed information center help desk is:

A.
increased difficulty in performing application audits.

B.
inadequate documentation for application systems.

C.
increased likelihood of use of unauthorized program code.

Correct D.
persistent errors in user interaction with systems.

A

The biggest risk in not having an adequately staffed help desk is that users will unknowingly persist in making errors in their interaction with the information systems.

Application audits should be about the same difficulty with or without an adequately staffed help desk.
Preparation of documentation is a development function, not a help desk function.
The likelihood of use of unauthorized program code is a function of change control, not a help desk.

How well did you know this?
1
Not at all
2
3
4
5
Perfectly
51
Q

Box Co. uses regression analysis to estimate the functional relationship between an independent variable (cost driver) and overhead cost. Assume that the following equation is being used:

y = A + Bx
What is the symbol for the independent variable?

A.
y

Correct B.
x

C.
Bx

D.
A

A

Linear regression with two variables takes the form y = Bx + A.

A is the y intercept.
B is the slope of the line. (Linear means a straight line, so the slope of the line is the same at any point on the line.)
y is the dependent variable since it is determined based on the other three terms.
x is the independent variable since it is an input to the equation, not based on the other terms.
When estimating a cost function (y), x is the cost driver that determines the value of y.

How well did you know this?
1
Not at all
2
3
4
5
Perfectly
52
Q

Trendy Co. produced and sold 30,000 backpacks during the last year at an average price of $25 per unit. Unit variable costs were the following:

Variable manufacturing costs $ 9
Variable selling and administrative costs 6

Total $15
Total fixed costs were $250,000. There was no year-end work-in-process inventory. If Trendy had spent an additional $15,000 on advertising, then sales would have increased by $30,000. If Trendy had made this investment, what change would have occurred in Trendy’s pretax profit?

A.
$3,000 increase

Incorrect B.
$4,200 increase

C.
$3,000 decrease

D.
$4,200 decrease

A

Contribution margin per unit was $10 ($25 sales price less variable costs of $9 and $6). The contribution margin ratio was 40% ($10 contribution per unit divided by $25 sales revenue per unit).

Additional sales of $30,000 would have increased the contribution margin by $12,000 ($30,000 × 0.40). The increased contribution margin of $12,000 fails to cover the advertising costs of $15,000, leaving a decrease of $3,000 in pretax profit.

How well did you know this?
1
Not at all
2
3
4
5
Perfectly
53
Q

Sago Co. uses regression analysis to develop a model for predicting overhead costs. Two different cost drivers (machine hours and direct materials weight) are under consideration as the independent variable. Relevant data were run on a computer using one of the standard regression programs, with the following results:

                      Machine   Direct Materials
                       Hours         Weight
                      -------   ---------------- Y intercept coefficient    2,500         4,600 B coefficient                5.0           2.6 R2                          0.70          0.50

Which regression equation should be used?

A.
Y = 2,500 + 5.0X

Incorrect B.
Y = 2,500 + 3.5X

C.
Y = 4,600 + 2.6X

D.
Y = 4,600 + 1.3X

A

Coefficient of determination is a measure of the extent to which the independent variable accounts for the variation of the dependent variable (i.e., the amount of variation in y that is explained by x). It is the measure of how well the regression line fits the actual data points. The symbol for the correlation coefficient is R and the coefficient of determination is r squared (R2). Values of R range between -1 and 1. The closer the value is to 1, the greater the association (correlation) between the two variables.

Since the coefficient of determination (R2) is greater for machine hours than for direct materials weight (0.7 instead of 0.5), there is a stronger relationship between machine hours (the independent variable) and the resulting cost than there is between the resulting cost and direct materials weight. Thus, the resulting cost can be best estimated by substituting machine hours for X in the equation Y = 2,500 + 5.0X.

How well did you know this?
1
Not at all
2
3
4
5
Perfectly
54
Q

Williams, Inc., is interested in measuring its overall cost of capital and has gathered the following data. Under the terms described as follows, the company can sell unlimited amounts of all instruments.

Williams can raise cash by selling $1,000, 8%, 20-year bonds with annual interest payments. In selling the issue, an average premium of $30 per bond would be received, and the firm must pay flotation costs of $30 per bond. The after-tax cost of funds is estimated to be 4.8%.
Williams can sell 8% preferred stock at par value, $105 per share. The cost of issuing and selling the preferred stock is expected to be $5 per share.
Williams’ common stock is currently selling for $100 per share. The firm expects to pay cash dividends of $7 per share next year, and the dividends are expected to remain constant. The stock will have to be underpriced by $3 per share, and flotation costs are expected to amount to $5 per share.
Williams expects to have available $100,000 of retained earnings in the coming year; once these retained earnings are exhausted, the firm will use new common stock as the form of common stock equity financing.
Williams’ preferred capital structure is long-term debt, 30%; preferred stock, 20%; and common stock, 50%.
The firm’s weighted average cost of capital would be:

A.
4.8%.

Correct B.
6.6%.

C.
6.8%.

D.
7.3%.

A

he weighted cost of capital is 6.6%.

Step 1: Calculate the after-tax cost of each source of capital.

The cost of long-term debt, after tax, is given at 4.8%.
The cost of new preferred stock can be calculated as:
kpm = D / (PO - u - f), or kpm = 8.40 / (105 - 0 - 5) = 8.4%
Where:
D = Annual dividend, or 0.08 × $105 (the par value), or $8.4
PO = Selling price to the public of the new issue
u = Underpricing
f = Flotation cost per share
New equity consists of retained earnings and/or new issues of common stock. In this case, 50% of the 200,000 of total new funds must come from equity. Since the firm has $100,000 in retained earnings, the relevant cost of new equity is the cost of retained earnings, 7 ÷ 100 + 0%, or 7.0%.
Step 2: Calculate the Weighted Average Cost of Capital:

 Source        After-Tax Cost  x  Weight  =
 ------        --------------     ------   a. L-T Debt           .048       x   .30    =  .0144   b. Pref. Stock        .084       x   .20    =  .0168   c. Ret. Earnings      .070       x   .50    =  .0350
                                             -----
         Weighted Average Cost of Capital =  .066 or 6.6%
How well did you know this?
1
Not at all
2
3
4
5
Perfectly
55
Q

Which of the following is most useful when risk is being prioritized?

A.
Low- and high-probability exposures

B.
Low- and high-degree loss exposures

Correct C.
Expected value

D.
Uncontrollable risks

A

Expected value is the sum of the outcomes (payoff) of each event multiplied by the probability of each event occurring. It combines the likelihood of each outcome with the payoff of that outcome, and so is a way of prioritizing alternatives while considering risk. None of the other answer choices consider both the likelihood and payoff of each alternative course of action.

How well did you know this?
1
Not at all
2
3
4
5
Perfectly
56
Q

Which of the following statements is correct concerning the security of messages in an electronic data interchange (EDI) system?

A.
Removable drives that can be locked up at night provide adequate security when the confidentiality of data is the primary risk.

B.
Message authentication in EDI systems performs the same function as segregation of duties in other information systems.

Correct C.
Encryption performed by a physically secure hardware device is more secure than encryption performed by software.

D.
Security at the transaction phase in EDI systems is not necessary because problems at that level will be identified by the service provider.

A

Correct C.

Encryption performed by a physically secure hardware device is more secure than encryption performed by software.

How well did you know this?
1
Not at all
2
3
4
5
Perfectly
57
Q

Wong Company utilizes both strategic planning and operational budgeting. Which one of the following items would normally be considered in a strategic plan?

A.
Setting a target of 12% return on sales

B.
Maintaining the image of the company as the industry leader

Incorrect C.
Setting a market price per share of stock outstanding

D.
Distributing monthly reports for departmental variance analysis

A

Strategic planning is the general planning done by top management that charts a company’s course through its operating environment. Maintaining the image of a company as an industry leader is therefore a strategic goal.

The other answer choices, listed below, are all operational or budgetary goals:

Setting a target of 12% return on sales
Setting a market price per share of stock outstanding
Distributing monthly reports for departmental variance analysis

How well did you know this?
1
Not at all
2
3
4
5
Perfectly
58
Q

The controller of Gray, Inc., has decided to use ratio analysis to analyze business cycles for the past two years in an effort to identify seasonal patterns. Which of the following formulas should be used to compute percentage changes for account balances for Year 1 to Year 2?

A.
(Prior balance - Current balance) / Current balance

B.
(Prior balance - Current balance) / Prior balance

Incorrect C.
(Current balance - Prior balance) / Current balance

D.
(Current balance - Prior balance) / Prior balance

A

The question concerns the change for Year 2 compared to Year 1. The amount of change is the increase or decrease from the prior year to the current year, equal to the current balance less the prior balance. The percentage change is calculated by dividing the amount of change by the beginning balance.

How well did you know this?
1
Not at all
2
3
4
5
Perfectly
59
Q

Data input validation routines include:

A.
terminal logs.

Correct B.
hash totals.

C.
backup controls.

D.
access logs.

A

Data input validation is the verification of accurate input of data which is an input control (a type of application control). A hash total is a kind of input control where some set nonfinancial numbers not normally totaled (such as invoice numbers or employee identification numbers) are totaled by the system after input and are compared to the total generated by the documents themselves. Terminal logs and access logs are all examples of access controls. A backup control is a kind of corrective control.

How well did you know this?
1
Not at all
2
3
4
5
Perfectly
60
Q

Data input validation also include?

A

Validation methods[edit]
Allowed character checks Checks that ascertain that only expected characters are present in a field. For example a numeric field may only allow the digits 0-9, the decimal point and perhaps a minus sign or commas. A text field such as a personal name might disallow characters such as , as they could be evidence of a markup-based security attack. An e-mail address might require at least one @ sign and various other structural details. Regular expressions are effective ways of implementing such checks. (See also data type checks below)

Batch totals Checks for missing records. Numerical fields may be added together for all records in a batch. The batch total is entered and the computer checks that the total is correct, e.g., add the ‘Total Cost’ field of a number of transactions together.

Cardinality check Checks that record has a valid number of related records. For example if Contact record classified as a Customer it must have at least one associated Order (Cardinality > 0). If order does not exist for a “customer” record then it must be either changed to “seed” or the order must be created. This type of rule can be complicated by additional conditions. For example if contact record in Payroll database is marked as “former employee”, then this record must not have any associated salary payments after the date on which employee left organization (Cardinality = 0).

Check digits Used for numerical data. An extra digit is added to a number which is calculated from the digits. The computer checks this calculation when data are entered. For example the last digit of an ISBN for a book is a check digit calculated modulus 10.[3]

Consistency checks Checks fields to ensure data in these fields corresponds, e.g., If Title = “Mr.”, then Gender = “M”.

Control totals This is a total done on one or more numeric fields which appears in every record. This is a meaningful total, e.g., add the total payment for a number of Customers.

Cross-system consistency checks Compares data in different systems to ensure it is consistent, e.g., The address for the customer with the same id is the same in both systems. The data may be represented differently in different systems and may need to be transformed to a common format to be compared, e.g., one system may store customer name in a single Name field as ‘Doe, John Q’, while another in three different fields: First_Name (John), Last_Name (Doe) and Middle_Name (Quality); to compare the two, the validation engine would have to transform data from the second system to match the data from the first, for example, using SQL: Last_Name || ‘, ‘ || First_Name || substr(Middle_Name, 1, 1) would convert the data from the second system to look like the data from the first ‘Doe, John Q’

Data type checks Checks the data type of the input and give an error message if the input data does not match with the chosen data type, e.g., In an input box accepting numeric data, if the letter ‘O’ was typed instead of the number zero, an error message would appear.

File existence check Checks that a file with a specified name exists. This check is essential for programs that use file handling.

Format or picture check Checks that the data is in a specified format (template), e.g., dates have to be in the format DD/MM/YYYY. Regular expressions should be considered for this type of validation.

Hash totals This is just a batch total done on one or more numeric fields which appears in every record. This is a meaningless total, e.g., add the Telephone Numbers together for a number of Customers.

Limit check Unlike range checks, data are checked for one limit only, upper OR lower, e.g., data should not be greater than 2 (

How well did you know this?
1
Not at all
2
3
4
5
Perfectly
61
Q

An incentive compensation program will:

A.
give all employees a bonus.

B.
lead to too much change.

C.
give only management bonuses.

Correct D.
lead employees toward the goals of the organization.

A

An incentive compensation program should be set up for the sake of leading employees toward accomplishing the goals of the organization. It is not the purpose of the program to give bonuses at will.

How well did you know this?
1
Not at all
2
3
4
5
Perfectly
62
Q

Cost allocation is the process of assigning indirect costs to a cost object. The indirect costs are grouped in cost pools and then allocated by a common allocation base to the cost object. The base that is employed to allocate a homogeneous cost pool should:

Correct A.
have a cause-and-effect relationship with the cost items in the cost pool.

B.
assign the costs in the pool uniformly to cost objects even if the cost objects use resources in a nonuniform way.

C.
be a nonfinancial measure (e.g., number of setups) because a nonfinancial measure is more objective.

D.
have a high correlation with the cost items in the cost pool as the sole criterion for selection.

A

All costs in a homogeneous cost pool should have a cause-and-effect relationship with the base that is employed for allocation.

An averaged or smoothed allocation may result in significant under or over allocation of costs.
Both financial and nonfinancial measures may be used as allocation bases.
Economic plausibility as well as high correlation are desirable for allocation bases.

How well did you know this?
1
Not at all
2
3
4
5
Perfectly
63
Q

Multiple regression differs from simple regression in that it:

A.
provides an estimated constant term.

B.
has more dependent variables.

C.
allows the computation of the coefficient of determination.

Correct D.
has more independent variables

A

Regression analysis seeks to identify change in a dependent variable (such as cost) related to change in an independent variable (such as a cost driver). Simple regression estimates a relationship between one dependent variable and one independent variable. On the other hand, multiple regression estimates a relationship between one dependent variable and two or more independent variables. Thus, multiple regression has more independent variables than does simple regression.

How well did you know this?
1
Not at all
2
3
4
5
Perfectly
64
Q

Cyclical fluctuations, random variations, seasonal variations, and secular trend are all components of:

A.
exponential smoothing.

B.
learning curve analysis.

C.
sales forecasting.

Correct D.
time series analysis.

A

Time series analysis focuses on evaluation of trends over time. It may entail several components including seasonal variation and secular trends.

How well did you know this?
1
Not at all
2
3
4
5
Perfectly
65
Q

Which of the following types of budgets is the last budget to be produced during the budgeting process?

Correct A.
Cash

B.
Capital

C.
Cost of goods sold

D.
Marketing

A

A cash budget is the last budget to be prepared in the budgeting process because the other steps must be completed before the effects of each part on cash can be estimated.

The steps to prepare a master budget are:

develop a sales forecast,
determine the desired level of finished goods inventory,
prepare a purchases or production budget,
estimate selling, administrative, and other general expenses,
organize the preceding information into an income statement, and
prepare a cash forecast.

How well did you know this?
1
Not at all
2
3
4
5
Perfectly
66
Q

In preparing the annual profit plan for the coming year, Wilkens Company wants to determine the cost behavior pattern of the maintenance costs. Wilkens has decided to use linear regression by employing the equation y = a + bx for maintenance costs. The prior year’s data regarding maintenance hours and costs, and the results of the regression analysis are given here.

                 Hours      Maintenance
              of Activity      Costs
              -----------   -----------
     Sum         4,480        $43,200
     Average       400          3,600
Average cost per hour = $9.00; a = 684.65; b = 7.2884; Standard error of a = 49.515; Standard error of b = .12126; Standard error of the estimate = 34.469; r2 = .99724.

Based upon the data derived from the regression analysis, 420 maintenance hours in a month would mean the maintenance costs (rounded to the nearest dollar) would be budgeted at:

A.
$3,780.

B.
$3,790.

Correct C.
$3,746.

D.
$3,756.

A

In this problem, hours of activity is the independent variable (x) with maintenance costs (y) being dependent upon the level of activity achieved. Using the standard simple regression equation and the other information given:

                     y = a + bx
                     y = 684.65 + 7.2884x If maintenance activity (x) in a month is 420 hours, then maintenance costs for the month would be:

                     y = 684.65 + 7.2884(420)
                       = 684.65 + 3061.13
                       = 3745.78
                       = $3,746
How well did you know this?
1
Not at all
2
3
4
5
Perfectly
67
Q

Probability (risk) analysis is:

A.
used only for situations involving five or fewer possible outcomes.

B.
used only for situations in which the summation of probability weights is greater than one.

Correct C.
an extension of sensitivity analysis.

D.
incompatible with sensitivity analysis.

A

Sensitivity analysis determines how the results will change if the original data or the underlying assumptions change. It is the process of identifying the data changes that alter optimal solutions and the decisions made based on that solution. Probability analysis combines the likelihood of various outcomes with sensitivity analysis.

Probability analysis can be used with an infinite number of outcomes, and 1.00 is the largest possible probability. It is helpful to combine probability analysis with sensitivity analysis to evaluate the sensitivity of various outcomes to risk.

How well did you know this?
1
Not at all
2
3
4
5
Perfectly
68
Q

Jones Corp. had an opportunity to use its capacity to produce an extra 5,000 units with a contribution margin of $5 per unit, or to rent out the space for $10,000. What was the opportunity cost of using the capacity?

A.
$35,000

B.
$25,000

C.
$15,000

Correct D.
$10,000

A

Opportunity costs are the earnings that could have been received had the storage been used in their best alternative use, such as foregone rental income. The company could have earned $10,000 renting out the space, and they have lost that opportunity by using the space for production. It may have been the correct decision, however, because they could earn more by producing in the space than by renting the space. Even if it is the correct decision, they have still lost the opportunity to earn the $10,000 from rental.

How well did you know this?
1
Not at all
2
3
4
5
Perfectly
69
Q

Which of the following inputs would be most beneficial to consider when management is developing the capital budget?

A.
Supply/demand for the company’s products

Incorrect B.
Current product sales prices and costs

C.
Wage trends

D.
Profit center equipment requests

A

Capital budgeting involves management making decisions about spending money on long-term assets. In order to make these decisions, management must first know what each profit center’s equipment needs are.

Supply and demand for the company’s products, current sales prices and costs, and wage trends would be helpful in order to determine future cash flows as they relate to implementing various capital projects, but these are not the best answers.

How well did you know this?
1
Not at all
2
3
4
5
Perfectly
70
Q

Pinecrest Co. had variable costs of 25% of sales and fixed costs of $30,000. Pinecrest’s breakeven point in sales dollars was:

A.
$24,000.

B.
$30,000.

Correct C.
$40,000.

D.
$120,000.

A

Contribution margin ratio = Gross profit ÷ Sales = 75% (0.75)
Breakeven point in dollars = Fixed costs ÷ CM ratio = $30,000 ÷ 0.75 = $40,000

How well did you know this?
1
Not at all
2
3
4
5
Perfectly
71
Q

When using a flexible budget, a decrease in production levels within a relevant range:

A.
decreases variable cost per unit.

Correct B.
decreases total costs.

C.
increases total fixed costs.

D.
increases variable cost per unit.

A

In a normal flexible budget situation (where both variable and fixed costs are present), a decrease in the level of production would be accompanied by a decrease in total costs.

Proof: Assume that variable cost per unit is $10 and total fixed cost is $50,000 per time period.

Production (units) 10,000 decreases to 8,000
——– ——–

Variable costs
(10,000 x $10) $100,000
( 8,000 x $10) $ 80,000
Fixed costs 50,000 50,000
——– ——–
Total cost $150,000 decreases to $130,000

How well did you know this?
1
Not at all
2
3
4
5
Perfectly
72
Q

For the current-period production levels, Woodwork Co. budgeted 11,000 board feet of production and purchased 15,000 board feet. The material cost was budgeted at $7 per foot. The actual cost for the period was $8.50 per foot. What was Woodwork’s material price variance for the period?

A.
$6,000 unfavorable

B.
$16,500 unfavorable

C.
$19,500 unfavorable

Correct D.
$22,500 unfavorable

A

MPV= AQ (SP-AP)

15,000(7-8.50)
22,500 Unfavorable

How well did you know this?
1
Not at all
2
3
4
5
Perfectly
73
Q

Brewster Co. has the following financial information:

Fixed costs $20,000
Variable costs 60%
Sales price $50
What amount of sales is required for Brewster to achieve a 15% return on sales?

A.
$33,333

B.
$50,000

Correct C.
$80,000

D.
$133,333

A

To achieve a 15% return on sales, $80,000 is required:

Net income = 0.15 x Sales

Sales - Variable costs - Fixed costs = Net income
Sales - (0.60 x Sales) - $20,000 = 0.15 x Sales

(0.85 x Sales) - (0.60 x Sales) = $20,000
0.25 x Sales = $20,000
Sales = $80,000

How well did you know this?
1
Not at all
2
3
4
5
Perfectly
74
Q

Augusta, Inc., expects manufacturing and sales of 70,000 units of product Maggie, its only product, to occur evenly over a 10-week period. Augusta pays for materials in the week following use. The balance of accounts payable for materials at the beginning of the 10-week period is $40,000. There are no beginning inventories. The fol­lowing information pertains to product Maggie for the 10-week period:

Sales price $11 per unit
Materials $3 per unit
Manufacturing conversion costs—Fixed $210,000
Variable $2 per unit
Selling and administrative costs—Fixed $45,000
Variable $1 per unit

What amount should Augusta budget for cash payments to material suppliers during the period?

A.
$189,000

B.
$229,000

Incorrect C.
$210,000

D.
$214,000

A

Manufacturing and sales occur evenly over the period, so each week has the same production and sales. At 70,000 units, that is 7,000 units per week. Cash payments for materials are $3 per unit, so payments for material for one week’s production is $21,000.

They will pay $40,000 at the beginning of the period for beginning accounts payable in addition to paying for 70,000 units at $3 per unit, or $210,000, for a total of $250,000. However, the materials used during the last week, 7,000 units at $3, or $21,000, will not be paid until the first week after the end of the period, reducing cash payments during the period down to $229,000.

How well did you know this?
1
Not at all
2
3
4
5
Perfectly
75
Q

What is the cost of ending inventory given the following factors?

Beginning inventory     $ 5,000
Total production costs   60,000
Cost of goods sold       55,000
Direct labor             40,000
A.	 	
$5,000

Correct B.
$10,000

C.
$45,000

D.
$50,000

A

Ending inventory = Production costs + Beginning inventory -
COGS (Cost of goods sold)
= $60,000 + $5,000 - $55,000
= $10,000
Note: Direct labor would have already been included in the total production costs.

How well did you know this?
1
Not at all
2
3
4
5
Perfectly
76
Q

To assist in an investment decision, Gift Co. selected the most likely sales volume from several possible outcomes. Which of the following attributes would selected sales volume reflect?

A.
The midpoint of the range

B.
The median

C.
The greatest probability

Incorrect D.
The expected value

A

“Most likely” is another way of saying the question is looking for the outcome with the greatest probability of occurrence.

Measures of central tendency include mean (average), mode (most common value), and median (the amount with half the values above and half below). Expected value is the weighted average of all the outcomes weighted by probability of occurrence.

The midpoint, median, and expected value may not have the greatest probability of occurrence, so none of those are necessarily the most likely to occur.

How well did you know this?
1
Not at all
2
3
4
5
Perfectly
77
Q

At annual sales of $900,000, the Ebo product has the following unit sales price and costs:

Sales price $20

Prime cost 6
Manufacturing overhead
Variable 1
Fixed 7
Selling and admin. costs
Variable 1
Fixed 3

18

Profit $ 2
===
What is Ebo’s breakeven point in units?

A

Prime cost consists of direct material and direct labor. Both of these are variable costs, so total variable cost is $8 per unit ($6 + $1 + $1).

Current unit sales = $900,000 / $20 per unit = 45,000 units
Total fixed costs = (45,000 x $7) + (45,000 x $3)
= $315,000 + $135,000
= $450,000
Breakeven units = Total fixed costs / (Selling price - Variable cost)
= $450,000 / ($20 - $8)
= $450,000 / $12
= 37,500 units

How well did you know this?
1
Not at all
2
3
4
5
Perfectly
78
Q

A company produces and sells two products. The first product accounts for 75% of units sold and the second product accounts for the remaining 25% of units sold. The first product has a selling price of $10 per unit, variable costs of $6 per unit, and allocated fixed costs of $100,000. The second product has a selling price of $25 per unit, variable costs of $13 per unit, and allocated fixed costs of $212,000. At the breakeven point, what number of units of the first product will have been sold?

A.
52,000

B.
39,000

Incorrect C.
25,000

D.
14,625

A

Total fixed costs of $312,000 ($100,000 from Product 1 and $212,000 from Product 2) must be covered by the contribution margin of the two products combined.

We know three times as many Product 1 units are sold as Product 2 units because 75% of unit sales is from Product 1. If we imagine a product package of 3 units of Product 1 and 1 unit of Product 2, it would be sold for (3 × $10) + $25, or $55. The variable cost of the package would be 3 × $6, or $18 for Product 1, plus $13 for Product 2, a total variable cost of $31. The contribution margin of the package would be $24 ($55 - $31).

Breakeven package units = Fixed costs ÷ Contribution margin per package = $312,000 ÷ $24 = 13,000 package units
Since there are 3 units of Product 1 in each package, sales of Product 1 will be 3 × 13,000 packages, or 39,000 units of Product 1.

How well did you know this?
1
Not at all
2
3
4
5
Perfectly
79
Q

Mien Co. is budgeting sales of 53,000 units of product Nous for October. The manufacture of one unit of Nous requires four kilos of chemical Loire. During October, Mien plans to reduce the inventory of Loire by 50,000 kilos and increase the finished goods inventory of Nous by 6,000 units. There is no Nous work-in-process inventory. How many kilos of Loire is Mien budgeting to purchase in October?

A.
138,000

B.
162,000

Correct C.
186,000

D.
238,000

A

Units produced = Unit sales + Increase in inventory
= 53,000 + 6,000
= 59,000 units of Nous

Purchases of Loire = Production requirement - Decrease in inventory
= (59,000 x 4) - 50,000
= 236,000 - 50,000
= 186,000 kilos of Loire

How well did you know this?
1
Not at all
2
3
4
5
Perfectly
80
Q

A ceramics manufacturer sold cups last year for $7.50 each. Variable costs of manufacturing were $2.25 per unit. The company needed to sell 20,000 cups to break even. Net income was $5,040. This year, the company expects the price per cup to be $9.00, variable manufacturing costs to increase 33.3%, and fixed costs to increase 10%. How many cups (rounded) does the company need to sell this year to break even?

A.
17,111

B.
17,500

C.
19,250

Incorrect D.
25,667

A

Total fixed costs were not given directly in the problem; however, they can be determined by first calculating the fixed costs for the prior year. Last year, the company needed to sell 20,000 cups in order to break even; therefore:

Last year’s fixed costs = 20,000 units x ($7.50 - $2.25)
= $105,000

The projected fixed costs are 10% more than the prior year.

Projected fixed costs = $105,000 x 1.10
= $115,500

Breakeven in units = FC ÷ (P - V) = $115,500 ÷ ($9 - $3) = 19,250 units
Terms

How well did you know this?
1
Not at all
2
3
4
5
Perfectly
81
Q

A company is considering outsourcing one of the component parts for its product. The company currently makes 10,000 parts per month. Current costs are as follows:

                       Per Unit    Total
                       --------   ------- Direct materials              $4      $40,000 Direct labor                   3       30,000 Fixed plant facility cost      2       20,000

The company decides to purchase the part for $8 per unit from another supplier and rents its idle capacity for $5,000/month. How will the company’s monthly costs change?

A.
Decrease $15,000

B.
Decrease $10,000

Correct C.
Increase $5,000

D.
Increase $10,000

A

The product should be produced if the incremental cost to produce the product, including any opportunity cost of idle facilities, is less than the purchase price.

Since the fixed plant charge will not change due to this decision, it is irrelevant and should not be considered. The direct materials and direct labor costs ($40,000 + $30,000 = $70,000) are relevant costs. These incremental costs total $70,000 per month to make the product, while they can buy the part for $80,000 per month, an increase in monthly costs of $10,000.

However, the rental income from renting the idle capacity of $5,000 reduces the monthly cost of purchasing the parts, for a net increase in monthly costs of $5,000.

How well did you know this?
1
Not at all
2
3
4
5
Perfectly
82
Q

A company’s controller is adjusting next year’s budget to reflect the impact of an expected 5% inflation rate. Listed below are selected items from next year’s budget before the adjustment:

Total salaries expense $250,000
Health costs 100,000
Depreciation expense 65,000
Interest expense on 10-year, fixed-rate notes 37,750
After adjusting for the 5% inflation rate, what is the company’s total budget for the selected items before taxes for next year?

Correct A.
$470,250

B.
$472,138

C.
$473,500

D.
$475,388

A

The expected inflation rate of 5% will affect items where the costs are being incurred due to activities taking place during the year, as opposed to being related to past actions or decisions. For example, the depreciation expense is based upon the systematic expensing of prior capital purchases. The interest expense is based upon a contract entered into sometime in the past.

The total budget before taxes for the items listed would be as follows:

Total salaries expense ($250,000 x 1.05 (105%)) $262,500
Health costs ($100,000 x 1.05) 105,000
Depreciation expense 65,000
Interest expense on 10-year, fixed-rate notes 37,750
——–
Adjusted budget $470,250
========
Terms

How well did you know this?
1
Not at all
2
3
4
5
Perfectly
83
Q

The operating results in summarized form for a retail computer store for 20X1 are:

Revenue:
Hardware sales $4,800,000
Software sales 2,000,000
Maintenance contracts 1,200,000
———-
Total revenue $8,000,000

Costs and expenses:
Cost of hardware sales $3,360,000
Cost of software sales 1,200,000
Marketing expenses 600,000
Customer maintenance costs 640,000
Administrative expenses 1,120,000
———-
Total costs and expenses $6,920,000
———-
Operating income $1,080,000
==========

The computer store is in the process of formulating its operating budget for 20X2 and has made the following assumptions:

The selling prices of hardware are expected to increase 10% but there will be no selling price increases for software or maintenance contracts.
Hardware unit sales are expected to increase 5% with a corresponding 5% growth in the number of maintenance contracts; growth in software unit sales is estimated at 8%.
The cost of hardware and software is expected to increase 4%.
Marketing expenses will be increased 5% in the coming year.
Three technicians will be added to the customer maintenance operations in the coming year, increasing the customer maintenance costs by $120,000.
Administrative costs will be held at the same level.
The retail computer store’s budgeted total revenue for 20X2 would be:

A.
$8,804,000.

Incorrect B.
$8,460,000.

C.
$8,904,000.

D.
$8,964,000.

A

Hardware sales = $4,800,000 x 1.05 x 1.10 = $5,544,000
Software sales = $2,000,000 x 1.08 = $2,160,000
Maintenance contracts = $1,200,000 x 1.05 = $1,260,000
———-
Total budgeted revenue for 20X2 $8,964,000
Hardware sales = $4,800,000 x 1.05 x 1.10 = $5,544,000 Correct
Software sales = $2,000,000 Incorrect(*)
Maintenance contracts = $1,200,000 x 1.05 = $1,260,000 Correct
———-
Total $8,804,000 Correct
$8,804,000 is incorrect because software sales do not include 8% growth.

Hardware sales = $4,800,000 x 1.05 = $5,040,000 Incorrect(*)
Software sales = $2,000,000 x 1.08 = $2,160,000 Correct
Maintenance contracts = $1,200,000 x 1.05 = $1,260,000 Correct
———-
Total $8,460,000
$8,460,000 is incorrect because hardware sales do not reflect a 10% price increase.

Hardware sales = $4,800,000 x 1.05 x 1.10 = $5,544,000 Correct
Software sales = $2,000,000 x 1.08 = $2,160,000 Correct
Maintenance contracts = $1,200,000 Incorrect(*)
———-
Total $8,904,000
$8,904,000 is incorrect because the computation does not include the 5% increase in maintenance contracts.

How well did you know this?
1
Not at all
2
3
4
5
Perfectly
84
Q

Carter Co. paid $1,000,000 for land three years ago. Carter estimates it can sell the land for $1,200,000, net of selling costs. If the land is not sold, Carter plans to develop the land at a cost of $1,500,000. Carter estimates net cash flow from the development in the first year of operations would be $500,000. What is Carter’s opportunity cost of the development?

A.
$1,500,000

Correct B.
$1,200,000

C.
$1,000,000

D.
$500,000

A

B. Correct! Opportunity cost is the (discounted) dollar value of benefits lost from an alternative (opportunity) as a result of choosing another alternative (opportunity). By choosing to develop the land, Carter would give up the opportunity to sell the land for $1,200,000, the opportunity cost.

How well did you know this?
1
Not at all
2
3
4
5
Perfectly
85
Q

Which of the following types of variances would a purchasing manager most likely influence?

A.
Direct materials price

Incorrect B.
Direct materials quantity

C.
Direct labor rate

D.
Direct labor efficiency

A

A purchasing manager contracts for purchases of raw materials, which affects the price per unit that is used in computing the direct materials price variance.

Direct materials quantity is incorrect because the quantity purchased is determined by the requisition from the using department, not negotiated by the purchasing manager. Direct labor rate and direct labor efficiency are incorrect because the purchasing manager purchases materials, not labor.

How well did you know this?
1
Not at all
2
3
4
5
Perfectly
86
Q

Clay Co. has considerable excess manufacturing capacity. A special job order’s cost sheet includes the following applied manufacturing overhead costs:

Fixed costs: $21,000
Variable costs: $33,000
The fixed costs include a normal $3,700 allocation for in-house design costs, although no in-house design will be done. Instead, the job will require the use of external designers costing $7,750. What is the total amount to be included in the calculation to determine the minimum acceptable price for the job?

A.
$36,700

B.
$40,750

C.
$54,000

Incorrect D.
$58,050

A

Relevant costs are expected future costs that are important or pertinent to the decision under consideration and will be affected by the decision. Historical or past (sunk) costs are irrelevant to the actual decision because the past costs will not be changed (recovered) by future action.

In this question, the fixed costs are a sunk cost that cannot be changed whether this special order is accepted or not. The relevant costs for this job include the variable costs of $33,000 plus the external design cost of $7,750 for a total of $40,750. Any price received for the special job above this will increase the company’s profit by the amount the price exceeds $40,750.

How well did you know this?
1
Not at all
2
3
4
5
Perfectly
87
Q

A company has gathered the following information from a recent production run:

Standard variable overhead rate    $10
Actual variable overhead rate        8
Standard process hours              20
Actual process hours                25
What is the company's variable overhead spending variance?

A.
$50 unfavorable

Correct B.
$50 favorable

C.
$40 unfavorable

D.
$40 favorable

A

The variable overhead spending variance is the difference between the actual amount paid ($8) and standard overhead ($10) for the 25 actual hours. The difference of $2 multiplied by 25 actual hours gives $50. The variance is favorable because the actual cost ($8) was less than the standard cost ($10).

How well did you know this?
1
Not at all
2
3
4
5
Perfectly
88
Q

The ABC Company is trying to decide between keeping an existing machine and replacing it with a new machine. The old machine was purchased just 2 years ago for $50,000 and had an expected life of 10 years. It now costs $1,000 a month for maintenance and repairs due to a mechanical problem. A new machine is being considered to replace it at a cost of $60,000. The new machine is more efficient and it will only cost $200 a month for maintenance and repairs. The new machine has an expected life of 10 years. In deciding to replace the old machine, which of the following factors, ignoring income taxes, should ABC not consider?

A.
Any estimated salvage value on the old machine

Correct B.
The original cost of the old machine

C.
The estimated useful life of the new machine

D.
The lower maintenance cost on the new machine

A

The original cost of the old machine is a sunk cost. It will not change whether or not the machine is replaced, and so it is irrelevant to the replacement decision.

The sales price (salvage value) of the old machine reduces the net cost of acquiring the new machine and should be considered. The longer the useful life of the new machine, the less the cost of using it each year. The lower maintenance cost should be considered since it reduces the cost of changing to the new machine.

How well did you know this?
1
Not at all
2
3
4
5
Perfectly
89
Q

In a computerized billing system, the computer generates a form that has two parts. The first part is a bill that customers are to retain and the second part is to be returned by customers with their payments. The return portion of the form is referred to as:

Correct A.
a turnaround document.

B.
a point-of-sale document.

C.
a transaction document.

D.
an accounts payable document

A

A turnaround form is an output form that is later used for input. The turnaround form increases efficiency and eliminates input errors.

Point-of-sale document may only be electronic in that the transaction is sent from a terminal without the use of a portion of a form.
A transaction document could be an internal, as well as external, document (e.g., deposit ticket, GL ticket).
An accounts payable document is a transaction document used in the accounts payable system (e.g., invoices, GL tickets).

How well did you know this?
1
Not at all
2
3
4
5
Perfectly
90
Q

Controllable revenue would be included in a performance report for:

Correct A.
a profit center.

B.
a cost center.

C.
both a profit center and a cost center.

D.
neither a profit center nor a cost center.

A

The performance report of a profit center would appear as follows:

           Controllable revenue     XXX
           Less Controllable costs   XX
                                    ---
           Profit center income      XX
                                    ===
Note that controllable revenue does appear in this performance report. On the other hand, only controllable costs appear in the cost center performance report.
How well did you know this?
1
Not at all
2
3
4
5
Perfectly
91
Q

In business information systems, the term “stakeholder” refers to which of the following parties?

A.
The management team responsible for the security of the documents and data stored on the computers or networks

B.
Information technology personnel responsible for creating the documents and data stored on the computers or networks

C.
Authorized users who are granted access rights to the documents and data stored on the computers or networks

Correct D.
Anyone in the organization who has a role in creating or using the documents and data stored on the computers or networks

A

A “stakeholder” is a broad term, encompassing all those with an interest in preparing or using the information. The other answer choices describe specific stakeholders, but there are other stakeholders as well.

How well did you know this?
1
Not at all
2
3
4
5
Perfectly
92
Q

A recent application of electronic data interchange (EDI) and the Internet is business-to-business (B2B) online commerce. Effective use of B2B should enable companies to:

A.
increase market efficiency.

B.
reduce purchasing costs.

Correct C.
both increase market efficiency and reduce purchasing costs.

D.
neither increase market efficiency nor reduce purchasing costs.

A

Purchasing raw materials and supplies online should reduce the cost of purchasing those goods. The ease of obtaining price quotations increases market intelligence.

How well did you know this?
1
Not at all
2
3
4
5
Perfectly
93
Q

A financial lease:

A.
may normally be canceled by the lessee on 30 days’ notice.

Correct B.
has a duration that corresponds to the useful life of the asset and payments that amortize the cost of the asset while providing the lessor an interest return.

C.
is only available through a bank.

D.
is only available on assets whose economic life exceeds 20 years.

A

Financial leases are financing-type leases. They typically are noncancelable and extend over the life of the leased asset with title transferring to lessee at the end of the lease term. Lease payments “pay for” the asset while providing the lessor with interest income.

How well did you know this?
1
Not at all
2
3
4
5
Perfectly
94
Q

A CPA would recommend implementing an activity-based costing system under which of the following circumstances?

A.
The client is a single-product manufacturer.

B.
Most of the client’s costs currently are classified as direct costs.

Correct C.
The client produced products that heterogeneously consume resources.

D.
The client produced many different products that homogeneously consume resources.

A

Activity-based costing (ABC) systems use a two-step process. First, a separate pool accumulates the overhead costs associated with each activity and some distinct measure is found for that activity. Overhead costs from each activity pool are allocated to product lines on the basis of the activity measure. In a second step, the overhead costs accumulated by product line are then allocated to the individual units in the product line.

The idea is that when various products consume significantly different levels of resources, costs can be more accurately assigned by identifying the level of resource use for each different product. Therefore, activity-based costing would be appropriate if the client’s products heterogeneously consume resources (each takes different levels of resources).

A single product manufacturer would not be able to benefit from closely analyzing the cost structure of different products. ABC systems are used to assign indirect costs, not direct costs clearly associated with specific products. If the different products homogeneously consume resources, they all have the same level of resource consumption, so the client would not benefit from analyzing the differences in resource consumption.

How well did you know this?
1
Not at all
2
3
4
5
Perfectly
95
Q

An employee mistakenly enters “April 31” in the date field. Which of the following programmed edit checks offers the best solution for detecting this error?

A.
Online prompting

B.
Mathematical accuracy

C.
Preformatted screen

Correct D.
Reasonableness

A

Programmed edit checks deal with the mathematical accuracy and reasonableness of data entries. There is no mathematical calculation in determining that April 31 is an incorrect date. However, the determination would be part of a reasonableness check comparing the actual number of days in a specific month with the day of the month entered in the date field.

How well did you know this?
1
Not at all
2
3
4
5
Perfectly
96
Q

Whatney Co. is considering the acquisition of a new, more efficient press. The cost of the press is $360,000, and the press has an estimated 6-year life with zero salvage value. Whatney uses straight-line depreciation for both financial reporting and income tax reporting purposes and has a 40% corporate income tax rate. In evaluating equipment acquisitions of this type, Whatney uses a goal of a 4-year payback period. To meet Whatney’s desired payback period, the press must produce a minimum annual before-tax, operating cash savings of:

Incorrect A.
$90,000.

B.
$110,000.

C.
$114,000.

D.
$150,000.

A

Payback period is the number years required to repay the initial investment in a capital project. A $360,000 capital investment would need an after-tax cash flow of $90,000 to meet the goal of a 4-year payback. The key to solving this problem involves remembering to consider the cash savings resulting from decreased taxes due to the depreciation expense. Note that the problem asks for the operating cash savings.

In this problem, operating cash savings would be total cash savings less nonoperating cash savings (e.g., cash savings from lower taxes). Annual depreciation on a $360,000 asset with an estimated 6-year life with zero salvage value using straight-line depreciation is $60,000. The tax savings generated by this depreciation is $60,000 times 40%, or $24,000. Now calculate after-tax operating cash savings: $90,000 - $24,000 = $66,000. However, the problem asks for before-tax operating cash savings, calculated as follows:

$66,000 ÷ (1 - Tax rate of 0.40) = $66,000 ÷ 0.60 = $110,000

How well did you know this?
1
Not at all
2
3
4
5
Perfectly
97
Q

A bank implemented an expert system to help account representatives consolidate the bank’s relationships with each customer. The expert system has:

A.
a sequential control structure.

B.
distinct input/output variables.

Correct C.
a knowledge base.

D.
passive data elements.

A

Expert systems have knowledge bases that represent the facts and inferences it knows, which were “taught” to it by human experts.

Traditional programs (e.g., COBOL) have sequential control structures, distinct input/output variables, and passive data elements; expert systems do not.

How well did you know this?
1
Not at all
2
3
4
5
Perfectly
98
Q

Most large-scale computer systems maintain at least three program libraries: production library (for running programs); source code library (maintains original source coding); and test library (for programs which are being changed). Which of the following statements is correct regarding the implementation of sound controls over computer program libraries?

Incorrect A.
Only programmers should have access to the production library.

B.
Users should have access to the test library to determine whether all changes are properly made.

C.
Only the program librarian should be allowed to make changes to the production library.

D.
The computer operator should have access to both the production library and the source code library to assist in diagnosing computer crashes.

A

Allowing only the program librarian to make changes to the production library would appropriately restrict access to the program modules that are running.

Good control dictates that programmers cannot make undetected, unrecorded changes to data or programs. Thus, programmers should be restricted from accessing the production library.
Programmers should be responsible for making program changes and users should be responsible for testing the changes. It would be poor control to allow users to have access to the test library. It would decrease accountability, as well as the competence to make the changes.
If the operator had access to both program libraries, the operator would be in a position to make unauthorized and undetected changes to the computer programs.

How well did you know this?
1
Not at all
2
3
4
5
Perfectly
99
Q

Which of the following is necessary to be an audit committee financial expert according to the criteria specified in the Sarbanes-Oxley Act of 2002?

A.
A limited understanding of generally accepted auditing standards

B.
Education and experience as a certified financial planner

Correct C.
Experience with internal accounting controls

D.
Experience in the preparation of tax returns

A

A. Although an understanding of GAAS would be likely to be at least tangentially helpful in this setting, this is not specified by SOX as one of the key criteria. These directors are not going to be doing any audits themselves.

B. SOX is not looking for financial planners.

C. (Correct!) SOX, in Section 407, provides that, in defining the term “financial expert” (which the SEC has done in detail), the Commission shall consider whether a person has through education and experience acquired: (1) an understanding of GAAP and financial statements; (2) experience in (a) preparation of financial statements and (b) application of such principles in connection with the accounting for estimates, accruals, and reserves; (3) experience with internal accounting controls; and (4) an understanding of audit committee functions.

D. Tax return preparation experience is not what SOX is concerned with

How well did you know this?
1
Not at all
2
3
4
5
Perfectly
100
Q

In an automated payroll processing environment, a department manager substituted the time card for a terminated employee with a time card for a fictitious employee. The fictitious employee had the same pay rate and hours worked as the terminated employee. The best control technique to detect this action using employee identification numbers would be a:

A.
batch total.

Incorrect B.
record count.

C.
hash total.

D.
subsequent check.

A

Assuming that the substitution takes place after the time cards have been batched for processing, the best control technique listed would be the hash total. The hash total is a type of batch control total. It is the summation of a quantitative but noninformational data field; for example, check numbers, purchase order numbers, and employee identification numbers.

How well did you know this?
1
Not at all
2
3
4
5
Perfectly
101
Q

Gartshore, Inc., is a mail-order book company. The company recently changed its credit policy in an attempt to increase sales. Gartshore’s variable cost ratio for obtaining credit is 70% and its required rate of return is 12%. The company projects that annual sales will increase from the current level of $360,000 to $432,000, but the average collection period on receivables will go from 30 to 40 days. Ignoring any tax implications, what is the cost of carrying additional investment in accounts receivable, using a 360-day year?

A.
$168

B.
$1,512

C.
$2,000

Incorrect D.
$2,160

A

Cost of holding accounts receivable before credit policy change: $360,000 sales ÷ 360 days = $1,000 average daily sales:

30 days average collection period = $30,000 average A/R balance
12% required rate of return = $3,600 annual interest

Cost of holding accounts receivable after credit policy change: $432,000 sales ÷ 360 days = $1,200 average daily sales:

40 days average collection period = $48,000 average A/R balance
12% required rate of return = $5,760 annual interest
$5,760 - $3,600 = $2,160 additional annual interest on holding A/R balance.

However, by stating the variable cost ratio, the problem implies it expects a distinction made between actual investment in A/R and margin earned. The actual investment by Gartshore is its variable cost, which for a mail-order book company represents cost of goods purchased for sale. That is, there is no change in fixed costs. Consequently, the $2,160 needs to be reduced to represent only the interest on the variable cost portion: $2,160 × 70% variable cost = $1,512.

How well did you know this?
1
Not at all
2
3
4
5
Perfectly
102
Q

Which of the following input controls would prevent an incorrect state abbreviation from being accepted as legitimate data?

A.
Reasonableness test

B.
Field check

C.
Digit verification check

Correct D.
Validity check

A

A validity check is an edit test in which an identification number or transaction code is compared with a table of valid identification numbers or codes maintained in computer memory. As an example, the system would compare an incorrect state abbreviation of “PS” with all possible valid state abbreviations and determine that it is not an abbreviation for one of the 50 states.

A reasonableness check or test would compare the data entry to the database to make sure that the input was logically correct, not valid. In other words, does the entry make sense in light of the data required?
A field check makes sure that the entry is the correct type for the field (numeric or alphanumeric). Any letters would satisfy this requirement, whether or not they were a valid state abbreviation.
A check digit verification uses redundant digits to detect errors in data transcription. This check would not prevent an incorrect state abbreviation from being accepted as legitimate data.

How well did you know this?
1
Not at all
2
3
4
5
Perfectly
103
Q

In an activity-based costing system, cost reduction is accomplished by identifying and eliminating:

A.
all cost drivers.

Correct B.
nonvalue-adding activities.

C.
all cost drivers and nonvalue-adding activities.

D.
neither all cost drivers nor nonvalue-adding activities.

A

The focal point in activity-based costing systems is activities, not cost drivers or products. Under activity-based costing, cost reduction is achieved through identifying and eliminating nonvalue-adding activities; that is, activities that do not add to the value of the finished product, such as unneeded material handling.

How well did you know this?
1
Not at all
2
3
4
5
Perfectly
104
Q

A software tool used to infrequently select or access items in the database would most likely be:

A.
a report generator.

B.
a program generator.

C.
an application generator.

Correct D.
a query utility program.

A

Report, program, and applications generators are fourth-generation languages that are used to create reports, programs, and applications on a routine basis.

Query utility programs enable a user to query or interrogate a database. Typically this is done on an as-needed basis.

How well did you know this?
1
Not at all
2
3
4
5
Perfectly
105
Q

Risk assessments, recovery plans for data systems, and implementation of safeguards are all components of:

A.
a control flowchart.

B.
a database.

Correct C.
a disaster recovery plan.

D.
an insurance claim form.

A

A disaster recovery plan should include a risk assessment, recommendation (and implementation) of safeguards, and recovery plans.

How well did you know this?
1
Not at all
2
3
4
5
Perfectly
106
Q

In a large multinational organization, which of the following job responsibilities should be assigned to the network administrator?

Correct A.
Managing remote access

B.
Developing application programs

C.
Reviewing security policy

D.
Installing operating system upgrades

A

Managing A. remote access

Database administrators (DBAs), also called network
administrators, are responsible for developing and maintaining the organization's
databases and for establishing controls to protect their integrity. Managing remote access
would be one method used by DBAs to protect the integrity of the organization's
databases
How well did you know this?
1
Not at all
2
3
4
5
Perfectly
107
Q

Which of the following inventory management approaches orders at the point where carrying costs equate nearest to restocking costs in order to minimize total inventory cost?

Correct A.
Economic order quantity

B.
Just-in-time

C.
Materials requirements planning

D.
ABC

A

Economic order quantity (EOQ) is the quantity of inventory that should be ordered at one time in order to minimize the associated costs of carrying and ordering inventory, such as purchase-order processing, transportation, and insurance. Carrying costs increase as the size of the order increases. Setup or ordering costs, however, decrease as the size of the production run or order increases.

Just-in-time (JIT) is a manufacturing philosophy where the company receives raw materials just in time to go into production, manufactures parts just in time to be assembled into products, and completes products just in time to be shipped to customers. JIT reduces the cost of carrying inventory but does not equate inventory carry costs with restocking costs.

Materials requirements planning (MRP) meets forecasted sales demand by balancing existing production capacity and raw materials needs. MRP does not attempt to equalize restocking and carrying costs.

The ABC inventory management system establishes priorities based on valuation of the inventory items (A = high value; B = medium value; C = small value) so that management can pay more attention to categories A and B. It has nothing to do with ordering points.

How well did you know this?
1
Not at all
2
3
4
5
Perfectly
108
Q

It is important to maintain proper segregation of duties in a computer environment. Which of the following access setups is appropriate?

Correct A.
Users have update access for production data

B.
Users have update access for production data and application programmers have update access for production programs

C.
Application programmers have update access for production data and users have update access for production programs

D.
Users have update access for production data and application programmers have update access for both production data and programs

A

Users need to update data through applications programs.

Application programmers should not be able to change production programs. They should submit changes to the change control unit.

Application programmers should never have update access to production data. Users have no need to change production programs.

How well did you know this?
1
Not at all
2
3
4
5
Perfectly
109
Q

Which of the following is the primary advantage of using a value-added network (VAN)?

A.
It provides confidentiality for data transmitted over the Internet.

Correct B.
It provides increased security for data transmissions.

C.
It is more cost effective for the company than transmitting data over the Internet.

D.
It enables the company to obtain trend information on data transmissions.

A

Value-added networks (VANs) are telecommunication networks providing communication facilities, enhancing basic telecommunication services by passing, storing, and converting messages using enhanced security techniques.

“It provides confidentiality for data transmitted over the Internet” is incorrect because many VANs use private networks instead of the public Internet. “It is more cost effective for the company than transmitting data over the Internet” is false because the use of a VAN comes at additional cost above that for using the Internet. “It enables the company to obtain trend information on data transmissions” is false because, although a VAN may supply trend information on data transmissions, that is not a primary advantage of using a VAN.

How well did you know this?
1
Not at all
2
3
4
5
Perfectly
110
Q

An online database management system for sales and receivables was recently expanded to include credit approval transactions. An evaluation of controls was not performed prior to implementation.

If certain data elements were not defined in the expansion, the following problem could result:

A.
Unlimited access to data and transactions

Correct B.
Incomplete transaction processing

C.
Unauthorized program execution

D.
Manipulation of the database contents by an application program

A

Failure to completely define the program specification blocks (PSB) prevents the application program from accessing or changing data, resulting in incomplete processing.

Data element definition allows application programs to access or change data; therefore, if they are not defined, no access takes place.
Without the program specification blocks, the application program cannot access data and cannot execute.
The desired manipulation of the database contents by an application program cannot take place if program specification blocks are not defined.

How well did you know this?
1
Not at all
2
3
4
5
Perfectly
111
Q

Company management completes event identification and analyzes the risks. The company wishes to assess its risk after management’s response to the risk. According to COSO, which of the following types of risk does this situation represent?

A.
Inherent risk

Correct B.
Residual risk

C.
Event risk

D.
Detection risk

A

Answer A is incorrect because inherent risk is the risk that exists before management takes any steps to control the likelihood or impact of a risk.

Answer B is correct because residual risk is the risk that remains after management reacts to the risk, such as by instituting appropriate internal controls.

Answer C is incorrect because event risk is the risk of unforeseen events associated with a particular entity, not after management responds to the risk.

Answer D is incorrect because detection risk is the risk that auditors fail to detect a material misstatement in financial statements.

How well did you know this?
1
Not at all
2
3
4
5
Perfectly
112
Q

A total interruption of processing throughout a distributed information technology system can be minimized through the use of:

A.
exception reporting.

Correct B.
fail-soft protection.

C.
backup and recovery.

D.
data file security.

A

The capability to continue processing at all sites except a nonfunctioning one is called fail-soft protection, an advantage of distributed systems.

Exception reporting can be used to control correctness and timeliness of updates but cannot minimize the impact of an interruption.
Backup procedures are intended to prevent the recovery process from introducing any erroneous changes into the system after computer failure.
Data file security is intended to prevent unauthorized changes to data files.

How well did you know this?
1
Not at all
2
3
4
5
Perfectly
113
Q

To ensure the completeness of update in an online system, separate totals are accumulated for all transactions processed throughout the day. The computer then agrees these totals to the total of items accepted for processing. This is an example of:

Correct A.
run-to-run controls.

B.
computer matching.

C.
computer sequence check.

D.
one-for-one checking.

A

Run-to-run controls for an online system are able to accumulate separate totals for all transactions processed during the day and then agree the totals to the total of items accepted for processing.

Computer matching compares transaction data to referenced fields or records.
Computer sequence checks identify changes or breaks in a numerical sequence.
One-for-one checking generally requires manual comparisons of input data elements to processing results.

How well did you know this?
1
Not at all
2
3
4
5
Perfectly
114
Q

A manufacturing company employs a process cost system. The company’s product passes through both Department 1 and Department 2 in order to be completed. Conversion costs are incurred uniformly throughout the process in Department 2. The direct material is added in Department 2 when conversion is 80% complete. This direct material is a preservative that does not change the volume. Spoiled units are discovered at the final inspection and are recognized then for costing purposes. The physical flow of units for the current month is presented below.

Beginning work-in-process 14,000
in Department 2 (90% complete
with respect to conversion costs)

Transferred in from Department 1 76,000

Completed and transferred to 80,000
finished goods

Spoiled units - all normal 1,500

Ending work-in-process in 8,500
Department 2 (60% complete
with respect to conversion costs)
If the manufacturing company uses the weighted-average method, the equivalent units for direct mat

A

Under weighted-average process costing, equivalent units is equal to units completed (both good and spoiled) plus work done on ending work-in-process. Thus:

Equivalent units for direct materials = 80,000 + 1,500 + 0 = 81,500

67,500 is the result of incorrectly subtracting beginning work-in-process units (i.e., 80,000 + 1,500 - 14,000 = 67,500). Under weighted average, work on beginning work-in-process is counted in the current period.
The 80,000 unit average is only partially correct. The good units completed plus spoiled units completed should be included in equivalent units.
90,000 is the result of adding units transferred-in to beginning work-in-process (76,000 + 14,000 = 90,000). Neither of these is used in computing weighted-average equivalent units.
Hint: There is a reason why the ending work-in-process (WIP) is not included in the answer to this problem. The information that precedes the question says that direct material is added in Department 2 when conversion is 80% complete. The question then asks for the equivalent units for direct materials in Department 2. Since the ending WIP is only 60% complete with respect to conversion costs, direct materials have not yet been added to the ending WIP. Therefore, 60% of the 8,500 is not part of the equivalent units for direct materials.

How well did you know this?
1
Not at all
2
3
4
5
Perfectly
115
Q

During a meeting with the CEO of the Marble Company, Connie CPA learned that the son of the CEO had worked at the company during one summer doing odd jobs. The year in question was included in the past financial records being used as a basis for the business valuation for which Connie had been engaged. The payroll records revealed that the son had received $40,000 (including taxes and other benefits) for this summer work. Connie should make a normalization adjust for:

A.
a nonoperating item for the full $40,000 received by the son.

Incorrect B.
a nonrecurring item for the full $40,000 received by the son since he only worked at the Mable Company for one summer.

C.
a comparability item for $20,000 since it is not likely that guideline companies would have paid $40,000 for the work performed.

D.
a discretionary item for $35,000 since the going market rate for the work performed by the son would have been $5,000.

A

Business valuators often have to make adjustments during the normalization process. There are four basic categories of normalization adjustments:

Nonoperating adjustments: the removal of nonoperating items included in the historical financial statements that are not part of normal operations. The performance of “odd jobs” around a business facility would be normal operating items.
Nonrecurring adjustments: the removal of unusual, unexpected, or items not likely to occur again from the financial statements. Although the son is not likely to perform these duties in the future, someone would need to do general maintenance on an ongoing basis.
Comparability adjustments: adjustments of the historical financial statements to match GAAP choices of potential guideline companies. The payment of wages is not a “GAAP” question, even though they may be excessive.
Discretionary adjustments: adjustments to the historical financial statement to include or to remove items not considered part of normal operations. Excessive wages paid to family members are considered to be discretionary items. The normalization adjustment would need to be made to bring the amount paid within going market rates.

How well did you know this?
1
Not at all
2
3
4
5
Perfectly
116
Q

Which of the following statements is correct regarding corporate debt and equity securities?

Both debt and equity security holders have an ownership interest in the corporation.
Both debt and equity securities have an obligation to pay income.
A.
I only

B.
II only

C.
Both I and II

Correct D.
Neither I nor II

A

Debt is a form of financing that increases the liabilities of a corporation, whereas equity securities are an ownership interest.

Debt holders receive interest that is a deductible expense to the payer and recognized as income by the debt holder. Due to the contractual nature of debt, the debt holder is obligated to pay the agreed-upon interest.

Equity holders may receive dividends that are a distribution of income earned by the subject company; however, a company is not obligated to pay dividends until the board of directors has declared them.

How well did you know this?
1
Not at all
2
3
4
5
Perfectly
117
Q

The CPA reviewed the minutes of a board of directors meeting of LQR Corp., an audit client. An order for widget handles was outsourced to SDT Corp. because LQR could not fill the order. By having SDT produce the order, LQR was able to realize $100,000 in sales profits that otherwise would have been lost. The outsourcing added a cost of $10,000, but LQR was ahead by $90,000 when the order was completed. Which of the following statements is correct regarding LQR’s action?

Correct A.
The use of resource markets outside of LQR involves opportunity cost.

B.
Accounting profit is total revenue minus explicit costs and implicit costs.

C.
Implicit costs are not opportunity costs because they are internal costs.

D.
Explicit costs are opportunity costs from purchasing widget handles from the resource market.

A

Each decision made by a company involves opportunity cost. Opportunity cost refers to the benefits given up by making one choice over another. Even though LQR Corp. seems to have made the right decision by outsourcing, intangible opportunity costs are always present when one plan of action is chosen over another.

Accounting profit equals revenue minus explicit costs (cash expenditures). Accounting profit does not take into account implicit costs (earnings that could have been received had the resources been used in an alternative use).

Opportunity costs are a form of implicit cost. Explicit costs are not opportunity costs.

How well did you know this?
1
Not at all
2
3
4
5
Perfectly
118
Q

The following information pertains to Base Manufacturing Co.:

Selected Cost Driver Costs
———————————- ———-
Estimated annual overhead $ 900,000
Estimated annual direct labor cost 1,800,000
Actual direct labor cost for March 160,000
Actual overhead for March 90,000
Base Manufacturing Co.’s applied overhead for March is:

A.
$320,000.

B.
$75,000.

C.
$80,000.

Incorrect D.
$90,000.

A

The overhead rate is calculated as follows:

Estimated annual overhead ÷ Estimated annual direct labor = Overhead Rate
$900,000 ÷ $1,800,000 = 0.50
Therefore, overhead is applied at 50% of actual direct labor cost:

$160,000 × 0.50 = $80,000 applied overhead

How well did you know this?
1
Not at all
2
3
4
5
Perfectly
119
Q

Universal Air, Inc., supplies instrumentation components to airplane manufacturers. Although there are only a few competitors in this market, the competition is fierce. In order to remain competitive, Universal Air’s executive team conducted a customer survey and developed thirty new indicators to measure middle-management performance. This system was not successful, so new cross-functional teams, consisting of executives and middle managers, were formed to develop new performance measures. To ensure that the cross-functional teams are effective, all necessary resources should be provided. The most important such resource would be:

A.
comfortable meeting rooms.

B.
daily progress reports.

Correct C.
strong top management commitment to the process.

D.
None of the choices listed are necessa

A

Strong top management commitment to the process that is clearly communicated is extremely important to the success of this participative management effort. Comfortable meeting rooms and frequent progress reports are also important but are not as critical as actual and indicated top management support.

How well did you know this?
1
Not at all
2
3
4
5
Perfectly
120
Q

What would be the primary reason for a company to agree to a debt covenant limiting the percentage of its long-term debt?

A.
To cause the price of the company’s stock to rise

B.
To lower the company’s bond rating

C.
To reduce the risk for existing bondholders

Correct D.
To reduce the interest rate on the bonds being sold

A

Agreeing to a debt covenant limiting the percentage of long-term debt would have an effect on a company’s capital structure (debt-to-equity ratio). A favorable debt-to-equity ratio (along with other indicators of financial health) means a higher bond rating. A high bond rating means lower interest rates for bonds being sold, which also lowers the cost of capital for future bond issuances.

A debt covenant would not have any effect on risk for current bondholders or on the price of the company’s stock.

How well did you know this?
1
Not at all
2
3
4
5
Perfectly
121
Q

A corporation manages inventory performance by monitoring its inventory turnover. Selected financial records for the corporation are as follows:

                              Year 1      Year 2      Year 3
                            ----------  ----------  ---------- Annual sales                    $1,262,500  $1,062,500  $1,459,000 Gross annual profit percentage      45%         30%         40% The beginning finished goods inventory for Year 2 was 20% of Year 2 sales. The ending finished goods inventory for Year 2 was 18% of Year 3 sales. What was the corporation's inventory turnover for Year 2?

A.
1.34

B.
2.83

C.
3.03

Correct D.
3.13

A

Inventory turnover = Cost of goods sold ÷ Average inventory

Cost of goods sold:

Gross profit for Year 2 = $1,062,500 × 0.30 = $318,750
Year 2 gross profit is revenue less cost of goods sold (CGS), so:
$1,062,500 - CGS = $318,750
CGS = $1,062,500 - $318,750 = $743,750
Average inventory:

Beginning finished goods inventory was 0.20 × $1,062,500, or $212,500.
Ending finished goods inventory was 0.18 × $1,459,000, or $262,620.
Average inventory = (Beginning inventory + Ending inventory) ÷ 2 = ($212,500 + $262,620) ÷ 2 = $237,560
Inventory turnover = Cost of goods sold ÷ Average inventory = $743,750 ÷ $237,560 = 3.13

How well did you know this?
1
Not at all
2
3
4
5
Perfectly
122
Q

The concept of a management information system (MIS) continues to evolve over time. Which of the following is generally understood to be a central element of an MIS?

A.
Maintenance of a large collection of raw, unorganized data to support a variety of information needs

B.
Processing of data items is based on decision models.

C.
The user-machine nature of an MIS means that users have to be skilled in the use of computers to realize any benefits.

Incorrect D.
A single, highly integrated computer system that combines processing for all organizational functions.

A

The use of decision models to organize data is a central element of MIS.

The management of data in an organized database is a central element of MIS.
Users of an MIS do not have to be computer experts to realize benefits.
The MIS concept is not based on computers, and consists of an organized federation of subsystems rather than a single, highly integrated system.

How well did you know this?
1
Not at all
2
3
4
5
Perfectly
123
Q

Which of the following allows customers to pay for goods or services from a website while maintaining financial privacy?

A.
Credit card

B.
Site draft

Correct C.
E-cash

D.
Electronic check

A

E-cash currencies, such as bitcoins, are anonymous and allow payment for purchases from websites.

A credit card, a sight draft (one that promises immediate payment to the holder of the draft), and an electronic check (such as created when a debit card is used for a purchase) are not anonymous.

How well did you know this?
1
Not at all
2
3
4
5
Perfectly
124
Q

Which of the following procedures would an entity most likely include in its disaster recovery plan?

A.
Convert all data from EDI format to an internal company format.

B.
Maintain a Trojan horse program to prevent illicit activity.

C.
Develop an auxiliary power supply to provide uninterrupted electricity.

Correct D.
Store duplicate copies of files in a location away from the computer center.

A

In the event of certain disasters, the company will need to restore computer files. For certain disasters, the backup will only be possible if the copies are stored at a separate location.

The computer operations procedures should call for a backup of the files. This is a process of making duplicate copies of files and storing them in a different location to protect against loss through fire, flood, and other disasters. If the backup copies are needed, they are copied back to the primary storage devices.

How well did you know this?
1
Not at all
2
3
4
5
Perfectly
125
Q

A senior executive of an international organization who wishes to demonstrate the importance of the security of company information to all team members should:

Correct A.
visibly participate in a global information security campaign.

B.
allocate additional budget resources for external audit services.

C.
review and accept the information security risk assessments in a staff meeting.

D.
refer to the organization’s U.S. human resources policies on privacy in a company newsletter.

A

“All team members” refers to the entire international organization, which implies the executive would provide this message to all employees worldwide. The tone at the top is most clearly demonstrated by personal example set by senior executives. The other answer choices are good behaviors but they are not visible to the worldwide entity.

How well did you know this?
1
Not at all
2
3
4
5
Perfectly
126
Q

The diagram below represents the production and sales relationships of joint products P and Q. Joint costs are incurred until split-off, then separable costs are incurred in refining each product. Market values of P and Q at split-off are used to allocate joint costs.

If the market value of P at split-off increases and all other costs and selling prices remain unchanged, then the gross margin of:

A.
P increases and Q decreases.

B.
P increases and Q increases.

C.
P decreases and Q decreases.

Correct D.
P decreases and Q increases.

A

If the market value of P at split-off increases while everything remains the same, a larger portion of the joint costs would be allocated to P and a smaller portion of joint costs would be allocated to Q. Therefore, P would have greater cost of goods sold, resulting in lower gross margin. Similarly, Q would have smaller cost of goods sold, resulting in a higher gross margin.

Assume a joint cost of $100, a sales price at split-off of $100 for P and $100 for Q, and separable costs of $25 for each product. Also assume an eventual sales price of $200 for each product. Allocating joint cost of $100 on the ratio of sales value at split-off ($100 for each product out of $200 sales value of both) would result in a joint cost of 100/200 × $100, or $50 allocated to each product.

What would be the gross profit of each? Each would have a sales price of $200, from which we would subtract $50 of allocated joint cost and $25 separable cost for gross profit of $200 − $75, or $125.

Now assume that the market value of P at split-off increases to $150. We would allocate 150/250 × $100 of joint cost ($60) to P. Product P would still be sold for $200, from which we would subtract $60 of allocated joint cost and $25 separable cost for gross profit of $200 − $60 − $25, or $115 of gross profit.

The gross profit of product P decreased from $125 to $115.

Like product P, product Q would have a gross profit of $125 under the original assumptions. When we increase the value of product P at split-off to $150, it changes the joint cost allocation for product Q to 100/250 × $100, or $40. Product Q would still be sold for $200, from which we would subtract $40 of allocated joint cost and $25 separable cost for gross profit of $200 − $40 − $25, or $135 of gross profit.

The gross profit of product Q increased from $125 to $135.

How well did you know this?
1
Not at all
2
3
4
5
Perfectly
127
Q

The ability to manage cash effectively requires a knowledge of disbursement float. The time necessary to clear funds received can be decreased through use of all of the following, except:

A.
wire transfers.

Correct B.
an overdraft system.

C.
a concentration bank.

D.
a lockbox.

A

An overdraft system allows a company to “overdraw” on a checking account since the bank will automatically transfer funds from an interesting-bearing account of the company’s to cover the checks presented for payment.

Wire transfers, concentration banks, and lockboxes are used to speed up the collection process.

How well did you know this?
1
Not at all
2
3
4
5
Perfectly
128
Q

An accountant has been retained by a company as an investment advisor for its employees. Research of historical rates of return yields the following information:

Type of Investment Mean Return Standard Deviation
———————————- ———– ——————
Common stocks 12% 20%
Long-term corporate bonds 6% 8%
Intermediate-term government bonds 5% 5%
U.S. Treasury bills 4% 3%
Which of the following investments has the greatest reward/risk ratio if a return’s standard deviation is an accurate assessment of investment risk?

A.
Common stocks

B.
Long-term corporate bonds

C.
Intermediate-term government bonds

Correct D.
U.S. Treasury bills

A

The reward/risk ratio is the rate of return divided by a measure of risk (the standard deviation in this question). Computing this reward-to-risk ratio for U.S. Treasury bills gives 4%/3%, or 1.33, which exceeds the ratio for the other investment alternatives:

Common stocks: 12%/20%, or 0.60
Long-term corporate bonds: 6%/8%, or 0.75
Intermediate-term government bonds: 5%/5%, or 1.00

How well did you know this?
1
Not at all
2
3
4
5
Perfectly
129
Q

Which of the following phrases defines the internal rate of return on a project?

A.
The number of years it takes to recover the investment

Correct B.
The discount rate at which the net present value of the project equals zero

C.
The discount rate at which the net present value of the project equals one

D.
The weighted-average cost of capital used to finance the project

A

The internal rate of return is the interest rate that will make the present value of the future net cash flows equal to the initial cash outlay. In other words, it is the interest rate that gives a net present value of zero.

The answer choice “the number of years it takes to recover the investment” is incorrect because it is the definition of the payback period, a number of years, not a rate of return. “The discount rate at which the net present value of the project equals one” is incorrect because the internal rate of return is the rate of return where the net present value is zero, not one. “The weighted-average cost of capital used to finance the project” is incorrect because weighted-average cost of capital is the interest rate that the company is paying on its other sources of financing. It is used to determine that project rate of return that would be acceptable to the company, but it is not used to calculate the internal rate of return.

How well did you know this?
1
Not at all
2
3
4
5
Perfectly
130
Q

Which of the following is a risk that is higher when an electronic funds transfer (EFT) system is used?

A.
Improper change control procedures

B.
Unauthorized access and activity

C.
Insufficient online edit checks

Incorrect D.
Inadequate backups and disaster recovery procedures

A

Unauthorized access is a risk which is higher in an EFT environment.

Improper change control procedures, insufficient online edit checks, and inadequate backups and disaster recovery procedures are risks that are common to all IT environments.

How well did you know this?
1
Not at all
2
3
4
5
Perfectly
131
Q

Computer program libraries can best be kept secure by:

Correct A.
restricting physical and logical access.

B.
denying access from remote terminals.

C.
monitoring physical access to program library media.

D.
installing a logging system for program access.

A

Restricting physical and logical access secures program libraries from unauthorized use, in person and remotely via terminals.

Installing a logging system for program access would permit detection of unauthorized access but would not prevent it. Monitoring physical access to program library media would control only unauthorized physical access. Denying all remote access via terminals would likely be inefficient and would not secure program libraries against physical access.

How well did you know this?
1
Not at all
2
3
4
5
Perfectly
132
Q

Which of the following solutions creates an encrypted communication tunnel across the Internet for the purpose of allowing a remote user secure access into the network?

A.
Packet-switched network

Incorrect B.
Digital encryption

C.
Authority certificate

D.
Virtual private network

A

A virtual private network (VPN) uses the Internet to provide secure remote access to an organization’s network.

The other answer choices (packet-switched network, digital encryption, and authority certificate) are incorrect because none of them create an encrypted communications tunnel across the Internet. They all refer to verification of the data being transmitted rather than the communication process.

How well did you know this?
1
Not at all
2
3
4
5
Perfectly
133
Q

Which of the following statements is correct when a corporation is earning excess profits?

A.
Participating preferred stock acts more like equity than cumulative preferred stock.

B.
Cumulative preferred stock acts more like equity than participating preferred stock.

C.
Cumulative preferred stock does not act more or less like equity than participating preferred stock.

Incorrect D.
No statement can be made comparing cumulative preferred and participating preferred stocks to equity.

A

Participating preferred stock acts more like equity than cumulative preferred stock when a corporation is earning excess profits because the stock does not receive a fixed percentage like debt. When excess profits are earned, the participating preferred stock receives additional dividends.

How well did you know this?
1
Not at all
2
3
4
5
Perfectly
134
Q

The market approach is one of the three basic methodologies available to the valuator. When determining whether this approach can be employed in a particular engagement (whether guideline companies are available), the valuator must keep all of the following in mind except:

A.
the normalized statements of the subject of the valuation need to use similar GAAP choices, such as LIFO and FIFO, as the guideline companies.

B.
one company does not make a comparable.

Correct C.
the guideline companies will need to be identical to the subject of the valuation.

D.
a guideline company needs to produce (supply) similar products, serve similar markets, and be within a similar size range as the subject company.

A

Finding guideline companies when performing a business valuation is often difficult. When developing a list of potential guideline companies, the business valuator assesses items such as whether the potential guideline company:

uses similar GAAP choices as the subject company.
has a similar product diversification as the subject company.
serves similar markets as the subject company.
has a similar geographic diversification as the subject company.
is of a similar size as the subject company.
has similar financial and operating leverage as the subject company.
has similar liquidity, solvency, growth, and profitability as the subject company.

How well did you know this?
1
Not at all
2
3
4
5
Perfectly
135
Q

Breakeven analysis assumes over the relevant range that

Total costs are linear.

Fixed costs are nonlinear.

Variable costs are nonlinear.

Selling prices are nonlinear.

A

Total costs are linear.

This answer is correct. There are a number of underlying assumptions to breakeven analysis that need to be considered in the calculation and interpretation of breakeven computations. One of these assumptions is that the behavior of total cost and total revenue is linear, even though in actuality it may not be. It is important to note that under breakeven analysis, the linearity assumption is only applicable for a particular relevant range of activity and is not assumed for all levels of activity. Since the actual behavior of total cost and total revenue within a relevant range is usually close to being linear, this assumption will only slightly affect the precision and reliability in a given breakeven calculation.

How well did you know this?
1
Not at all
2
3
4
5
Perfectly
136
Q

In a competitive market for labor in which demand is stable, if workers try to increase their wage,

A

This answer is correct. If wages rise in a stable market, demand for labor will decline and employment will fall.

How well did you know this?
1
Not at all
2
3
4
5
Perfectly
137
Q

A static budget contains which of the following amounts?
Actual costs for actual output.

Actual costs for budgeted output.

Budgeted costs for actual output.

Budgeted costs for budgeted output.

A

This answer is correct. A static budget includes budgeted costs for budgeted output.

How well did you know this?
1
Not at all
2
3
4
5
Perfectly
138
Q
Which of the statements best describes the concept of six-sigma quality?
10 defects per million.
3.4 defects per million.
6.0 defects per million.
100 defects per million.
A

What is six-sigma? A statistical measure expressing how close a product comes to its quality goal. One-sigma means 68% of products are acceptable; three-sigma means 99.7%. Six-sigma is 99.999997% perfect: 3.4 defects per million parts.

Six-sigma black belts must attend a minimum of four months of training in statistical and other quality improvement methods. Six-sigma black belts are experts in the six-sigma methodology. They learn and demonstrate proficiency in the DMAIC methodology and statistical process control (SPC) techniques within that methodology. DMAIC is the structured methodology for process improvement within the six-sigma framework. It stands for define, measure, analyze, improve, and control.

How well did you know this?
1
Not at all
2
3
4
5
Perfectly
139
Q

Which of the following characteristics represent an advantage of the internal rate of return technique over the accounting rate of return technique in evaluating a project?

I. Recognition of the project’s salvage value.
II. Emphasis on cash flows.
III. Recognition of the time value of money.
I only.
I and II.
II and III.
I, II, and III.

A

Close
The following chart summarizes the strengths and weaknesses of the capital budgeting methods.

Methods Strengths Weaknesses**
Payback 1. Easy to understand and use 1. Ignores time value of money

2. Emphasizes liquidity 2. Ignores cash flows after payback period
3. Does not measure profitability

Net Present Value (NVP) 1. Emphasizes cash flows 1. Favors larger, longer projects

2. Recognizes time value of money 2. Assumes no change in required rate of return
3. Assumes discount rate is reinvestment rate*
4. Easy to apply

Internal Rate of Return (IRR) 1. Emphasizes cash flows 1. Assumes IRR is the reinvestment rate*

2. Recognizes time value of money
2. Favors shorter projects
3. Computes true return of projects

Accounting rate of Return (ARR) 1. Easy to understand and use

1. Does not emphasize cash flows
2. Ties in with income statement and performance evaluation
2. Ignores time value of money
3. Misstates the true return of projects

  • Note that assuming the discount rate is the reinvestment rate results in using the same reinvestment rate for projects of similar risk, while assuming the IRR is the reinvestment rate assumes higher reinvestment rates for projects with higher true returns, regardless of the risk involved.
How well did you know this?
1
Not at all
2
3
4
5
Perfectly
140
Q

Greater barriers to collusion exist for oligopolistic firms when
The firms’ costs are more similar.
There are fewer firms in the industry.
General economic conditions are recessionary.
The firms’ products are standardized.

A

This answer is correct. In a recession, firms in the industry have lower sales volume, higher average costs per unit, and excess capacity. The resulting profit squeeze makes price cutting more of a temptation, in the hope of gaining sales at the expense of rivals.

How well did you know this?
1
Not at all
2
3
4
5
Perfectly
141
Q

An overall description of a database, including the names of data elements, their characteristics, and their relationship to one another, would be defined by using a

Data definition language.

Data control language.

Data manipulation language.

Data command interpreter language.

A

This answer is correct. The data definition language defines the database structure and content, especially the schema and subschema descriptions, including the names of the data elements contained in the database and their relationship to each other.

How well did you know this?
1
Not at all
2
3
4
5
Perfectly
142
Q

Which of the following topics is the focus of managerial accounting?

Financial statements and other financial reports.

Historical cost principles.

The needs of creditors.

The needs of the organization’s internal parties.

A

This answer is correct. Managerial accounting focuses on providing information for decision making by employees and management (internal parties).

How well did you know this?
1
Not at all
2
3
4
5
Perfectly
143
Q

This question is based on the following information.

Total units of product	Average fixed cost	Average variable cost	Average total cost
6	$15.00	 $25.00	 $40.00
7	  12.86	   24.00	   36.86
8	  11.25	   23.50	   34.75
9	  10.00	   23.75	   33.75
The total cost of producing seven units is
$90.02
$168.00
$258.02
$280.00
A

This answer is correct. The total cost would equal the average cost multiplied by the number of units, or $258.02 (7 × $36.86).

How well did you know this?
1
Not at all
2
3
4
5
Perfectly
144
Q

Which of the following procedures should be included in the disaster recovery plan for an Information Technology department?
Replacement of personal computers for user departments.
Identification of critical applications.
Physical security of warehouse facilities.
Cross-training of operating personnel.

A

This answer is correct because a disaster recovery plan must identify the critical applications.

How well did you know this?
1
Not at all
2
3
4
5
Perfectly
145
Q
The budgeting tool or process where estimates of revenues are prepared for each product beginning with the product’s research and development phase and traced through to its customer support phase is a(n)
Master budget.
Activity-based budget.
Zero-based budget.
Life-cycle budget.
A

A life-cycle budget is one that develops a budget for a product from its research and development phase to the last sales of the product.

How well did you know this?
1
Not at all
2
3
4
5
Perfectly
146
Q
Which of the following is themost expensive form of additional capital?
New debt.
New preferred stock.
Retained earnings.
New common stock.
A

This answer is correct. Common stock is the most expensive form of financing and because of floatation costs new common stock is more expensive than retained earnings.

How well did you know this?
1
Not at all
2
3
4
5
Perfectly
147
Q

Net present value (NPV) and internal rate of return (IRR) differ in that
NPV assumes reinvestment of project cash flows at the cost of capital while IRR assumes reinvestment of project cash flows at the internal rate of return.
NPV and IRR make different accept or reject decisions for independent projects.
IRR can be used to rank mutually exclusive investment projects but NPV cannot.
NPV is expressed as a percentage while IRR is expressed as a dollar amount

A

This answer is correct. NPV assumes that cash inflows from the investment project can be reinvested at the cost of capital while IRR assumes that cash flows from each project can be reinvested at the IRR for that particular project. This underlying assumption is considered to be a weakness of the IRR technique.

How well did you know this?
1
Not at all
2
3
4
5
Perfectly
148
Q

In an interest rate swap the first company?

Sells its right to low interest rate financing at a financial institution to the second company that is seeking to borrow funds.

Agrees to service the debt of the second company by making interest payments directly to the bank of the second company, while the second company agrees in exchange to make interest payments to the bank of the first company.

Buys the outstanding public debt of the second company and swaps the interest payments it receives on that debt for the interest payments it must make on its own debt.

Agrees to exchange with the second company the difference between the interest charges on its own borrowings and the interest charges on the borrowings of the second company.

A

Agrees to exchange with the second company the difference between the interest charges on its own borrowings and the interest charges on the borrowings of the second company.

This answer is correct. In an interest rate swap, two companies exchange their debt servicing obligations on some amount of debt principal. The actual exchange of funds during the agreement is in the form of a net payment from the party owing the greater amount for the period.

How well did you know this?
1
Not at all
2
3
4
5
Perfectly
149
Q

The Internet is made up of a series of networks which include
Gateways to allow mainframe computers to connect to personal computers.
Bridges to direct messages through the optimum data path.
Repeaters to physically connect separate local area networks (LANs).
Routers to strengthen data signals between distant computers.

A

This answer is correct. Gateways connect Internet computers of dissimilar networks.

How well did you know this?
1
Not at all
2
3
4
5
Perfectly
150
Q
An organization relied heavily on e-commerce for its transactions.  Evidence of the organization’s security awareness manual would be an example of which of the following types of controls?
Preventive.
Detective.
Corrective.
Compliance.
A

This answer is correct because the use of such a manual is designed to prevent breaches of security.

How well did you know this?
1
Not at all
2
3
4
5
Perfectly
151
Q

n a daily computer run to update checking account balances and print out basic details on any customer’s account that was overdrawn, the overdrawn account of the computer programmer was never printed. Which of the following control procedures would have been most effective in detecting this fraud?

Use of the test-data approach by the author in testing the client’s program and verification of the subsidiary file.
Use of a running control total for the master file of checking account balances and comparison with the printout.

A program check for valid customer code.

Periodic recompiling of programs from documented
source files, and comparison with programs currently in use.

A

This answer is correct because a periodic recompiling of the program from the original source files and comparison with the program currently in use would allow the auditor to detect the modification in the program that has permitted the fraud to occur.

How well did you know this?
1
Not at all
2
3
4
5
Perfectly
152
Q

Which of the following is not one of the four perspectives of the balanced scorecard?

Customer perspective.

Learning and growth perspective.

Internal business processes perspective.

Research and development perspective.

A

Four Perspectives of the Balanced Scorecard

a. Financial perspective. This perspective focuses on return on investment and other supporting financial performance measures. Example performance measures include profitability, return on invested capital, and revenue growth.
b. Customer perspective. This perspective focuses on performance in areas that are most critical to the customer. Example performance measures include customer satisfaction and customer retention.
c. Internal business processes perspective. This perspective focuses on operating effectively and efficiently and includes performance measures on cost, quality, and time for processes that are critical to the customers. Example performance measures include number of defects and cycle time.
d. Learning and growth perspective. This perspective focuses on performance measures relating to employees, infrastructure, teaming, and capabilities necessary for the internal processes to achieve customer and financial objectives. Example performance measures include employee satisfaction, hours of training per employee, and information technology expenditures per employee.

How well did you know this?
1
Not at all
2
3
4
5
Perfectly
153
Q

This question is based on the following information.

Total units of product	Average fixed cost	Average variable cost	Average total cost
6	$15.00	   $25.00	   $40.00
7	  12.86	    24.00	    36.86
8	  11.25	    23.50	    34.75
9	  10.00	    23.75	    33.75
The marginal cost of producing the ninth unit is
$23.50
$23.75
$25.75
$33.75
A

Marginal cost is the additional cost of producing one more unit. The amount may be obtained by subtracting the total cost of 9 units from the total cost of 8 units. $25.75 = ($33.75 × 9) – ($34.75 × 8).

How well did you know this?
1
Not at all
2
3
4
5
Perfectly
154
Q

The U.S. dollar has a free-floating exchange rate. When the dollar has fallen considerably in relation to other currencies, the

Trade account in the U.S. balance of payments is neither in a deficit nor in a surplus because of the floating exchange rates.

Capital account in the U.S. balance of payments is neither in a deficit nor in a surplus because of the floating exchange rates.

Fall in the dollar’s value cannot be expected to have any effect on the U.S. trade balance.

Cheaper dollar helps U.S. exporters of domestically produced goods.

A

Close
International purchasing power effect—When domestic price levels increase relative to foreign currencies, foreign products become less expensive causing an increase in imported goods and a decrease in exported goods. This decreases the aggregate demand of domestic products.

How well did you know this?
1
Not at all
2
3
4
5
Perfectly
155
Q
More than one file may be stored on a single magnetic disc.  Several programs may be in the core storage unit simultaneously. In both cases it is important to prevent the mixing of data.  One way to do this is to use
File integrity control.
Boundary protection.
Interleaving.
Paging.
A

This answer is correct because the primary purpose of boundary protection is to prevent the mixing of data on a magnetic memory disc and a core storage unit.

How well did you know this?
1
Not at all
2
3
4
5
Perfectly
156
Q

A company has a foreign-currency-dominated trade payable, due in 60 days. In order to eliminate the foreign exchange risk associated with the payable, the company could

Sell foreign currency forward today.

Wait 60 days and pay the invoice by purchasing foreign currency in the spot market at that time.

Buy foreign currency forward today.

Borrow foreign currency today, convert it to domestic currency on the spot market, and invest the funds in a domestic bank deposit until the invoice payment date.

A

This answer is correct. The company can arrange today for the exchange rate at which it will purchase the foreign currency in 60 days’ time by buying the currency in the forward market. This will eliminate the exchange risk associated with the trade payable.

How well did you know this?
1
Not at all
2
3
4
5
Perfectly
157
Q

Under a standard cost system, the material efficiency variances are the responsibility of

Production and industrial engineering.

Purchasing and industrial engineering.

Purchasing and sales.

Sales and industrial engineering.

A

This answer is correct. The production and industrial engineering departments have control over the usage of materials.

How well did you know this?
1
Not at all
2
3
4
5
Perfectly
158
Q

If controls add to the efficiency of operations, management must:

A.
implement the controls immediately.

B.
ask the internal auditor for recommendations.

Correct C.
weigh the benefit of reducing loss or inefficiency against the cost of the control.

D.
consider only the cost of the control.

A

Managers must weigh the benefit of reducing loss or inefficiency against the cost of the controls. They should not implement controls without first understanding whether any benefits of implementing these controls outweigh the costs. Although management can solicit recommendations from the internal auditor, it is not a requirement.

How well did you know this?
1
Not at all
2
3
4
5
Perfectly
159
Q

To identify those components of a telecommunication system (i.e., network) that present the greatest risk, the internal auditor should first:

A.
review the open systems interconnect (OSI) network model.

B.
identify the network operating costs.

Correct C.
determine the business purpose of the network.

D.
map the network software and hardware products into their respective layers.

A

Determining the business purpose of the network will be the best first step to identify those components of a telecommunication system which present the greatest risk.

Reviewing the open systems interconnect (OSI) network model may be done as part of audit preparation.
Identifying the network operating costs may be an audit step.
Mapping the network software and hardware products into their respective layers may be a subsequent audit step.

How well did you know this?
1
Not at all
2
3
4
5
Perfectly
160
Q

Internal auditors play a role in assessing an organization’s risk management by determining if:

A.
organizational objectives are distinct and separate from the organization’s mission.

B.
significant risks are completely and fully mitigated.

C.
risk responses have been selected that increase the organization’s risk appetite.

Correct D.
relevant risk information is captured and communicated in a timely manner.

A

Internal auditors are required by the International Standards for the Professional Practice of Internal Auditing (set forth by the IIA, Institute of Internal Auditors) to evaluate the effectiveness and contribute to the improvement of risk management processes.

Determining whether risk management processes are effective is a judgment resulting from the internal auditor’s assessment that:

organizational objectives support and align with the organization’s mission,
significant risks are identified and assessed,
appropriate risk responses are selected that align risks with the organization’s risk appetite, and
relevant risk information is captured and communicated in a timely manner across the organization, enabling staff, management, and the board to carry out their responsibilities.
IIA International Standards for the Professional Practice of Internal Auditing 2120

How well did you know this?
1
Not at all
2
3
4
5
Perfectly
161
Q

Communications risk has increased in recent years primarily because of changes in which of the following?

Correct A.
Technology used

B.
Asset accessibility

C.
Business regulations

D.
Duties of managers

A

Technological change, particularly in the area of telecommunications, has greatly increased communications risk for businesses utilizing this technology. Lack of security access to telecommunication systems tied into entity computer files can lead to loss of data and theft of assets.

How well did you know this?
1
Not at all
2
3
4
5
Perfectly
162
Q

Management’s attitude toward aggressive financial reporting and its emphasis on meeting projected profit goals most likely would significantly influence an entity’s control environment when:

A.
internal auditors have direct access to the board of directors and the entity’s management.

Correct B.
management is dominated by one individual who is also a shareholder.

C.
external policies established by parties outside the entity affect its accounting practices.

D.
the audit committee is active in overseeing the entity’s financial reporting policies.

A

The auditor must consider the client’s control environment when measuring control risk. One of the factors of the control environment, management philosophy and operating style, presents as an example the situation where management is dominated by only one or a few individuals. This domination directly affects the control environment because it is the only item listed above over which management has significant influence. The other answers (external policies, internal auditors, and the audit committee) all involve a degree of independence from management.

How well did you know this?
1
Not at all
2
3
4
5
Perfectly
163
Q

Communications risk is concerned with the unauthorized access to and manipulation of which of the following?

A.
Signatures

B.
Assets

C.
Liabilities

Correct D.
Data

A

Recent significant increases in the use of and reliance on telecommunications procedures and devices have resulted in greater risk of data theft and manipulation.

How well did you know this?
1
Not at all
2
3
4
5
Perfectly
164
Q

A company reports the following account balances at year-end:

        Account	Balance
Long-term debt	
               $200,000
Cash	
                   50,000
Net sales	
                 600,000
Fixed assets (net)	
                 320,000
Tax expense	
                   67,500
Inventory	
                   25,000
Common Stock	
                 100,000
Interest expense	
                   20,000
Administrative expense	
                   35,000
Retained earnings	
                 150,000
Accounts payable	
                   65,000
Accounts receivable	
                 120,000
Cost of goods sold	
                 400,000
Depreciation expense	
                   10,000

Additional Information:
The opening balance of common stock was $100,000
The opening balance of retained earnings was $82,500
The company had 10,000 common shares outstanding all year
No dividends were paid during the year
For the year just ended, the company has a gross margin of

A

This answer is correct. Gross margin is equal to gross profit divided by net sales. In this case gross profit is equal to $200,000 ($600,000 net sales – $400,000 cost of goods sold), and gross margin equals 33.3% ($200,000 ÷ 600,000).

How well did you know this?
1
Not at all
2
3
4
5
Perfectly
165
Q

Which of the following forms of compensation is more likely to result in shirking by management?

Fixed compensation.

Base salary and bonus
.
Base salary and stock options.

Base salary and stock grants.

A

This answer is correct. With fixed compensation management may not be inclined to work hard or take appropriate risks.

How well did you know this?
1
Not at all
2
3
4
5
Perfectly
166
Q

A management accountant performs a linear regression of maintenance cost versus production using a computer spreadsheet. The regression output shows an “intercept” value of $322,897. How should the accountant interpret this information?

Y has a value of $322,897 when X equals zero.
X has a value of $322,897 when Y equals zero.
The residual error of the regression is $322,897.
Maintenance cost has an average value of $322,897

A

This answer is correct. The general form of the linear regression equation is y = a (the intercept) + bx. Therefore, when x equals 0, y is equal to a (the intercept).

How well did you know this?
1
Not at all
2
3
4
5
Perfectly
167
Q

The trough of a business cycle is generally characterized by
Shortages of essential raw materials and rising costs.

Increasing purchasing power and increasing capital investments.

Rising costs and an unwillingness to risk new investments.

Unused productive capacity and an unwillingness to risk new investments.

A

. In the trough of a business cycle there is unused capacity and an unwillingness to make investments.

How well did you know this?
1
Not at all
2
3
4
5
Perfectly
168
Q

Which of the following risks can be minimized by requiring all employees accessing the information system to use passwords?

Collusion.

Data entry errors.

Failure of server duplicating function.

Firewall vulnerability.

A

This answer is correct because passwords can prevent unauthorized individuals from penetrating the firewall.

How well did you know this?
1
Not at all
2
3
4
5
Perfectly
169
Q

Which of the following would illustrate vertical financial statement analysis?

Interest expense as a percentage of net sales.

Gross profit compared to industry averages.

Net sales over time.

Earnings per share as compared to the prior year.

A

This answer is correct. Vertical analysis involves presenting everything within a financial statement as a percentage of a base. For a common-size income statement all items would be presented as a percentage of net sales.

How well did you know this?
1
Not at all
2
3
4
5
Perfectly
170
Q

Minon, Inc. purchased a long-term asset on the last day of the current year. What are the effects of this purchase on return on investment and residual income?

A

This answer is correct because purchasing an asset increases the average investment which reduces both return on investment and residual income.

171
Q

What accounts for the difference between the stated rate and the effective annual rate of a loan?

A

This answer is correct. The frequency of compounding explains the difference between the stated rate and the effective rate.

172
Q
An American importer expects to pay a British supplier 500,000 British pounds in three months.  Which of the following hedges is best for the importer to fix the price in dollars?
Buying British pound call options.
Buying British pound put options.
Selling British pound put options.
Selling British pound call options.
A

Buying British pound call options.

This answer is correct because a call option allows the importer to lock in the price of British pounds at the current exchange rate.

173
Q

Four Perspectives of the Balanced Scorecard

a. Financial perspective. This perspective focuses on return on investment and other supporting financial performance measures. Example performance measures include profitability, return on invested capital, and revenue growth.
b. Customer perspective. This perspective focuses on performance in areas that are most critical to the customer. Example performance measures include customer satisfaction and customer retention.
c. Internal business processes perspective. This perspective focuses on operating effectively and efficiently and includes performance measures on cost, quality, and time for processes that are critical to the customers. Example performance measures include number of defects and cycle time.
d. Learning and growth perspective. This perspective focuses on performance measures relating to employees, infrastructure, teaming, and capabilities necessary for the internal processes to achieve customer and financial objectives. Example performance measures include employee satisfaction, hours of training per employee, and information technology expenditures per employee.

A

Four Perspectives of the Balanced Scorecard

a. Financial perspective. This perspective focuses on return on investment and other supporting financial performance measures. Example performance measures include profitability, return on invested capital, and revenue growth.
b. Customer perspective. This perspective focuses on performance in areas that are most critical to the customer. Example performance measures include customer satisfaction and customer retention.
c. Internal business processes perspective. This perspective focuses on operating effectively and efficiently and includes performance measures on cost, quality, and time for processes that are critical to the customers. Example performance measures include number of defects and cycle time.
d. Learning and growth perspective. This perspective focuses on performance measures relating to employees, infrastructure, teaming, and capabilities necessary for the internal processes to achieve customer and financial objectives. Example performance measures include employee satisfaction, hours of training per employee, and information technology expenditures per employee.

174
Q

In an interest rate swap the first company?

A

Agrees to exchange with the second company the difference between the interest charges on its own borrowings and the interest charges on the borrowings of the second company.

In an interest rate swap, two companies exchange their debt servicing obligations on some amount of debt principal. The actual exchange of funds during the agreement is in the form of a net payment from the party owing the greater amount for the period.

175
Q

The purchase of treasury stock with a firm’s surplus cash:

A.
increases a firm’s assets.

B.
increases a firm’s financial leverage.

Incorrect C.
increases a firm’s equity.

D.
increases a firm’s interest coverage ratio.

A

Treasury stock is shares of the firm’s own stock held by the firm.

The firm’s financial leverage increases because the debt-to-equity ratio increases (as a result of the decrease in total stockholders’ equity).

176
Q

The Forming Department is the first of a two-stage production process. Spoilage is identified when the units have been completed in the Forming process. Costs of spoiled units are assigned to units completed and transferred to the second department in the period spoilage is identified. The following information concerns Forming’s conversion costs in May of this year:

                                            Conversion
                                  Units       Costs
                                  -----     ---------- Beginning work-in-process  (50% complete)                       2,000      $10,000 Units started during May              8,000       75,500 Spoilage-normal                         500 Units completed and transferred       7,000 Ending work-in-process  (80% complete)                       2,500 Using the weighted-average method, what was Forming's conversion cost transferred to the second production department?

A.
$59,850

B.
$64,125

Correct C.
$67,500

D.
$71,250

A

Beginning units 2,000
Units started 8,000
——
Units to account for 10,000
Units completed 7,000 x 100% = 7,000
Normal spoilage 500 x 100% = 500
Ending units 2,500 x 80% = 2,000
—— —–
Units accounted for 10,000 9,500 equivalent units

Conversion cost per
equivalent unit = Conversion Cost / Equivalent Units
= ($10,000 + $75,500) / 9,500
= $85,500 / 9,500 = $9 per unit

Conversion cost = Cost per Unit x (Good units + Normal spoilage)
transferred = $9 (7,000 + 500) = $67,500

177
Q

Select Co. had the following current-year financial statement relationships:

Asset turnover 5
Profit margin on sales 0.02
What was Select’s current-year percentage return on assets?

Incorrect A.
0.1%

B.
0.4%

C.
2.5%

D.
10.0%

A

Consider the following relationships:

Profit margin on Sales = Income ÷ Sales or Income = Profit margin on sales × Sales
Asset turnover = Sales ÷ Assets or Assets = Sales ÷ Asset turnover
Return on assets = Income ÷ Assets
In expanded fashion:

Return on assets = (Profit margin on sales × sales) ÷ (Sales ÷ Asset turnover) or (Profit margin on sales × Sales × Asset turnover) ÷ Sales
Since Sales cancels out:

Return on assets = Profit margin on sales x Asset turnover
= .02 x 5
= .10 or 10%

178
Q

If the average age of inventory is 90 days, the average age of accounts payable is 60 days, and the average age of accounts receivable is 65 days, the number of days in the cash flow cycle is:

A.
215 days.

B.
150 days.

Correct C.
95 days.

D.
85 days.

A

The cash flow cycle is the full cycle of cash inflows adjusted for outflows, calculated as 90 + 65 - 60 = 95 days. By this formula, one can see that reducing either the inventory holding period or the collection period for receivables will reduce the number of days in the cash flow cycle by accelerating sales turnover and cash collections, respectively. Increasing the payables period also reduces the cash flow cycle, since cash outflows can be delayed while new inventory and receivables periods can begin.

179
Q

Wong Company utilizes both strategic planning and operational budgeting. Which one of the following items would normally be considered in a strategic plan?

A.
Setting a target of 12% return on sales

Correct B.
Maintaining the image of the company as the industry leader

C.
Setting a market price per share of stock outstanding

D.
Distributing monthly reports for departmental variance analysis

A

Strategic planning is the general planning done by top management that charts a company’s course through its operating environment. Maintaining the image of a company as an industry leader is therefore a strategic goal.

The other answer choices, listed below, are all operational or budgetary goals:

Setting a target of 12% return on sales
Setting a market price per share of stock outstanding
Distributing monthly reports for departmental variance analysis

180
Q

Which of the following decision-making models equates the initial investment with the present value of the future cash inflows?

A.
Accounting rate of return

B.
Payback period

Correct C.
Internal rate of return

D.
Cost-benefit ratio

A

The internal rate of return (IRR) can be referred to as the yield (return) expected over the life of a project. It is computed by equating the initial investment with the present value of the cash flows over the life of the project. IRR is the discount rate that results in the net present value of all cash flows to be zero.

Neither the accounting rate of return nor the payback methods consider the time value of money, and the cost-benefit ratio is not a capital budgeting tool.

181
Q

Which of the following decision-making models equates the initial investment with the present value of the future cash inflows?

A.
Accounting rate of return

B.
Payback period

Correct C.
Internal rate of return

D.
Cost-benefit ratio

A

The internal rate of return (IRR) can be referred to as the yield (return) expected over the life of a project. It is computed by equating the initial investment with the present value of the cash flows over the life of the project. IRR is the discount rate that results in the net present value of all cash flows to be zero.

Neither the accounting rate of return nor the payback methods consider the time value of money, and the cost-benefit ratio is not a capital budgeting tool.

182
Q

Jones Corp. had an opportunity to use its capacity to produce an extra 5,000 units with a contribution margin of $5 per unit, or to rent out the space for $10,000. What was the opportunity cost of using the capacity?

A.
$35,000

B.
$25,000

C.
$15,000

Correct D.
$10,000

A

Opportunity costs are the earnings that could have been received had the storage been used in their best alternative use, such as foregone rental income. The company could have earned $10,000 renting out the space, and they have lost that opportunity by using the space for production. It may have been the correct decision, however, because they could earn more by producing in the space than by renting the space. Even if it is the correct decision, they have still lost the opportunity to earn the $10,000 from rental.

183
Q

A high debt to total assets ratio for a company indicates:

Correct A.
extensive use of financial leverage.

B.
that assets are over-valued.

C.
extensive use of financial leverage and that assets are over-valued.

D.
None of the answer choices are correct.

A

Financial leverage involves using borrowed funds to finance asset acquisition(s) which will generate returns exceeding the cost of borrowing. Thus, a higher debt to total assets ratio points toward the likelihood of more extensive financial leverage usage.

Most of the time, assets are undervalued (i.e., at acquisition cost) rather than overvalued. A high debt to total assets ratio would often be the result from such undervaluation.

184
Q

Enert Inc.’s current capital structure is shown as follows. This structure is optimal, and the company wishes to maintain it.

 Debt                       25%
 Preferred equity            5%
 Common equity              70% Enert's management is planning to build a $75 million facility that will be financed according to this desired capital structure. There is currently $15 million of cash that is available for capital expansion. The percentage of the $75 million that will come from a new issue of common stock is:

A.
52.50%.

B.
56.25%.

Incorrect C.
70.00%.

D.
56.00%.

A

To find the percentage that will come from a new stock issue, first assume that the $15 million available for capital expansion will be used. Then apply the 70% rate to the remaining $60 million. That results in $42 million, or 56% of the $75 million.

185
Q

Which of the following variables is not one of the variables traditionally found in national income calculations?

A.
Disposable income

B.
Gross domestic product

C.
Net domestic product

Correct D.
Real per capita gross domestic product

A

Real per capita gross domestic product is gross domestic product for a particular year adjusted for inflation compared to a base year using a price index. This adjusted number is then divided by population to derive real per capita GDP (gross domestic product). This measure is often used as an estimate of changes in well-being in a society over time, but it is not one of the variables one would traditionally see in national income calculations.

186
Q

A direct labor overtime premium should be charged to a specific job when the overtime is caused by the:

A.
increased overall level of activity.

Correct B.
customer’s requirement for early completion of job.

C.
management’s failure to include the job in the production schedule.

D.
management’s requirement that the job be completed before the annual factory vacation closure.

A

When actual amounts differ from standard amounts, a variance is recorded. The variance should be assigned to the department or division that has the ability to control the activity that causes the variance. The customer’s early completion requirement for this job was the activity that caused the overtime cost, so the additional cost should be charged to this job.

“Increased overall level of activity” is incorrect because an increase in general demand is not a cost that can be controlled. “Management’s failure to include the job in the production schedule” is incorrect because the job did not cause the overtime; management’s planning error did. “Management’s requirement that the job be completed before the annual factory vacation closure” is incorrect because management’s decision caused the overtime rather than the current specific job.

187
Q

R Beck, Inc., has learned that 4,971 of 310,000 of its commercial DVDs over the past 12 months have arrived to the customer as blank discs with no data contained on them. Which improvement initiative would R Beck most likely adapt to reduce or eliminate the blank discs?

A.
Lean production

B.
Business excellence framework

Correct C.
Six Sigma

D.
Business process re-engineering

A

While each answer choice represents a quality improvement initiative, Six Sigma is an improvement initiative specifically designed to address errors in defects. If R Beck were to achieve Six Sigma on 92,000 units, it would represent a defect rate of one DVD per year per 310,000 DVDs produced.

188
Q

Inflation and recession, energy market fluctuations and changing interest rates are all components of ________ risk.

A.
company-specific

B.
credit

C.
liquidity

Correct D.
market

A

Market risk or systemic risk is related to conditions that have an overall effect on companies. These specific risk conditions cannot be eliminated by use of diversification. Examples include conditions such as inflation and recession, congressional tax reform, energy market fluctuations, and changing interest rates.

189
Q

Within the COSO Internal Control—Integrated Framework, which of the following components is designed to ensure that internal controls continue to operate effectively?

A.
Control environment

B.
Risk assessment

C.
Information and communication

Correct D.
Monitoring activities

A

Monitoring of controls assesses the quality of internal control performance over time, including assessing the design and operation of controls on a timely basis and taking necessary corrective actions.

The control environment includes items such as a corporate code of conduct and ethical attitude of those charged with governance. Risk assessment refers to the identification, analysis, and management of risks relevant to the preparation of financial statements. The information and communication system refers to processing the data, including the source documents through the final reports.

190
Q

What does business process re-engineering do within an organization?

A.
Only analyzes design workflows

B.
Only designs work processes

Correct C.
Analyzes and designs workflows and processes

D.
Tests and implements processes

A

Business process re-engineering (BPR) identifies processes and workflows, analyzes processes and workflows, and designs processes and workflows

191
Q

The Committee of Sponsoring Organizations of the Treadway Commission (COSO) has developed a widely accepted and used framework for internal control that was designed to provide reasonable assurance for a company’s objectives related to all items except:

A.
effectiveness and efficiency of operations.

B.
reliability of financial reporting.

Correct C.
expansion of markets.

D.
compliance with laws and regulations.

A

COSO has developed a widely accepted and used framework for internal control that was designed to provide reasonable assurance for a firm’s objects related to:

effectiveness and efficiency of operations,
reliability of financial reporting, and
compliance with laws and regulations.
Although the expansion of markets may be in the strategic plan for a company and may come about due to the effectiveness and efficiency of operations, it is not a focus of the COSO Framework designed for internal control.

192
Q

If the net present value of a capital budgeting project is positive, it would indicate that the:

A.
present value of cash outflows exceeds the present value of cash inflows.

B.
payback period is less than one-half of the life of the project.

C.
internal rate of return is equal to the discount percentage rate used in the net present value computation.

Correct D.
rate of return for this project is greater than the discount percentage rate used in the net present value computation.

A

Net present value (NPV) is defined as the excess of present value of cash inflows from a project over the discounted net cash outflows.

A positive net present value indicates that the project’s rate of return is greater than the discount (bundle) rate of interest. Projects promising a positive NPV should be undertaken if funds are available.

193
Q

Which of the following forecasting methods relies mostly on judgment?

A.
Time series models

B.
Econometric models

Correct C.
Delphi

D.
Regression

A

The Delphi method is a collaborative process whereby managers or members of a group are independently surveyed to reach a consensus on something that will happen in the future. With no empirical evidence, this method relies mostly on judgment.

A time series model involves specific measurements taken over equally spaced intervals that assist with forecasting those measurements in the future.
Econometric models combine mathematical economics, statistics, economic statistics, and economic theory. This model involves regression analysis.
Regression uses an independent variable to predict the value of another variable.
Time series, econometric, and regression analyses all utilize mathematics and observation. Each is less subjective than the Delphi method of forecasting.

194
Q

Kanban is:

Correct A.
a technique for managing a just-in-time (JIT) inventory system developed by the Japanese.

B.
a method of evaluating alternative credit policies developed by the Japanese.

C.
a method of determining the economic order quantity expressed in mathematical terms.

D.
a method of developing a relationship between sales and inventory used in forecasting.

A

Kanban is a technique for managing a just-in-time inventory system. The kanban is a tag attached to the storage container where component parts are kept. As a component is used, a kanban is placed in a box. Managers determine the number of kanbans needed to be in the box of the component part before a reorder of that part is necessary.

195
Q

A banking system with a reserve ratio of 20% and a change in reserves of $1 million can increase its total demand deposits by:

A.
$200,000.

B.
$5 million.

C.
$1 million.

Incorrect D.
$800,000.

A

The banking system can increase its total demand deposits by $5 million, computed as follows:

If reserves increase by $1 million and the reserve ratio is 20%, and
Reserve ratio = Reserves / Total Demand Deposits, then
0.20 = 1,000,000 / increase in deposits.
Thus, the increase in deposits = $1,000,000 / 0.20, or $5,000,000.

196
Q

Mien Co. is budgeting sales of 53,000 units of product Nous for October. The manufacture of one unit of Nous requires four kilos of chemical Loire. During October, Mien plans to reduce the inventory of Loire by 50,000 kilos and increase the finished goods inventory of Nous by 6,000 units. There is no Nous work-in-process inventory. How many kilos of Loire is Mien budgeting to purchase in October?

A.
138,000

B.
162,000

Correct C.
186,000

D.
238,000

A

Units produced = Unit sales + Increase in inventory
= 53,000 + 6,000
= 59,000 units of Nous

Purchases of Loire = Production requirement - Decrease in inventory
= (59,000 x 4) - 50,000
= 236,000 - 50,000
= 186,000 kilos of Loire

197
Q

Which of the following pricing policies results in establishment of a price to external customers higher than the competitive price for a given industry?

Correct A.
Collusive pricing

B.
Dual pricing

C.
Predatory pricing

D.
Transfer pricing

A

Collusion is when competitors agree to restrict production so as to increase the price they receive for their product. A cartel is one form of collusion, where producers agree how much each member will produce and charge. OPEC (the Organization of the Petroleum Exporting Countries) is an example of a cartel.

Dual pricing is a practice of setting different prices for a product dependent on the currency used to buy it. It often is used to set lower-than-normal prices to gain access to a particular foreign market. Predatory pricing means lowering prices to such an extent as to drive competitors out of business. A transfer price is the price charged by one unit within a larger business to another unit in that business.

198
Q

Ribbonwood Corporation, a defense contractor, spent June completing Job CC818 for a government space contract. Information about Job CC818, other June costs, and relevant annual estimates follows:

Material issued:
(60% for direct use on Job CC818 and 40%
for indirect factory use through June) $ 4,000

Labor:
Direct labor for Job CC818
(200 direct labor hours at $15 per hour) 3,000
Indirect factory labor for June 850

Other June Costs:
Depreciation (50% factory, 50% administrative) 600
Other (60% factory and 40% administrative) 1,000
Cost driver is Direct Labor Hours
Estimated annual overhead 323,000
Estimated annual direct labor hours 4,000
Mark-up as a percent of costs 150%
What is Ribbonwood’s actual overhead for June?

A.
$1,750

Correct B.
$3,350

C.
$16,150

D.
$26,917

A

1,600 (Indirect materials: $4,000 × 40%) + $850 (Indirect labor: $850) + $300 (factory depreciation: $600 × 50%) + $600 (Other factory costs: $1,000 × 60%) = $3,350

199
Q

A company enters into an agreement with a firm which will factor the company’s accounts receivable. The factor agrees to buy the company’s receivables, which average $100,000 per month and have an average collection period of 30 days. The factor will advance up to 80% of the face value of receivables at an annual rate of 10% and charge a fee of 2% on all receivables purchased. The controller of the company estimates that the company would save $18,000 in collection expenses over the year. Fees and interest are not deducted in advance. Assuming a 360-day year, what is the annual cost of financing?

Incorrect A.
10.0%

B.
12.0%

C.
16.0%

D.
17.5%

A

A factor buys a company’s accounts receivables, assumes the risk of collection and the company gets its money immediately. In this problem, the standard factoring solution is complicated by the fact the fees and interest are not deducted in advance, changing the amount of interest.

Amount of receivables to be factored $100,000
Less: 20% factoring reserve (1.00 - .80) (20,000)
——–
Amount loaned to company $ 80,000
========
2% factoring fee (assessed on entire $100,000) $2,000
10% interest for 30 days (assessed on the entire $80,000) 667
——
Total fees assessed on $80,000 loan $2,667
======
These fees are offset by the savings in collection expenses. This amount is $1,500 for one month ($18,000 divided by 12). The net cost per month is $1,167 ($2,667 - 1,500). The monthly interest rate is 1.46% ($1,167 divided by $80,000). This represents an annual rate of 17.5% (1.46% × 12).

200
Q

In the definition of fair value as presented in FASB ASC 820, the principal market is considered to be a market where:

Correct A.
the holder of the asset or liability could find the greatest volume of asset sales or liability transfers of items similar to the one being valued.

B.
the holder of an asset could maximize the price received in an asset sale or minimize the transfer costs in the conveyance of a liability.

C.
the buyer and seller are not related parties and both have usual and customary knowledge of the items being transferred.

D.
all transfer costs rest solely with either the buyer or the seller.

A

Under the fair value assumption as presented in FASB ASC 820, the hypothetical transaction is considered to have occurred in either the principal market for transactions similar to the subject of the valuation or the most advantageous market if a principal market does not exist. The principal market is considered to be one where the holder of the asset or liability being valued could find the greatest volume of similar transfers. The most advantageous market is one where the holder of an asset could maximize the price received in an asset sale or minimize the transfer costs in the conveyance of a liability.

The characteristics of the hypothetical parties to the transaction as being unrelated and having usual and customary knowledge of the items being transferred do not define the principal market involved in the hypothetical transaction.

201
Q

A company that is conservative with respect to its working capital policy would tend to have a:

A.
low current ratio.

B.
low ratio of current assets to fixed assets.

Correct C.
low ratio of current liabilities to long-term financing.

D.
short operating cycle.

A

A company which wishes to minimize risk will tend to have a low ratio of current liabilities to long-term financing, as the latter entails less risk of default. It would want to have a high current ratio (current assets to current liabilities) and a longer operating cycle.

202
Q

Many organizations are critically dependent on information systems to support daily business operations. Consequently, an organization may incur significant loss of revenues or incur significant expenses if a disaster such as a hurricane or power outage causes information systems processing to be delayed or interrupted. A bank, for example, may incur significant penalties as a result of missed payments.

Which of the following activities is necessary to determine what would constitute a disaster for an organization?

A.
Risk analysis

Incorrect B.
File and equipment backup requirements analysis

C.
Vendor supply agreement analysis

D.
Contingent facility contract analys

A

Risk analysis is necessary to determine an organization’s definition of a disaster and evaluate the effect of that disaster.

System backup analysis, vendor supply agreement analysis, and contingent facility contract analysis are all contingency planning strategies to react to a disaster.

203
Q

Which of the following balanced scorecard perspectives examines a company’s success in targeted market segments?

A.
Financial

Correct B.
Customer

C.
Internal business process

D.
Learning and growth

A

he balanced scorecard concept is a management method used to help companies achieve the goals in their mission statements. There are four sections, or perspectives, to this method: financial performance, customer knowledge, internal business processes, and learning and growth.

Targeted market segments would be specific groups of customers to whom a company wishes to advertise and sell. The study or review of success in these markets would fall under the customer perspective.

Learning and growth encompasses the continued improvement of the major resource of companies—the training and mentoring of human resources, or employees.
The financial perspective is the more traditional way to manage a business. It is based on financial data such as overall profits and financial ratios.
Internal business processes allow managers to receive information that provides feedback regarding how well the products or services fulfill the overall mission of the business.

204
Q

Edwards Manufacturing Corporation uses the standard economic order quantity (EOQ) model. If the EOQ for Product A is 200 units and Edwards maintains a 50-unit safety stock for the item, what is the average inventory of Product A?

Incorrect A.
250 units

B.
150 units

C.
100 units

D.
50 units

A

he average inventory level when the standard economic order quantity model is used is one-half of the EOQ. The EOQ for Product A is 200 units. One-half of 200 is 100. Add the 50-unit safety stock to arrive at 150 units as the average inventory of Product A.

205
Q

The following information pertains to Lap Co.’s Palo Division for the month of April:

                       Number of Units   Cost of Materials
                       ---------------   ----------------- Beginning work-in-process       15,000           $ 5,500 Started in April                40,000            18,000 Units completed                 42,500 Ending work-in-process          12,500

All materials are added at the beginning of the process. Using the weighted-average method, the cost per equivalent unit for materials is:

A.
$0.59.

B.
$0.55.

C.
$0.45.

Correct D.
$0.43.

A

Weighted average means beginning work-in-process inventory is included with the current period production. Therefore, costs from the previous period ($5,500) are added to those incurred this period ($18,000) and the total ($23,500) is divided by equivalent completed units.

Since material is added at the beginning of the process, all units finished, as well as all those in ending work-in-process inventory, are 100% complete as to material. Equivalent completed units for the weighted-average method include equivalent units completed (100% of 42,500) as well as equivalent units in ending inventory (100% of 12,500) for total of 55,000 equivalent completed units.

Dividing total material costs of $23,500 by equivalent completed units of 55,000 results in $0.43 per equivalent completed unit.

206
Q

The standard direct labor cost to produce one pound of output for a company is presented below. Related data regarding the planned and actual production activities for the current month for the company are also given below.

(Note: DLH = Direct Labor Hours)

Direct Labor Standard:
.40 DLH at $12.00 per DLH = $4.80
Planned production 15,000 pounds
Actual production 15,500 pounds
Actual direct labor costs (6,250 DLH) $75,250
The company’s direct labor rate variance for the current month would be:

A.
$10 unfavorable.

B.
$240 unfavorable.

C.
$248 unfavorable.

Correct D.
$250 unfavorable.

A

$250 unfavorable derives from the actual direct labor hours (6,250) times the difference between the standard direct labor rate ($12.00) and the actual direct labor rate ($75,250 ÷ 6,250 = $12.04). Therefore, 6,250 × ($12.00 - $12.04) = $250.

$10 unfavorable derives from the difference between the planned direct labor hours (15,000 × .4 = 6,000) and the actual direct labor hours (6,250) times the difference between the standard direct labor rate ($12.00) and the actual direct labor rate ($75,250 ÷ 6,250 = $12.04) producing ((6,000 - 6,250) × ($12.00 - 12.04) = $10).
$240 unfavorable derives from the planned direct labor hours (15,000 × .4 = 6,000) times the difference between the standard direct labor rate ($12.00) and the actual direct labor rate ($75,250 ÷ 6,250 = $12.04) producing ((6,000) × ($12.00 - 12.04) = $240).
$248 unfavorable derives from the direct labor hours allowed for actual production (15,500 × .4 = 6,200) times the difference between the standard direct labor rate ($12.00) and the actual direct labor rate ($75,250 ÷ 6,250 = $12.04) resulting in ((6,200) × ($12.00 - 12.04) = $248).

207
Q

Morton Company needs to pay a supplier’s invoice of $50,000 and wants to take a cash discount of 2/10, net 40. The firm can borrow the money for 30 days at 12% per annum plus a 10% compensating balance.

Assuming Morton Company borrows the money on the last day of the discount period and repays it 30 days later, the effective interest rate on the loan is:

A.
13.61%.

Correct B.
13.33%.

C.
13.20%.

D.
13.48%.

A

6,000/50,000-5000=13.33

208
Q

Baby Frames, Inc., evaluates manufacturing overhead by using variance analysis. The following information applies to the month of May:

                                Actual            Budgeted
                             -----------   ------------------------ Number of frames manufactured    19,000        20,000 Variable overhead costs          $4,100        $2 per direct labor hour Fixed overhead costs             $22,000       $20,000; $1 per unit Direct labor hours               2,100 hours   0.1 hour per frame What is the variable overhead efficiency variance?

Incorrect A.
$200 favorable

B.
$200 unfavorable

C.
$400 favorable

D.
$400 unfavorable

A

The variable overhead efficiency variance indicates the dollar result of either efficient or inefficient usage of the cost driver for variable overhead.

For Baby Frames, this cost driver is direct labor hours (DLHs).

The computation is:

Variable overhead = Standard variable x Difference between actual
efficiency variance overhead rate and budgeted DLHs
= $2 (2,100 - (19,000 x .1))
= $2 (2,100 - 1,900)
= $2 (200)
= $400
Since actual DLHs exceeded budgeted DLHs, this variance is unfavorable.

Budgeted DLHs are based on budgeted hours for actual product output (19,000 frames), not what Baby Frames thought it would produce (i.e., 20,000 frames). The idea is to use the budgeted rate (0.1 hour per frame) but multiply it by the actual level of production (19,000 frames).

Terms

209
Q

Quality Supply Company (QSC) has a current ratio 2.5 to 1. Partial end-of-year data for QSC includes:

Cash $ 50,000
Net accounts receivable 80,000
Inventories 120,000
QSC’s quick or acid-test ratio is:

A.
0.5 to 1.

Correct B.
1.3 to 1.

C.
2.1 to 1.

D.
2.5 to 1.

A

Note that:

Current Ratio = Current Assets / Current Liabilities
2.5 = ($50,000 + $80,000 + $120,000) / Current Liabilities
2.5 = $250,000 / Current Liabilities
Therefore:

    (2.5) Current Liabilities = $250,000
          Current Liabilities = $250,000 / 2.5
                              = $100,000

Quick ratio = Quick Assets / Current Liabilities
= ($50,000 + $80,000) / $100,000
= $130,000 / $100,000
= 1.3 to 1

210
Q

Johnson Co., distributor of candles, has reported the following budget assumptions for Year 1: No change in candles inventory level; cash disbursement to candle manufacturer, $300,000; target accounts payable ending balance for year 1 is 150% of accounts payable beginning balance; and sales price is set at a markup of 20% of candle purchase price. The candle manufacturer is Johnson’s only vendor, and all purchases are made on credit. The accounts payable has a balance of $100,000 at the beginning of Year 1. What is the budgeted gross margin for Year 1?

A.
$60,000

Correct B.
$70,000

C.
$75,000

D.
$87,500

A

Accounts payable increased from $100,000 at the beginning of the year to $150,000 at the end of the year (the 150% increase). Purchases of candles include the $300,000 paid in cash plus the $50,000 increase in payables for total purchases of $350,000. Since inventory does not change, the cost of goods sold equals the cost of purchases.

Sales price is 120% times purchase cost, so sales are budgeted at 1.20 × $350,000, or $420,000. Gross margin is sales revenue of $420,000 less cost of sales of $350,000, or $70,000.

211
Q

The following information pertains to a byproduct called Moy:

     Sales in 20X2                 5,000 units
     Selling price per unit                 $6
     Selling costs per unit                  2
     Processing costs                        0 Inventory of Moy was recorded at net realizable value when produced in 20X1. No units of Moy were produced in 20X2. What amount should be recognized as profit on Moy's 20X2 sales?

A.
$0

B.
$10,000

Incorrect C.
$20,000

D.
$30,000

A

Net realizable value equals expected market value (selling price) less any separable processing and selling costs. If byproduct Moy was recorded at net realizable value, the following entry would have been made in 20X1:

Dr. Byproduct Inventory 5,000($6-$2) $20,000
Cr. Work-In-Process $20,000
When the 5,000 units of Moy were sold in 20X2, the sale would be recorded using the following summary journal entry:

Dr. Cash (5,000 x $6) $30,000
Cr. Byproduct Inventory $20,000
Cr. Cash (for selling costs) (5,000 x $2) 10,000
As can be seen, no profit is recognized when byproduct inventory is recorded at net realizable value.

212
Q

A company implements an enterprise resource planning application to help improve its financial and operational reporting, while gaining other efficiencies related to sales and inventory management. For the implementation, the company hires an individual specializing in preparing the company for the changes through documenting new policies and procedures and developing new training. This is an example of:

Correct A.
change management.

B.
a social event.

C.
segregation of duties.

D.
an economic event.

A

Answer A is correct because implementing an ERP application is a change to the entity’s internal controls and documenting the change is part of the process of managing the change.

Answer B is incorrect because documenting an application is part of the entity’s internal controls, not a social event.

Answer C is incorrect because segregating one duty from another is an example of a control. It is not related to documentation of policies or training.

Answer D is incorrect because this is the implementation of a change in the financial reporting system, not an economic event.

213
Q

Selected information concerning the operations of a company for the year ended December 31 is as follows:

Units produced 20,000
Units sold 18,000
Direct materials used $80,000
Direct labor incurred $40,000
Fixed factory overhead $50,000
Variable factory overhead $24,000
Fixed selling and administrative expenses $60,000
Variable selling and administrative expenses $9,000
Work-in-process inventories at the beginning and end of the year as well as the beginning finished goods inventory were zero. What was the company’s finished goods inventory cost at December 31 under the variable (direct) costing method?

A.
$23,900

B.
$19,400

C.
$17,000

Correct D.
$14,400

A

Under the variable costing method, fixed manufacturing costs are assumed to be period costs. Only variable production costs are inventoriable.

The cost of the ending finished goods inventory can be determined by calculating the total product costs incurred during the period and dividing that total into two components:

Cost of goods sold (COGS) for the period
Ending finished goods inventory

Direct materials used      $ 80,000
Direct labor incurred        40,000
Variable factory overhead    24,000
                           --------
Total production costs     $144,000
Units produced             / 20,000
                           --------
Cost per unit              $   7.20

Cost of ending
finished goods inventory = Units in inventory x Cost per unit
= 2,000 x $7.20
= $14,400

214
Q

The following information pertains to Clove Co. for the year ending December 31:

  Budgeted sales                      $1,000,000
  Breakeven sales                        700,000
  Budgeted contribution margin           600,000
  Cashflow breakeven                     200,000 Clove's margin of safety is:

Correct A.
$300,000.

B.
$400,000.

C.
$500,000.

D.
$800,000.

A

Margin of safety = Budgeted sales - Breakeven sales
= $1,000,000 - $700,000
= $300,000

215
Q

A CPA would recommend implementing an activity-based costing system under which of the following circumstances?

A.
The client is a single-product manufacturer.

B.
Most of the client’s costs currently are classified as direct costs.

Correct C.
The client produced products that heterogeneously consume resources.

D.
The client produced many different products that homogeneously consume resources.

A

Activity-based costing (ABC) systems use a two-step process. First, a separate pool accumulates the overhead costs associated with each activity and some distinct measure is found for that activity. Overhead costs from each activity pool are allocated to product lines on the basis of the activity measure. In a second step, the overhead costs accumulated by product line are then allocated to the individual units in the product line.

The idea is that when various products consume significantly different levels of resources, costs can be more accurately assigned by identifying the level of resource use for each different product. Therefore, activity-based costing would be appropriate if the client’s products heterogeneously consume resources (each takes different levels of resources).

A single product manufacturer would not be able to benefit from closely analyzing the cost structure of different products. ABC systems are used to assign indirect costs, not direct costs clearly associated with specific products. If the different products homogeneously consume resources, they all have the same level of resource consumption, so the client would not benefit from analyzing the differences in resource consumption.

216
Q

Which of the following positions best describes the nature of the board of directors of XYZ Co.’s relationship to the company?

A.
Agent

B.
Executive

Correct C.
Fiduciary

D.
Representative

A

The board of directors of XYZ Co.’s relationship to the company is a fiduciary relationship. To understand why, you must first define “fiduciary.” A fiduciary relationship is a legal or ethical relationship of trust between two people, organizations, or other such parties.

217
Q

A profit-maximizing firm operating in a competitive market in the short run will increase output:

A.
until marginal cost begins to rise.

B.
until total revenue equals total costs

Correct C.
as long as marginal revenue is greater than marginal costs.

D.
as long as marginal costs are greater than marginal revenue.

A

In pure competition, a firm will continue to produce until marginal costs = marginal revenue; therefore, as long as marginal revenue is greater than marginal costs, the firm will continue to increase output.

218
Q

Under the balanced scorecard concept developed by kaplan and norton, employee satisfaction and retention are measures used under which of teh following perspectives?

A

Learning and growth
Employee satisfaction and retention measures are used under the learning and growth perspective of the balanced scorecard. Employee satisfaction typically correlates with productivity employee effectiveness and retention. Retention itself often relates to reduced retraining increased opportunity for human resource development and reduced investment in learning curve.

219
Q

Default risk as it relates to accounts receivable includes which of the following?

A.
The chance that the principal will not be paid by the due date

B.
The chance that the cash received will not be the promised amount

C.
The chance that the product may have declined in value if it is necessary to repossess it

Correct D.
All of the answer choices are correct.

A

Default risk as it relates to accounts receivable includes the following:

The chance that the principal will not be paid by the due date
The chance that the cash received will not be the promised amount
The chance that the product may have declined in value if it is necessary to repossess it

220
Q

If a project has differing required rates of return over its life, i.e., 8% in the first three years and 12% in the second three years, the analyst should:

A.
choose the highest rate to compare to the firm’s cost of capital.

B.
choose the lowest rate to compare to the firm’s cost of capital.

Correct C.
compute the project’s net present value using both rates for the appropriate years and accept the project if the net present value is at least zero.

D.
use the average of the multiple rates to compare to the firm’s cost of capital.

A

With differing rates of return in different years, the best way to make a comparison is to calculate the net present value using both rates for the appropriate years and accept those projects with a net present value (NPV) of zero or greater. Net present value is appropriate because it allows discounting different years using the respective required rates of return for those years.

221
Q

Project management is the key to:

Correct A.
planning, implementing, and monitoring.

B.
planning only.

C.
implementing and monitoring only.

D.
None of the answer choices are correct

A

Project management is the key to planning, implementing, and monitoring. It does not stop with only planning, but encompasses all three processes of the project.

222
Q

A preferred stock is sold for $101 per share, has a face value of $100 per share, underwriting fees of $5 per share, and annual dividends of $10 per share. If the tax rate is 40%, the cost of funds (capital) for the preferred stock is:

A.
6.2%.

B.
10.0%.

Correct C.
10.4%.

D.
5.2%.

A

The cost of the preferred stock to the firm will be 10.4%.

The cost to the firm of selling preferred stock is equal to the annual dividend the firm must pay, divided by the net funds received when the stock was sold. Since preferred dividends are not tax deductible, there is no tax adjustment for the cost of preferred stock as there is for debt. The formula for calculating the cost of preferred stock to the firm is as follows:

ks = D1 / (PO - u - f)
Where:

D1 = Annual preferred dividends
PO = Current market price of the stock
u = Underpricing per share, if any
f = Flotation costs paid the investment banker
Here, ks = 10 / (101 - 0 - 5) = 10 / 96 = 0.104, or 10.4%.

223
Q

In a process cost system, the application of factory overhead usually would be recorded as an increase in:

A.
finished goods inventory control.

B.
factory overhead control.

C.
cost of goods sold.

Correct D.
work-in-process inventory control.

A

Process costing is a system of accounting for production in which costs are assigned to units of finished goods indistinguishable from each other and produced in a continuous process.

During the process, raw material cost, direct labor cost, and applied overhead increase the work-in-process inventory account. As units are finished, the cost of those units reduces work-in-process and increases the finished goods account.

“Finished goods inventory control” is incorrect because overhead is applied as work is performed, not when units are finished. “Factory overhead control” is incorrect because the application of overhead to work-in-process would increase work-in-process, not the factory overhead control account. “Cost of goods sold” is incorrect because the application of overhead does not affect cost of goods sold. The overhead applied will be included in work-in-process, which is transferred to finished goods, which is then transferred to cost of goods sold as the goods are sold, but cost of goods sold is not affected when overhead is applied to work-in-process.

224
Q

A client wants to know how many years it will take before the accumulated cash flows from an investment exceed the initial investment, without taking the time value of money into account. Which of the following financial models should be used?

Correct A.
Payback period

B.
Discounted payback period

C.
Internal rate of return

D.
Net present value

A

The payback period method can be used as an easy screening tool for capital budgeting projects since it determines the length of time required to recover the initial cash investment through net cash flows. As a general rule, the sooner the investment is recovered, the better.

Weaknesses of the payback method are the following:

All cash flows occurring after the payback period are ignored.
The time value of money is not considered.

225
Q

A company’s internal controls are established to provide protection for the company’s assets as well as to detect fraud. An internal control allows for the firm’s resources to be all of the following except:

A.
monitored.

Correct B.
designed.

C.
properly used.

D.
measured.

A

A financial transaction control is a procedure that is developed to discover and/or prevent errors, misappropriations, or policy noncompliance in a financial transaction process. Such a control will aid an organization in achieving specific goals and objectives. It is an internal control that allows for a firm’s resources to be properly:

used,
monitored, and
measured.
It is hoped that such controls will detect fraud and provide adequate protection for the company’s assets.

226
Q

Rodder, Inc., manufactures a component in a router assembly. The selling price and unit cost data for the component are as follows:

Selling price $15
Direct materials cost 3
Direct labor cost 3
Variable overhead cost 3
Fixed manufacturing overhead cost 2
Fixed selling and administration cost 1
The company received a special one-time order for 1,000 components. Rodder has an alternative use for production capacity for the 1,000 components that would produce a contribution margin of $5,000. What amount is the lowest unit price Rodder should accept for the component?

A.
$9

B.
$12

Correct C.
$14

D.
$24

A

Rodder needs to sell the components in order to earn as much money as he would have had he used them for his alternate production use. In other words, he needs to determine the sales price that would allow him to keep the $5,000 contribution margin.

The contribution margin is the difference between the revenue for the components and the variable cost of the components.

The variable cost to produce these components is $9 per unit ($3 direct material + $3 direct labor + $3 variable overhead), or $9,000 total ($9 per unit × 1,000 units).

$5,000 (Contribution margin) = Revenue - $9,000
Revenue = $5,000 + $9,000 = $14,000
$14,000 revenue for $1,000 units = $14 per unit
$14 per unit is the lowest price Rodder should accept in order to keep his contribution margin of $5,000.

227
Q

Which of the following types of unemployment typically results from technological advances?

A.
Cyclical

B.
Frictional

Correct C.
Structural

D.
Short-term

A

Structural unemployment arises because of changes in technology and international competitiveness, which change the skills required to perform jobs and/or change the location of jobs.

Cyclical unemployment occurs during declines in the business cycle, and unemployment should be reduced as the economy recovers. Frictional unemployment occurs due to normal labor turnover such as people seeking employment in a higher-paid occupation or another location. Short-term unemployment is incorrect because changing technology often results in longer-term unemployment. Workers laid off due to technological change often find it difficult to obtain jobs without retraining, relocating, or additional education.

228
Q

Which of the following events would decrease the internal rate of return of a proposed asset purchase?

A.
Decrease tax credits on the asset

Incorrect B.
Decrease related working capital requirements

C.
Shorten the payback period

D.
Use accelerated instead of straight-line depreciation

A

Two general rules can be developed related to the internal rate of return (IRR) of a proposed asset purchase:

Increases in cash inflows (decreases in cash outflows) will result in a higher internal rate of return.
The earlier the cash inflows (the later the cash outflows) the higher the internal rate of return, all else being equal.
Taxes in general will lower the IRR of a proposed asset purchase, as will decreases in the tax credits available when an asset is purchased.

229
Q

You walk into a little boutique in the nearby mall. As you walk up to the cash register with an item that you wish to purchase, you notice that there appears to be only one employee in this small store. With a limited number of personnel in the store at any given time, what would be the best internal control procedure to provide a reasonable guarantee that all cash sales are being rung up properly and cash put in the cash drawer?

A.
Carefully screen all new employees

Incorrect B.
Require that all sales be rung up on the cash register using barcodes

C.
Increase the minimum number of employees at the store at any given time to three

D.
Post a sign in a visible spot near the checkout counter that states, “If you do not get a receipt, your purchase is free.”

A

D.
Post a sign in a visible spot near the checkout counter that states, “If you do not get a receipt, your purchase is free.”

Internal controls are designed to provide reasonable assurance that objectives are achieved and compliance to laws and regulations is obtained. All of the items listed would be reasonable control procedures; however, the store receipts may not be able to support a minimum of three employees at the store at any given time. The cost of an internal control procedure is not expected to exceed its benefit. Although it would be important to carefully screen all new employees, it is often difficult to judge an individual’s character during one or two short interviews, and in today’s litigious society, it is often difficult to get valuable information from prior employees or other references.

Following are some of the internal control goals related to this transaction:

Validity: The owner would want only valid, authorized, and legal transactions to be processed. By requiring all transactions to be rung up on the cash register, the owner has the ability to review all transactions. The owner could examine items sold, discounts given, and any other adjustments recorded in sales. However, when only one employee is in the store, it would be difficult to enforce the use of the cash register for cash sales.
Accuracy of recording and evidence of supportability: The owner would want transactions to be recorded free of omissions. By using the customer as a part of the internal control process, the customer can be a “monitor” of the transaction when the owner is not in the store and/or when only one employee is on the selling floor. The transaction would need to be entered into the cash register (recorded) in order to print a register receipt. Given that it may be necessary to have only one employee in the store at slower times during the day, the owner has a final “backup” to have reasonable assurance that the employees are using the other required internal control procedures—thus making this procedure key in the internal control process.

230
Q

For purposes of allocating joint costs to joint products, the sales price at point of sale, reduced by cost to complete after split-off, is assumed to be equal to the:

A.
total costs.

B.
joint costs.

Incorrect C.
sales price less a normal profit margin at point of sale.

D.
relative sales value at split-off.

A

Since joint products frequently are not saleable at split-off, there is no sales value at split-off. Thus, the sales price at point of sale, reduced by cost to complete after split-off, is assumed to be equal to the relative sales value at split-off.

Since the question uses the sales price, it is unrealistic to equate this amount to either total costs or joint costs (costs incurred prior to split-off). The question says nothing about profit margin.

Allocation of joint costs based on relative sales value at split-off is considered the best method and is widely used. Under this method, costs are allocated on an “ability to bear” basis.

231
Q

The use of an accelerated method instead of the straight-line method of depreciation in computing the net present value of a project has the effect of:

A.
raising the hurdle rate necessary to justify the project.

B.
lowering the net present value of the project.

Correct C.
increasing the present value of the depreciation tax shield.

D.
increasing the cash outflows at the initial point of the project.

A

Cash flows in the form of revenue are taxable so revenues must be computed net of tax. Cash outflows in the form of expenses are deductible in computing taxes payable. Therefore, these cash outflows must be computed net of tax also. Depreciation is not a cash flow but it is a deductible expense and therefore affects the amount of taxes payable. An accelerated method of depreciation will cause larger amounts of depreciation to be deducted sooner than straight-line. Although the same amount of tax will be shielded in nominal dollars, the present value of the tax savings (or shield) is greater under accelerated depreciation because less tax is paid out earlier in the project’s life.

232
Q

Which of the following characteristics would indicate that an item sold would have a high price elasticity of demand?

Correct A.
The item has many similar substitutes.

B.
The cost of the item is low compared to the total budget of the purchasers.

C.
The item is considered a necessity.

D.
Changes in the price of the item are regulated by governmental agency.

A

The price elasticity of demand is the absolute value of the percentage change in quantity demanded divided by the percentage change in price. If the elasticity is greater than 1.0, the elasticity is elastic.

If there are many similar substitutes available, many fewer units of this product will be demanded as the price increases, so the change in quantity purchased will be greater than the change in price, giving an elasticity greater than 1.0, a high price elasticity of demand.

The other answer choices all represent situations where a purchaser would react to a price change, but not to the same percentage extent as the percentage price change. “The cost of the item is low compared to the total budget of the purchasers” is incorrect because a change in price of an inexpensive product would not be significant to buyers. “The item is considered a necessity” is incorrect because many buyers will continue to purchase necessities even with a price increase. “Changes in the price of the item are regulated by governmental agency” is incorrect because the government regulation might change the prices, but the effect on quantities purchased could be either greater or lesser than the price change effect.

233
Q

When performing a fair value valuation, John CPA has found a quoted market price for a similar asset to the one held by the reporting organization. There are some questions, however, related to the condition of the asset being valued in comparison to the similar asset. John should:

A.
consider this a Level 1 fair value measurement since a current market price was available for an asset similar to the one being valued.

B.
consider this a Level 2 fair value measurement since a current market price was available for an asset similar to the one being valued.

Correct C.
consider this a Level 3 fair value measurement if the potential adjustments necessary due to the condition of the assets being valued merit the classification of an unobservable input.

D.
make all possible efforts to find another input that will be a closer comparison to the subject asset.

A

Per FASB ASC 820, there are three groups of inputs used when developing fair value:

Level 1: directly observable inputs of identical items, such as quoted active market prices
Level 2: directly or indirectly observable inputs of similar items
Level 3: unobservable inputs
In John’s valuation situation, the quoted market price is for the allegedly similar asset; therefore, this can be no more than a Level 2 fair value measurement. However, since there is a question as to the similarity of the conditions between the subject asset and the comparable, John will need to consider whether these questions related to the condition of the subject asset are significant enough to lower the fair value measurement to a Level 3.

Although FASB ASC 820 expects that the CPA will attempt to use more observable than unobservable inputs when doing a fair value valuation, it only requires the use of information that is available without undue cost or effort.

234
Q

The proper discount rate to use in calculating certainty equivalent net present value is the:

A.
risk-adjusted discount rate.

Correct B.
risk-free rate.

C.
cost of equity capital.

D.
cost of debt.

A

Normally, investment projects are perceived to be subject to some degree of risk with regards to amount and timing of future cash flows. This risk is incorporated into the analysis by using a discount rate higher than the risk-free rate. If there is no risk (i.e., the cash flows are certain), the appropriate discount rate is the risk-free rate.

235
Q

Given that demand exceeds capacity, that there is no spoilage or waste, and that there is full utilization of a constant number of assembly hours, the number of components needed for an assembly operation with an 80% learning curve should:

increase for successive periods.
decrease per unit of output.
A.
I only

B.
II only

Incorrect C.
Both I and II

D.
Neither I nor II

A

The learning curve is a graphical description of the learning process that shows the impacts of learning over a number of practice opportunities on work behaviors. The learning curve usually shows increases in work performance as an employee integrates learning experiences (classes, on-the-job training, etc.) into work practices. A basic assumption of the learning curve model is that the direct labor required for the n + 1st unit will always be less than the labor required for the n unit.

Since demand exceeds supply, the company will keep increasing production with a constant number of assembly hours. Increased production requires more units of raw material (components).

236
Q

Joan CPA has been engaged to do some estate planning for a client. As part of that estate planning, Joan is expected to do a valuation of the client’s small business. In the process of performing this engagement, Joan will be determining the ________ of the small business.

A.
price

B.
worth

Correct C.
value

D.
cost

A

The terms “price,”“worth,” and “value” are often used interchangeably when dealing with business-related topics. In actuality, however, these terms have quite different meanings. Price is the observed exchange price that occurs in the marketplace; therefore, it is not determined in the process of doing a business valuation, due to the fact that no actual transaction is taking place.

Worth refers to the advantages of ownership based upon the perceived benefits at a particular point in time and for a particular use. Joan’s client is probably concerned about worth, but worth is not the focus of a business valuation.

Value is the determination of what would be received in an exchange between two willing parties in an arm’s-length transaction in the marketplace where both parties had acted with knowledge, with prudence, and without compulsion. This is the focus of a business valuation.

Cost is what was paid for an asset in the marketplace and is not the subject of a business valuation.

237
Q

Which of the following types of variances would a purchasing manager most likely influence?

Correct A.
Direct materials price

B.
Direct materials quantity

C.
Direct labor rate

D.
Direct labor efficiency

A

A purchasing manager contracts for purchases of raw materials, which affects the price per unit that is used in computing the direct materials price variance.

Direct materials quantity is incorrect because the quantity purchased is determined by the requisition from the using department, not negotiated by the purchasing manager. Direct labor rate and direct labor efficiency are incorrect because the purchasing manager purchases materials, not labor.

238
Q

Regarding the requirements of the Sarbanes-Oxley Act, officers of a company are not permitted to:

Correct A.
move the activities of the organization outside of the United States to avoid complying with the Sarbanes-Oxley Act.

B.
report deficiencies of internal controls.

C.
report material misstatements.

D.
keep the organization transparent.

A

Officers of an organization are not permitted to move the activities of the company outside of the United States in order to avoid the Sarbanes-Oxley Act requirements.

239
Q

Mutual interdependence means that:

A.
each firm is an oligopolistic industry produces a product that is a close substitute for those produced by rival firms.

B.
when a monopolist chooses a price for its product, the quantity it will produce is dependent on the demand curve the firm faces.

Correct C.
each firm in an oligopolistic industry must consider the reactions of its rivals when it makes decision concerning how to price its product.

D.
when a monopolistic competitive firm chooses the type of product differentiation to pursue, it is dependent on the desires and whims of the consumer.

A

The term “mutual interdependence” relates to the fact that the outcome of pricing decisions in an oligopoly is dependent upon the reactions of organization’s rivals.

240
Q

All of the following are the rates used in net present value analysis except for the:

A.
cost of capital.

B.
hurdle rate.

Correct C.
accounting rate of return.

D.
required rate of return.

A

The accounting rate of return is a capital budgeting method or technique which disregards the time value of money. It is not a rate used in a net present value analysis. Each of the other terms—cost of capital, hurdle rate, and required rate of return—describes a rate used in net present value analysis.

241
Q

Asta, Inc., is a medical laboratory that performs tests for physicians. Asta anticipates performing between 5,000 and 12,000 tests during the month of April. Compared to industry averages, at the low range of activity Asta has a lower sales price per test, higher fixed costs, and the same breakeven point in number of tests performed. At the high range of activity, Asta’s sales price per test and fixed costs are the same as industry averages, and Asta’s variable costs are lower. At the low range of activity (0 to 4,999 tests performed) fixed costs are $160,000. At the high range of activity (5,000 to 14,999 tests performed) fixed costs are $200,000.

Sales price per test $60
Variable costs per test 20

What is the number of units Asta must sell to achieve a gross profit of $160,000?

Incorrect A.
4,000

B.
5,000

C.
8,000

D.
9,000

A

The contribution margin per test is sales price less variable cost, or $40 ($60 - $20).

Unit sales necessary to produce a desired profit can be found by adding the profit to fixed costs and dividing the sum by the contribution margin per unit.

At low activity with fixed costs of $160,000, the unit sales necessary for a profit of $160,000 is fixed costs of $160,000 plus the profit of $160,000 (a total of $320,000) divided by unit contribution margin, or $320,000 ÷ $40 = 8,000 tests. This is outside the range within which fixed costs equal $160,000, so this answer is incorrect.

At high activity with fixed costs of $200,000, the unit sales necessary for a profit of $160,000 is fixed costs of $200,000 plus the profit of $160,000 (a total of $360,000) divided by unit contribution margin, or $360,000 ÷ $40 = 9,000 tests. This is the correct answer, since 9,000 tests can be performed with fixed costs of $200,000.

242
Q

The payback reciprocal can be used to approximate a project’s:

A.
net present value.

B.
accounting rate of return if the cash flow pattern is relatively stable.

C.
payback period.

Correct D.
internal rate of return if the cash flow pattern is relatively stable.

A

Payback reciprocal = 1/Payback period

Where:

Payback = Net cash invested/Annual cash inflow
If the cash flow pattern is relatively stable, the payback reciprocal number serves as a good approximation of a present value of an annuity table factor. Using the payback number and a PV of an Annuity table, it becomes a relatively simple matter to look up an interest rate corresponding to the appropriate number of years’ life of a project. This interest rate will be a close approximation of the internal rate of return.

243
Q

There will be a national election in 15 months. Your planning team believes that the current administration in Washington will actively seek to follow the basic tenets of the political business cycle. Given that fact, as you develop your 2-year forecast, you are more likely to:

Correct A.
increase your sales forecast for the near term and plan to access the debt markets earlier than you had otherwise anticipated.

B.
make no changes to your plans since the actions of politicians prior to an election have no impact on the economy.

C.
reduce your sales forecast for the near term and postpone a planned bond issue until after the election.

D.
reduce inventories and loosen your credit policies.

A

According to the political business cycle theory, politicians want the economy to be “pointed in the right direction” as the election approaches with the unemployment rate and the inflation rate falling. Thus, there would be relatively expansionary fiscal policy prior to the election, followed by more restrictive fiscal policy soon after the election, often placing the blame for the reversal on the previous administration if there were a change.

244
Q

During 20X1, Deet Corp. experienced the following power outages:

    Number of
Outages per Month   Number of Months
-----------------   ----------------
        0                   3
        1                   2
        2                   4
        3                   3
                           --
                           12
                           ==
Each power outage results in out-of-pocket costs of $400. For $500 per month, Deet can lease an auxiliary generator to provide power during outages. If Deet leases an auxiliary generator in 20X2, the estimated savings (or additional expenditures) for 20X2 would be:

A.
$(3,600).

B.
$(1,200).

Correct C.
$1,600.

D.
$1,900.

A

No. outages No. of Total Out-of-Pocket Expected
per month x months = outages x Costs = Cost
0 x 3 = 0 x $400 = $ 0
1 x 2 = 2 x $400 = 800
2 x 4 = 8 x $400 = 3,200
3 x 3 = 9 x $400 = 3,600
——
Total expected cost of outages $7,600
Less cost of leasing generator (12 x $500) 6,000
——
Estimated SAVINGS from generator $1,600

245
Q

If income tax considerations are ignored, how is depreciation handled by the following capital budgeting techniques?

Correct A.
Accounting rate of return is included; internal rate of return and payback are excluded.

B.
Internal rate of return and payback are included; accounting rate of return is excluded.

C.
Payback is included; internal rate of return and accounting rate of return are excluded.

D.
Internal rate of return, accounting rate of return, and payback are included.

A

A is correct

If taxes are ignored, depreciation is not a consideration in any of the methods based on cash flows because it is a non-cash expense.
Thus, the internal rate of return, net present value, and payback methods would not consider depreciation because these methods are based on cash flows.
However, the accounting rate of return is based on net income as calculated on an income statement. Because depreciation is included in the determination
of accrual accounting net income, it would affect the calculation of the accounting rate of return.

246
Q

The government has a number of policy options designed to stabilize the level of aggregate demand. If policy makers expected a recession, it might be expected that the government would pursue:

Correct A.
an expansionary monetary policy and an expansionary fiscal policy.

B.
an expansionary monetary policy and a contractionary fiscal policy.

C.
an expansionary fiscal policy and a contractionary monetary policy.

D.
a contractionary monetary policy and a contractionary fiscal policy.

A

During a recession there is insufficient aggregate demand. An expansionary fiscal policy would increase government spending or cut taxes, both of which would increase the level of aggregate demand. An expansionary monetary policy would attempt to decrease interest rates to stimulate business investment and the consumption of durable goods.

247
Q

Spotech Co.’s budgeted sales and budgeted cost of sales for the coming year are $212,000,000 and $132,500,000, respectively. Short-term interest rates are expected to average 5%. If Spotech could increase inventory turnover from its current eight times per year to 10 times per year, its expected cost savings in the current year would be:

A.
$331,250.

B.
$250,000.

Correct C.
$165,625.

D.
$81,812.

A

The key to this problem is to determine how much inventory is reduced by the increased inventory turnover and the resulting savings in interest costs due to reduced working capital requirements. A company must either borrow funds to acquire working capital or give up the next best investment opportunity to fund working capital requirements (opportunity cost). Either way, it costs a company to hold inventory. Any reduction in inventory levels reduces that cost.

The formula for inventory turns is annual cost of sales divided by inventory. Solve for inventory by dividing annual cost of sales by inventory turns. Initially, Spotech has an inventory level of $16,562,500 ($132,500,000 divided by 8 turns). Spotech hopes to decrease the level to $13,250,000 by increasing inventory turns to 10 ($132,500,000 divided by 10 turns). Working capital is reduced by this change in inventory ($16,562,500 - $13,250,000 = $3,312,500). The interest avoided on the $3,312,500 represents a savings of $165,625 ($3,312,500 × 5%).

248
Q

In which of the following situations would there be inelastic demand?

Correct A.
A 5% price increase results in a 3% decrease in the quantity demanded

B.
A 4% price increase results in a 6% decrease in the quantity demanded

C.
A 4% price increase results in a 4% decrease in the quantity demanded

D.
A 3% price decrease results in a 5% increase in the quantity demanded

A

The price elasticity of demand is the absolute value of the percentage change in quantity demanded divided by the percentage change in price. If the elasticity is less than 1.0, the elasticity is inelastic. Since a 3% decrease in quantity demanded results from a 5% price increase, the elasticity of demand is:

0.3 ÷ 0.5 = 0.6
This is a number less than 1.0, indicating inelastic demand.

A 4% price increase that results in a 6% decrease in the quantity demanded, and a 3% price decrease that results in a 5% increase in the quantity demanded, represent elastic demand since the change in quantity is greater than the change in price. A change of 4% in quantity demanded due to a 4% price increase results in an elasticity of exactly 1.0, neither elastic nor inelastic.

249
Q

Your firm currently has on hand some idle cash that will be needed in three months to pay dividends to shareholders. Which of the following would be the most appropriate investment for that cash?

A.
30-year U.S. Treasury bonds with a current annual yield of 7.8%

B.
Ford Motors’ long-term AAA-rated bonds with a current annual yield of 9.25%

C.
Shares of Ford Motors’ common stock, which have been appreciating in price approximately 6% annually and paying a quarterly dividend that is the equivalent of a 5% annual yield

Correct D.
90-day Ford Motors commercial paper with a current annual yield of 6%

A

The most appropriate investment would appear to be the 90-day commercial paper. Commercial paper is short-term unsecured debt that may be issued directly to a purchaser on a discount basis and set to mature on a specific date. By purchasing directly, broker fees and other changes are avoided

250
Q

A company is considering outsourcing one of the component parts for its product. The company currently makes 10,000 parts per month. Current costs are as follows:

                       Per Unit    Total
                       --------   ------- Direct materials              $4      $40,000 Direct labor                   3       30,000 Fixed plant facility cost      2       20,000

The company decides to purchase the part for $8 per unit from another supplier and rents its idle capacity for $5,000/month. How will the company’s monthly costs change?

A.
Decrease $15,000

B.
Decrease $10,000

Correct C.
Increase $5,000

D.
Increase $10,000

A

The product should be produced if the incremental cost to produce the product, including any opportunity cost of idle facilities, is less than the purchase price.

Since the fixed plant charge will not change due to this decision, it is irrelevant and should not be considered. The direct materials and direct labor costs ($40,000 + $30,000 = $70,000) are relevant costs. These incremental costs total $70,000 per month to make the product, while they can buy the part for $80,000 per month, an increase in monthly costs of $10,000.

However, the rental income from renting the idle capacity of $5,000 reduces the monthly cost of purchasing the parts, for a net increase in monthly costs of $5,000.

251
Q

Information related to the economic activity for a country is given as follows with values stated in billions of dollars.

--  Gross domestic product (GDP)            $4,000
--  Transfer payments                          500
--  Corporate income taxes                      50
--  Social Security contributions              200
--  Indirect business taxes                    210
--  Personal income taxes                      250
--  Undistributed corporate profits             25
--  Depreciation                               500
--  Net income earned abroad for the country     0 National income is:

Incorrect A.
$3,500.

B.
$3,290.

C.
$3,515.

D.
$3,265.

A

National income (NI) is defined as net domestic product (NDP), plus net income earned abroad, minus indirect business taxes (e.g., sales taxes). NDP is gross domestic product ($4,000) minus depreciation ($500), or $3,500. Thus, national income is $3,290 ($3,500 NDP + $0 net income earned abroad - $210 indirect business taxes).

Net domestic product         $3,500
Net income earned abroad          0
Indirect business taxes        (210)
                             ------
National income              $3,290
                             ======

$3,500 is the NDP. Recall that NDP = Gross domestic product (GDP) - depreciation.
$3,515 equals personal income (PI). Recall that PI = NI - corporate income taxes and undistributed profits - Social Security contributions + transfer payments (public and private). PI = NI ($3,290) - corporate income taxes ($50) - undistributed corporate profits ($25) - Social Security contributions ($200) + transfer payments ($500) = $3,515.
$3,265 equals disposable income (DI). Recall that DU = PI - personal income taxes. DI = PI ($3,515) - personal income taxes ($250) = $3,265.

252
Q

The optimal capitalization for an organization usually can be determined by the:

A.
maximum degree of financial leverage (DFL).

B.
maximum degree of total leverage (DTL).

Correct C.
lowest total weighted-average cost of capital (WACC).

D.
intersection of the marginal cost of capital and the marginal efficiency of investment.

A

The cost of capital is the weighted-average cost (percentage) that a firm will incur in raising new funds to finance future investment.

Given a need to finance specific assets, the optimal capital structure is determined by computing the lowest weighted-average cost of capital to raise that amount.

The maximum degree of financial leverage (DFL). and maximum degree of total leverage (DTL) are incorrect because leverage is a measure of the portions of capital from debt and from equity; it does not determine the optimal allocation between the two. “Intersection of the marginal cost of capital and the marginal efficiency of investment” is incorrect because the marginal efficiency of investment indicates the sensitivity of the cost of capital to changes in interest rates.

253
Q

The gross margin ratio of a firm is affected by changes in:

A.
cost per unit.

B.
unit sales price.

Correct C.
both cost per unit and unit sales price.

D.
neither cost per unit nor unit sales price.

A

Sales = Unit Price x Number of units
COGS = Unit Cost x Number of Units
Gross Margin = (Unit Price - Unit Cost) x Number of Units
Clearly, both unit cost and unit sales price affect the calculated amount of gross margin and additionally, the gross margin ratio.

254
Q

Gartshore, Inc., is a mail-order book company. The company recently changed its credit policy in an attempt to increase sales. Gartshore’s variable cost ratio for obtaining credit is 70% and its required rate of return is 12%. The company projects that annual sales will increase from the current level of $360,000 to $432,000, but the average collection period on receivables will go from 30 to 40 days. Ignoring any tax implications, what is the cost of carrying additional investment in accounts receivable, using a 360-day year?

A.
$168

B.
$1,512

C.
$2,000

Incorrect D.
$2,160

A

Cost of holding accounts receivable before credit policy change: $360,000 sales ÷ 360 days = $1,000 average daily sales:

30 days average collection period = $30,000 average A/R balance
12% required rate of return = $3,600 annual interest
Cost of holding accounts receivable after credit policy change: $432,000 sales ÷ 360 days = $1,200 average daily sales:

40 days average collection period = $48,000 average A/R balance
12% required rate of return = $5,760 annual interest
$5,760 - $3,600 = $2,160 additional annual interest on holding A/R balance.

However, by stating the variable cost ratio, the problem implies it expects a distinction made between actual investment in A/R and margin earned. The actual investment by Gartshore is its variable cost, which for a mail-order book company represents cost of goods purchased for sale. That is, there is no change in fixed costs. Consequently, the $2,160 needs to be reduced to represent only the interest on the variable cost portion: $2,160 × 70% variable cost = $1,512.

255
Q

Which of the following does not support business process management?

Incorrect A.
Approaches

B.
Systems

C.
Techniques

D.
Measures

A

Business process management is supported by the approaches, techniques, and measures of the organization (not the systems). These processes are analyzed throughout the life of the organization.

After a decade of what has often been conflicting and contentious debate over the meaning of “BPM”, we at BPM.com have joined with other industry experts on the establishment of an official and definitive definition of BPM:

“Business Process Management (BPM) is a discipline involving any combination of modeling, automation, execution, control, measurement and optimization of business activity flows, in support of enterprise goals, spanning systems, employees, customers and partners within and beyond the enterprise boundaries.”

The goal of this effort is to find the definition that closely represents the concept that most people (including both experts and otherwise) have for the term BPM. The goal was not offer judgment on different BPM methods, technologies or products, many of which are discussed on this site. This definition is by design short and concise, yet definitive and complete. Some broader considerations

BPM is a discipline; it is a practice; it is something you do.
Business stems from the state of being busy, and it implies commercially viable and profitable work. A business exists to provide value to customers in exchange for something else of value.
Process means a flow of business activities and seeing those activities as connected toward the achievement of some business transaction. Flow is meant loosely here: the order may or may not be strictly defined.
A person doing BPM must consider a process at the scope of interrelated business activities which holistically cooperate to fulfill a business objective. This is the key difference from a functional view of business where each function might be optimized independent of the other functions. In a complex system like a business, it is well known that local optimization of part of the system will rarely lead to good overall results. A BPM practitioner must consider the metrics of the entire system when evaluating a specific process.
Modeling means that they would identify, define, and make a representation of the complete process to support communication about the process. There is no single standard way to model, but the model must encompass the process.
Automation refers to the work that is done in advance to assure the smooth execution of the process instances. In many cases this means writing software, but it might include building machinery or even creating signage to direct participants.
Execution meaning that instances of a process are performed or enacted, which may include automated aspects. Conceptually, the process instance executes itself, following the BPM practitioner’s model, but unfolding independent of the BPM practitioner.
Control means that the there is some aspect of making sure that the process follows the designed course. This can be strict control and enforcement, or it might be loose control in the form of guidelines, training, and manual practices.
Measurement means that effort is taken to quantitatively determine how well the process is working in terms of serving the needs of customers.
Optimization means that the discipline of BPM is an ongoing activity that builds over time to steadily improve the measures of the process. Improvement is relative to the goals of the organization, and ultimately in terms of meeting the needs of customers.
Enterprise is used here simply to mean a business organization; any organization where people are working together to meet common goals; it does not need to be exceptionally large, and it does not need to be for profit.
The mention of enterprise goals is included here to emphasize that BPM should be done in the context of the goals of the enterprise, and not some small part of it. This might seem a bit redundant in one sense: any improvement of a process must be an improvement in terms of the enterprise goals – anything else would not be called an improvement.
Within and beyond the enterprise boundaries recognizes that the enterprise is part of a larger system. Customers are part of the business process. Their interaction, along with those of employees should be considered as part of the end-to-end interaction.

256
Q

Which one of the following is a sales forecasting technique?

A.
Linear programming

B.
Exponential smoothing

C.
Queuing theory

Incorrect D.
Cost-volume-profit analysis

A

Exponential smoothing is a statistical method that is useful as a sales forecasting technique. This forecasting procedure is a special type of weighted moving average: it is reverse geometric progression in which the effect of past events (in this case sales) is discounted based on some multiple so that the effect which the past event has on current projections decreases as the time since the event increases.

The other methods are not useful in forecasting sales:

Linear programming is a model for the allocation of scarce resources.
Queuing theory relates to the balancing of the cost of waiting with the cost of service; for example, the cost of lost sales resulting from long lines at the cash register versus the cost of opening another cash register.
Cost-volume-profit analysis is a model used to aid decision making relating to product lines, pricing of products, marketing strategy, and utilization of production facilities.

257
Q

A zero-balance account is:

A.
a checking account that substitutes drafts for regular checks so that management does not have to keep funds in the account for check clearing.

B.
a checking account that moves funds from a local bank to a concentration bank; therefore, the balance maintained in the account is zero.

Correct C.
a checking account that maintains a zero-balance since funds only sufficient to cover the checks presented are transferred from another account.

D.
None of the answer choices are correct.

A

A zero-balance account is one that maintains a zero-balance since funds only sufficient to cover the checks presented are transferred from another account.

Payable-through drafts are legal instruments that look like checks, but they are not drawn on the bank. Payable-through drafts are drawn on and approved by the issuing company against its demand deposit account.

Concentration banking is when a company uses a geographically dispersed collection center to speed up the collection process.

258
Q

The Sarbanes-Oxley Act changed the way financial reports are treated. What section of the act requires the CEO to review the financial statements?

A.
Section 202

Correct B.
Section 302

C.
Section 102

D.
Section 402

A

Section 302 of the Sarbanes-Oxley Act requires that CEOs and CFOs certify that the periodic statutory financial statements were reviewed before being signed.

259
Q

There are two types of valuation engagements described by the Statement on Standards for Valuation Services (SSVS 1), issued by the AICPA, that went into effect in 2008. Valid comparisons of these two types of engagements include which of the following?

A.
In a valuation engagement, the valuator is free to use any valuation approach of method deemed to be professional appropriate. In a calculation engagement, the valuator and the client agree upon the specific valuation methods or approaches to be used.

Incorrect B.
The results of a valuation engagement can be either a single number or expressed as a range. The result of a calculation engagement can only be expressed as a single number.

C.
A valuation engagement is used when a full business is being valued. A calculation engagement is used when a minority interest is being valued.

D.
In a valuation engagement, the valuator will use a going concern as the premise of value. In a calculation engagement, the valuator will use liquidation as the premise of value.

A

There are two types of business valuation engagements outlined in the SSVS 1, Valuation of a Business, Business Ownership Interest, Security, or Intangible Asset:

In a valuation engagement, the valuation analyst is free to employ the use of any valuation approach or method that is professionally deemed appropriate under the circumstances. The results are expressed in terms of a conclusion of value and can either be a single number or a range. The premise of value can be either a going concern or liquidation.
In a calculation engagement, the valuation analyst and the client agree upon the valuation methods and approaches to be used; therefore, the analyst is not free to use any approach or method available. The results are expressed in terms of a calculated valued and can be either a single number or a range. The premise of value can be either a going concern or liquidation.

260
Q

Boyle, Inc., makes two products, X and Y, that require allocation of indirect manufacturing costs. The fol­lowing data was compiled by the accountant before making any allocations:

                              Product X   Product Y
                              ---------   --------- Quantity produced                   10,000      20,000 Direct manufacturing labor hours    15,000       5,000 Setup hours                            500       1,500

The total cost of setting up manufacturing processes and equipment is $400,000. The company uses a job-costing system with a single indirect cost rate. Under this system, allocated costs were $300,000 and $100,000 for X and Y, respectively. If an activity-based system is used, what would be the allocated costs for each product?

Correct A.
Product X, $100,000; Product Y, $300,000

B.
Product X, $150,000; Product Y, $250,000

C.
Product X, $200,000; Product Y, $200,000

D.
Product X, $250,000; Product Y, $150,000

A

Using activity-based costing, the setup cost of $400,000 would be allocated based on the activity driver that is most closely related to the cost. The relevant activity for setup costs would be setup time required for each product rather than labor hours. Total setup hours of 2,000 (500 for X and 1,500 for Y) would be allocated using the ratio 500/2,000 × $400,000, or $100,000 for Product X. Product Y would be allocated using the ratio 1500/2000 × $400,000, or $300,000 for Product Y.

261
Q

Johnson Co. is preparing its master budget for the first quarter of next year. Budgeted sales and production for one of the company’s products are as follows:

   Month           Sales         Production
  --------        -------        ----------
  January         10,000           12,000
  February        12,000           11,000
  March           15,000           16,000 Each unit of this product requires 4 pounds of raw materials. Johnson's policy is to have sufficient raw materials on hand at the end of each month for 40% of the following month's production requirements. The January 1 raw materials inventory is expected to conform with this policy.

How many pounds of raw materials should Johnson budget to purchase for January?

A.
11,600

Correct B.
46,400

C.
48,000

D.
65,600

A

Johnson must budget for purchases to cover (1) 60% of the units produced in January (since 40% was left from December) and (2) 40% of units produced in February.

Raw materials needed for January’s production = 12,000 × 4 lbs. of materials per unit × 60% = 28,800 pounds
Raw materials to cover 40% of February’s needs = 11,000 × 4 lbs. of materials per unit × 40% = 17,600 pounds
Total materials to be purchased in January = 28,800 + 17,600 = 46,400 pounds

262
Q

The Sarbanes-Oxley Act requires financial issuers to publish what kind of information?

A.
The immaterial condition of the company

B.
Internal control performance relative to industry best practice benchmarks

C.
Only positive impacts on internal controls

Correct D.
The scope and capabilities of the internal control structure

A

Section 404 of the Sarbanes-Oxley Act requires issuers of annual reports to include the scope and capabilities of the internal control system. It also requires the issuer to include procedures for financial reporting.

263
Q

Under which one of the following conditions is the internal rate of return method less reliable than the net present value technique?

A.
When the net present value of the project is equal to zero

B.
When income taxes are considered in the analysis

C.
When both benefits and costs are included, but each is separately discounted to the present

Correct D.
When there are net cash inflows of sizable amounts early in the project

A

The real issue here is the reinvestment assumption applied to “recovered funds.” Net present value (NPV) assumes reinvestment at the cost of capital whereas internal rate of return (IRR) assumes reinvestment at the IRR.

NPV makes the more realistic assumption about the rate of return that can be earned on cash flows from the project.

264
Q

When is data collection most critical in the implementation of an improvement initiative for a hospital attempting to improve emergency-room care patient satisfaction?

A.
Before the initiative begins

B.
During the initiative

C.
After completion of the initiative

Correct D.
All of the answer choices are correct.

A

Organizations must collect data before, during, and after the implementation to reach a desired target. By performing a SWOT or gap analysis, the organization can first establish a baseline of where it stands and where it needs to go, and also determine if the initiative will fit into the organization as a whole. However, once an initiative has been chosen and launched, ongoing data collection and analysis is critical to determine if the initiative is on target or whether changes need to be made. Once the initiative is fully implemented, additional data is collected and analyzed to determine a cost benefit analysis.

265
Q

How does a change in net investment affect the level of income?

A.
An increase in net investment will be offset directly by a decrease in the level of income.

B.
A decrease in net investment will be offset directly by an increase in the level of income.

C.
A decrease in net investment will cause a more than proportional decrease in the level of income.

Incorrect D.
An increase in net investment will cause a more than proportional decrease in the level of income.

A

In macroeconomics, equilibrium national income is affected by changes in autonomous consumption, net investment, and government expenditures.

The actual impact on national income will be multiplied, positively or negatively, by some multiple of the initial change. This is due to the “multiplier effect,” which magnifies small changes in C, In, or G into larger overall changes to national income.

Thus, a decrease in net investment (In) will decrease national income by a larger amount than the original decline in investment.

266
Q

A firm has $2 million in excess cash that they do not expect to need for approximately six months. They want to receive a good return on their money, but are concerned about not taking an excessive level of default risk or interest rate risk. Which of the following investments does not fit within the firm’s parameters?

A.
A new issue of 10-year Treasury bonds issued at par and yielding 6%

Incorrect B.
A new issue of 180-day, AAA-rated commercial paper yielding 5.1%

C.
A new issue of 180-day T-bills yielding 4.95%

D.
A 10-year federal agency security with a remaining maturity of six months yielding 5.25%

A

The 10-year Treasury bond is subject to significant interest rate risk. If the firm purchased the commercial paper, they would receive $51,000 = ($2,000,000 × .051) ÷ 2. While they would receive $60,000 in interest on the T-bond during the same period, they would have to sell the bond at the end of six months to obtain the needed cash. They would be forced to take a capital loss of more than $9,000 on the sale of the bond if interest rates climbed above 6.18% when the bond had to be sold at the end of the 6-month period.

The other securities all mature at the time the firm expects to need the funds and thus would not expose the firm to interest rate risk.

While commercial paper is unsecured debt, the AAA-rating indicates a low level of default risk.

Federal agency securities are less marketable than Treasury securities, but this security matures at the time the funds are expected to be required. These other securities have slightly higher default risk than Treasury securities.

267
Q

Moss Converters, Inc., uses 100,000 pounds of raw material annually in its production process. Material cost is $12 per pound. The cost to process a purchase order is $45, which includes variable costs of $35 and allocated fixed costs of $10. Out-of-pocket handling and storage costs amount to 20% of the per pound cost. The company’s cost of capital is 15%.

The formula to determine the economic order quantity is:

EOQ = Square root of 2AD/K
Where:

A = Annual unit demand
D = Cost per order
K = Cost of carrying one unit per year
Moss’ economic order quantity is:

A.
1,291 units.

B.
1,464 units.

C.
1,708 units.

Incorrect D.
1,936 units.

A

Annual Demand = A = 100,000 Units (Given)
Cost per Order = D = $35 (Variable Cost Only, Ignore Allocation
of Fixed Cost)

Carrying Cost/Unit/Year = K = Handling Costs + Interest
Handling Cost = 20% of Cost (Given) = (.2) ($12) = $2.40
Interest (Capital Cost) = 15% (Given) of Cost (Given) =
(.15) ($12) = $1.80

Carrying Cost = K = $2.40 + $1.80 = $4.20
EOQ = Sq. Root of 2AD/K = Sq. Root of ((2 x 100,000 x $35)/$4.20)
EOQ = Sq. Root of 7,000,000/4.20
EOQ = Sq. Root of 1,666,666
EOQ = 1,291

268
Q

Universal Air, Inc., supplies instrumentation components to airplane manufacturers. Although there are only a few competitors in this market, the competition is fierce. In order to remain competitive, Universal Air’s executive team conducted a customer survey and developed thirty new indicators to measure middle-management performance. This system was not successful, so new cross-functional teams, consisting of executives and middle managers, were formed to develop new performance measures. To ensure that the cross-functional teams are effective, all necessary resources should be provided. The most important such resource would be:

A.
comfortable meeting rooms.

B.
daily progress reports.

Correct C.
strong top management commitment to the process.

D.
None of the choices listed are necessary.

A

Strong top management commitment to the process that is clearly communicated is extremely important to the success of this participative management effort. Comfortable meeting rooms and frequent progress reports are also important but are not as critical as actual and indicated top management support.

269
Q

Dowell Co. manufactures a wooden item. Which of the following is included with the inventoriable cost under absorption costing and excluded from the inventoriable cost under variable costing?

A.
Cost of electricity used to operate production machinery

Correct B.
Straight-line depreciation on factory equipment

C.
Cost of scrap pieces of lumber

D.
Wages of assembly-line personnel

A

Absorption costing is a method of costing in which both fixed and variable production costs are assigned to the units produced. Fixed costs follow the units through work-in-process and finished goods as an inventoriable cost and are expensed through cost of goods sold when the units are sold.

Direct (variable) costing is a method of costing in which fixed costs are charged to expense as a period cost when incurred. It is more useful for management decision-making because it separates out fixed costs that do not change with the level of activity (volume). Depreciation on factory machinery is part of fixed factory overhead that is inventoried under absorption costing and expensed under variable costing.

The cost of electricity used to operate production machinery is a variable cost that is inventoried under variable costing. The cost of material scrapped is a variable inventoriable product cost under both absorption and variable costing. Wages of assembly-line personnel are a direct labor cost, a variable inventoriable product cost under both absorption and variable costing.

270
Q

In using regression analysis, which measure indicates the extent to which a change in the independent variable explains a change in the dependent variable?

Incorrect A.
p-value

B.
r-squared

C.
Standard error

D.
t-statistic

A

The coefficient of determination (r2) is a ratio that indicates the proportion of variance in the dependent variable determined by the independent variable using the regression equation.

The purpose of regression analysis is to use an independent variable to predict the value of another variable.

It is a mathematical equation used to fit the best line possible to the observed data. It is the line that minimizes the sum of the squares of the lengths of the vertical lines drawn from the data point to the line (i.e., the deviation of the actual observation from the estimate). The smaller the sum of the deviations, the better the fit of the line to the data.

Multiple regression involves the analysis of more than two variables. The key to regression analysis is the computation of the constant and the coefficient b in the linear equation Y = a + bX. Consult a cost accounting or business statistics text for the equations for computing a and b.

271
Q

A company reported a significant material efficiency variance for the month of January. All of the following are possible explanations for this variance, except:

A.
cutbacks in preventive maintenance.

B.
an inadequately trained and supervised labor force.

C.
processing a large number of rush orders.

Correct D.
producing more units than planned for in the master budget.

A

Producing more units than planned in the master budget will not affect the efficiency of the materials used for each unit.

Poorly functioning machines will have more material waste and spoilage.
An inadequately trained and supervised labor force will have more material waste and spoilage than an adequately trained and supervised labor force.
Rush orders disrupt the manufacturing process by interfering with normal work routines, practices, and procedures. These disruptions will adversely affect each of the manufacturing processes, including the efficient use of material, labor, and overhead.

272
Q

A lower than average inventory turnover ratio could be the result of any or all of the following except:

A.
past outages of popular products has led to overstocking.

B.
sales in the last half of the year were less than expected.

C.
several inventory categories contain obsolete goods.

Correct D.
stronger than expected demand for most products.

A

Stronger than expected sales would produce higher cost of goods sold and lower end-of-year inventories than planned.

Since inventory turnover is computed as cost of goods sold divided by average inventory, the overall effect is a higher turnover ratio.

Each of the other conditions mentioned affects cost of goods sold and/or average inventory in a negative manner, causing a lower than expected inventory turnover.

273
Q

In the short-run, average variable cost for a firm is rising; therefore:

A.
average variable cost is below average fixed cost.

B.
average fixed cost is constant.

C.
average total cost is at a minimum.

Correct D.
marginal cost is above average variable cost.

A

In the short-run, average variable cost for a firm is rising; therefore:

A.
average variable cost is below average fixed cost.

B.
average fixed cost is constant.

C.
average total cost is at a minimum.

Correct D.
marginal cost is above average variable cost.

274
Q

During the recessionary phase of a business cycle:

A.
costs generally rise dramatically forcing consumers to delay spending.

B.
the natural rate of unemployment will begin to climb.

Correct C.
there will be a decline in the number of hours worked in an average week for production workers in the manufacturing sector.

D.
the utilization rate for plant and equipment exceeds normal levels, increasing costs, and reducing profits.

A

The recessionary phase is a period in which more resources become unemployed and actual output falls below potential output. There will be a decline in the number of hours in the average workweek since output falls. There will be an increase in unemployment claims; however, the natural rate of unemployment will remain unchanged. Cost will have a tendency to fall.

275
Q

Wright Corporation planned to produce 3,000 units of its single product, Pactium, during November. The standard specifications for one unit of Pactium include six pounds of materials at $.30 per pound. Actual production in November was 3,100 units of Pactium. The accountant computed a favorable materials purchase price variance of $380 and an unfavorable materials quantity variance of $120. Based on these variances, one could conclude that:

A.
more materials were purchased than were used.

B.
more materials were used than were purchased.

Correct C.
the actual cost of materials was less than the standard cost.

D.
the actual usage of materials was less than the standard allowed.

A

A material purchase price variance is computed:

MPPV = Quantity Purchased × Difference between actual and standard price
If this variance is favorable, the actual material price must have been less than the standard material price.

276
Q

Which of the following ratios would be used to evaluate a company’s profitability?

A.
Current ratio

B.
Inventory turnover ratio

C.
Debt to total assets ratio

Correct D.
Gross margin ratio

A

Profitability is the ability of an enterprise to maintain a satisfactory dividend policy while at the same time steadily increasing ownership equity. Gross margin as a percentage of sales revenue is an indication of profitability on those sales.

The current ratio does not measure profit; it measures current assets and current liabilities. The inventory turnover ratio does not measure profit; it measures the time from acquiring inventory to selling it, a current asset. Debt to total assets measures leverage, not profitability.

277
Q

What is the definition of shared services?

A.
Sharing personnel to cut back on costs

B.
Combining two companies who share the same vendor

C.
Creating an external service provider

Correct D.
Combining the efforts of two departments that share the same resources

A

Shared services is combining efforts of two departments that share the same resources. Why have two internal departments working on one common goal and spending twice the money? Shared services combines the two internal departments that can then share in the resources and costs

278
Q

Which one of the following statements relating to the drum-buffer-rope (DBR) theory developed by Dr. Eliyahu Goldratt is incorrect?

A.
DBR assumes that within a manufacturing system, there is at least one (or a limited number) of constraints created by scarce resources.

B.
In order to best protect the throughput of a manufacturing operation, the limiting factor of the manufacturing process must be protected.

C.
It is important to focus on the queuing throughout the entire manufacturing system in order to provide for a smooth transition from one area to another.

Incorrect D.
It is important to protect against inflationary inventory levels (inventory build ups) that can occur at bottlenecks.

A

The drum-buffer-rope theory:

assumes that within a manufacturing system there is at least one (or a limited number) of constraints created by scarce resources.
states that in order to best protect the throughput of a manufacturing operation, the limiting factor of the manufacturing process must be protected.
states that it is important to protect against inflationary inventory levels (inventory build ups) and the associated carrying costs which can occur at bottlenecks (constraints).
focuses on only the queuing area within a manufacturing firm that is in front of the constraint (bottleneck).

279
Q

Which of the following items is one of the eight components of COSO’s enterprise risk management framework?

A.
Operations

B.
Reporting

Correct C.
Monitoring

D.
Compliance

A

The eight components of COSO’s ERM framework are internal environment, objective setting, event identification, risk assessment, risk response, control activities, information and communication, and monitoring. ERM processes must be monitored, deficiencies reported to management, and modifications performed when required.

280
Q

Tam Co. is negotiating for the purchase of equipment that would cost $100,000, with the expectation that $20,000 per year could be saved in after-tax cash costs if the equipment were acquired. The equipment’s estimated useful life is 10 years, with no residual value, and would be depreciated by the straight-line method. Tam’s predetermined minimum desired rate of return is 12%. Present value of an annuity of 1 at 12% for 10 periods is 5.65. Present value of 1 due in 10 periods at 12% is .322.

In estimating the internal rate of return, the factors in the table of present values of an annuity should be taken from the columns closest to:

A.
0.65.

B.
1.30.

Correct C.
5.00.

D.
5.65.

A

The payback period serves as a fair approximation of the annuity factor value from the table of present values of an annuity.

Using the data given:

Payback = Investment / Annual saving
= $100,000 / $20,000
= 5.00
Thus, internal rate of return can be obtained as a percentage rate from an annuity table for 10 periods nearest the annuity factor of 5.00.

Internal rate of return (IRR) is the method used to determine the rate of return that causes the present value of the net cash flows to equal the initial investment. It is a way of evaluating an investment as the present value of the net future cash flows from the investment, expressed as:

Investment = PV (i,t)
…where i (the rate at which the cash flows are discounted) is unknown.

An acceptable or beneficial proposal is one for which the IRR is equal to or greater than the firm’s predetermined minimum acceptable rate of return on the investment.

281
Q

Internal auditors play a role in assessing an organization’s risk management by determining if:

A.
organizational objectives are distinct and separate from the organization’s mission.

B.
significant risks are completely and fully mitigated.

C.
risk responses have been selected that increase the organization’s risk appetite.

Correct D.
relevant risk information is captured and communicated in a timely manner.

A

Internal auditors are required by the International Standards for the Professional Practice of Internal Auditing (set forth by the IIA, Institute of Internal Auditors) to evaluate the effectiveness and contribute to the improvement of risk management processes.

Determining whether risk management processes are effective is a judgment resulting from the internal auditor’s assessment that:

organizational objectives support and align with the organization’s mission,
significant risks are identified and assessed,
appropriate risk responses are selected that align risks with the organization’s risk appetite, and
relevant risk information is captured and communicated in a timely manner across the organization, enabling staff, management, and the board to carry out their responsibilities.
IIA International Standards for the Professional Practice of Internal Auditing 2120

282
Q

A ratio that examines the percentage change in earnings available to common stockholders that is associated with a given percentage change in earnings before interest and taxes is a measure of:

A.
the degree of operating leverage.

Correct B.
the degree of financial leverage.

C.
return on investment.

D.
return on equity.

A

This ratio basically compares the change in earnings after interest and taxes to the change in earnings before interest and taxes. The higher this ratio, the greater the return available to companies who finance their asset purchases with debt.

283
Q

For the current-period production levels, XL Molding Co. budgeted 8,500 board feet of production and used 9,000 board feet for actual production. Material cost was budgeted at $2 per foot. The actual cost for the period was $3 per foot. What was XL’s material efficiency variance for the period?

Incorrect A.
$1,000 favorable

B.
$1,000 unfavorable

C.
$1,500 favorable

D.
$1,500 unfavorable

A

The material efficiency (usage) variance is the difference between the budgeted cost of materials used ($2 × 9,000 actual board feet) and the budgeted cost of materials that should have been used ($2 × 8,500 standard board feet), or $18,000 less $17,000, which is $1,000. The variance is unfavorable because the company actually used more material than the standard amount. The company used an extra 500 board feet at a standard price of $2 per foot for an unfavorable efficiency variance of $1,000.

284
Q

Day Mail Order Co. applied the high-low method of cost estimation to customer order data for the first four months of the current year. What is the estimated variable order filling cost component per order?

Months          Orders           Cost
--------        ------          -------
January         1,200           $ 3,120
February        1,300             3,185
March           1,800             4,320
April           1,700             3,895
                -----           -------
                6,000           $14,520
                =====           =======
Correct	A.	 	
$2.00

B.
$2.42

C.
$2.48

D.
$2.50

A

Note that based on the number of orders, March is the “high” month and January is the “low” month.

Variable
Rate Per = Change in cost / Change in orders
Order
= ($4,320 - $3,120) / (1,800 - 1,200)
= $1,200 / 600 orders
= $2.00 per order

285
Q

As part of a benchmarking process, a company’s costs of quality for the current month have been
identified as follows:

Employee training $20,000
Product recalls 8,000
Scrap 4,500
Quality inspectors 48,000
Preventive maintenance 19,500
Supplier education expense 17,500
Materials inspection expense 60,000
Processing product returns 2,500
What amount is the company's prevention cost for the current month?
a. $39,500
b. $57,000
c. $165,000
d. $175,500
Explanation
Choice "b" is correct.

I got the answer wrong, and I am guessing the answer “Employee Training + Preventive Maintenance + Supplier Education Expense”

My question is why are materials inspection expense & quality inspectors not included, but supplier education expense included in preventive cost?

  • See more at: http://www.another71.com/cpa-exam-forum/topic/question-regarding-preventive-cost-from-cpa-released-questions#sthash.8NqVaOn6.dpuf
A

Perhaps because material inspection would only detect problems that had already been purchased it wouldn’t prevent them. Same with the quality inspectors.

Prevention costs are the costs of production process changes that reduce the rate at which product defects occur. This category includes employee training ($20,000), preventative maintenance ($19,500), and supplier education ($17,500).

The cost of quality inspectors is an inspection cost that identifies a defect but does not prevent it. Internal failure costs include reworking or scrapping defective products that are identified by the inspection process. External failure costs include warranty and repair expenses and product recalls.

286
Q

Kerner Manufacturing uses a process cost system to manufacture laptop computers. The following information summarizes operations relating to laptop computer model no. KJK20 during the quarter ending March 31:

                                              Direct
                                    Units     Labor
                                    -----    -------- Work-in-process inventory, January 1     100     $ 50,000 Started during the quarter               500  Completed during the quarter             400 Work-in-process inventory, March 31      200 Costs added during the quarter                   $720,000 Beginning work-in-process inventory was 50% complete for direct labor costs. Ending work-in-process inventory was 75% complete for direct labor costs. What is the total value of direct labor costs in ending work-in-process inventory using the weighted-average unit cost inventory valuation method?

A.
$183,000

B.
$194,000

Correct C.
$210,000

D.
$216,000

A

The weighted-average process cost method includes in equivalent units (EUs) all units completed plus work done on ending work-in-process (WIP).

Thus:

Weighted-average EUs = Units completed + Work done on ending WIP
For Kerner Manufacturing:  
Direct labor EUs = 400 + 75% (200)
                 = 400 + 150 
                 = 550

Cost per direct labor EU = Total labor cost / Equivalent units
= ($50,000 + $720,000) / 550
= $770,000 / 550
= $1,400

Ending direct labor WIP = 150 units in ending WIP x $1,400 per EU
= $210,000

Under the FIFO assumption, only the costs incurred this period are allocated between finished goods and ending work-in-process (WIP) because FIFO maintains beginning inventory costs that are completely separate from current-period costs.

Goods finished this period are costed separately as either started last period or started this period.
Under FIFO, equivalent units of production (EUP) equals work done in the current period on Beginning WIP, plus the units started and completed in the current period, plus the work done on Ending WIP.
The weighted-average assumption averages all materials and all processing (conversion) costs (both those incurred this period and those in Beginning WIP).

No distinction is made between goods started in the preceding and the current periods.
EUP differ from the FIFO calculation by the amount of EUP in Beginning WIP.
EUP under weighted average equals the EUP transferred to finished goods plus the EUP in Ending WIP.

287
Q

Kane Corp. estimates that it would incur a $100,000 cost to prepare a bid proposal. Kane estimates also that there would be an 80% chance of being awarded the contract if the bid is low enough to result in a net profit of $250,000. What is the expected value of the payoff?

A.
$0

Incorrect B.
$150,000

C.
$180,000

D.
$220,000

A

Expected value is the mean or average value of a random variable over the possible outcomes. It is calculated by weighting the value of each possible outcome by its probability and summing over all values.

Here there are two possible outcomes:

$250,000 profit with an 80% probability gives an expected value of 0.8 × $250,000, or $200,000.
Zero profit with a 20% probability gives an expected value of -$100,000 (the cost of preparing the bid) for an expected value of -$20,000.
Summing an expected value of $200,000 with an expected loss of $20,000 gives a net expected value of $180,000.

We would not want to subtract the $100,000 cost of preparing the bid from the $250,000 profit because profit means revenue minus expenses. Therefore, the $250,000 profit is the amount after the $100,000 cost of preparing the bid has been subtracted from revenue.

288
Q

Wexford Co. has a subunit that reported the following data for Year 1:

Asset (investment) turnover: 1.5 times
Sales: $750,000
Return on sales: 8%
The imputed interest rate is 12%. What is the division residual income for Year 1?

A.
$60,000

B.
$30,000

C.
$20,000

Correct D.
$0

A

The profit margin on sales is a profitability ratio that measures the profit generated from each dollar of sales. Profit margin on sales can be computed for gross profit, operating profit, or net income. Return on sales is net income divided by net sales revenue.

Wexford has a return of 8% on sales revenue of $750,000, so net income is $60,000.

Asset turnover is a measure of how effectively a business unit uses its assets. It equals sales revenue divided by average total assets. If the turnover is 1.5 times, the revenue of $750,000 can be divided by the turnover of 1.5 to find that total assets equal $500,000.

Residual income is operating income less the imputed interest on the assets used to generate the income. The imputed income at 12% multiplied by assets of $500,000 is $60,000. Subtracting imputed income of $60,000 from net income of $60,000 leaves residual income of zero.

289
Q

Using a real options approach in capital investments planning helps managers in dealing with which of the following?

A.
Competitors

B.
Prolonged strikes

C.
Tax issues

Correct D.
Uncertainty

A

The major benefit of using a real options approach is its effectiveness in dealing with uncertainty.

An investment is considered similar to acquisition of stock using options. The option is purchased but will be exercised only if investment in the related shares of stock appears profitable. If not, the option is not exercised and the potential loss minimized. Capital investment projects can be dealt with similarly.

290
Q

Which of the following is not a typical characteristic of a just-in-time (JIT) production environment?

Incorrect A.
Lot sizes equal to one

B.
Insignificant setup times and costs

C.
Push-through systems

D.
Balanced and level workloads

A

The just-in-time (JIT) production environment is characterized by production generated by need. This is a “demand-pull” system in which sales occur first and trigger the production of units. Typical features of a JIT system include small lot sizes, low setup times/costs, and balanced workloads.

Traditional production systems, on the other hand, produce products based on expected demand. They produce on a fixed schedule in a “push-through” mode.

291
Q

The profitability index is a variation on which of the following capital budgeting models?

A.
Internal rate of return

B.
Economic value added

Correct C.
Net present value

D.
Discounted payback

A

The profitability index is also known as the excess present value index. It is calculated by dividing the project’s initial (or average) cost into the present value of the cash flows. If the profitability index is greater than one, then a project’s net present value (NPV) is positive. This index allows comparisons between two projects.

The profitability index is a variation on the net present value method of capital budgeting. The net present value method adjusts for the time value of money by comparing the present value of the estimated net future cash inflows to the cost of the investment. The profitability index uses these same two variables in its calculation.

The internal rate of return budgeting model calculates the rate of discount (interest) that equates the present value of the net cash flows (including the initial cash outlay) to zero. If the resulting rate of return is lower than the desired rate of return, the project should not be undertaken.

The calculation of the economic value added amount (EVA) is not considered a capital budgeting method.

The discounted payback method is the length of time required to recover the initial cash investment using a sum of the discounted future cash flows.

292
Q

Which of the following is a reliable early predictor of future inflation?

A.
Consumer price index

Incorrect B.
Cost-push multiplier

C.
Demand-pull multiplier

D.
Wholesale price index

A

The wholesale price index (WPI) reflects the change in prices of goods at the wholesale level. Since price increases are generally passed on to consumers, the WPI serves as an early predictor for changes in consumer price levels.

293
Q

When there is equilibrium in a monopolistically competitive industry, a firm:

Incorrect A.
will operate efficiently at minimum average total cost.

B.
will not engage in advertising to promote product differentiation.

C.
will operate inefficiently with price greater than marginal revenue.

D.
will be able to make economic profits in the long run.

A

Given free entry and exit in a monopolistically competitive industry, firms only earn normal profits in the long-run. However, since the firm faces a downward-sloping demand curve with MR

294
Q

Following is information relating to Kew Co.’s Vale Division for the current year:

           Sales                         $500,000
           Variable costs                 300,000
           Traceable fixed costs           50,000
           Average invested capital       100,000
           Imputed interest rate               6% What was Vale's residual income?

Correct A.
$144,000

B.
$150,000

C.
$156,000

D.
$200,000

A

Sales $500,000
- Variable costs (300,000)
- Traceable fixed costs ( 50,000)
——–
= Segment margin $150,000
- Imputed interest (6% of $100,000) (6,000)
——–
= Residual income $144,000
========

295
Q

A financial lease:

A.
may normally be canceled by the lessee on 30 days’ notice.

Correct B.
has a duration that corresponds to the useful life of the asset and payments that amortize the cost of the asset while providing the lessor an interest return.

C.
is only available through a bank.

D.
is only available on assets whose economic life exceeds 20 years.

A

Financial leases are financing-type leases. They typically are noncancelable and extend over the life of the leased asset with title transferring to lessee at the end of the lease term. Lease payments “pay for” the asset while providing the lessor with interest income.

296
Q

Which of the following performance measures may lead a manager of an investment center to forgo investments that could benefit the company as a whole?

Correct A.
Return on investment

B.
Residual income

C.
Profitability index

D.
Economic value added

A

Return on investment (ROI) is net income divided by invested capital. It does not consider profitability to the company as a whole because it does not use the corporate cost of capital. A division manager may decide to forego an investment that has an ROI below the division ROI even though it is above the overall corporation cost of capital.

Residual income (RI) is net income above a minimum desired rate of return on invested capital. The minimum desired net income is found by multiplying the desired rate of return by invested capital. Division managers would desire to accept projects that exceed the overall corporate cost of capital.

The profitability index is an analysis of investment alternatives rather than a performance measurement. It is calculated as the present value of the cash flows not counting the initial investment divided by the amount of that investment.

Economic value-added (EVA) is the earnings above the required cost of capital for shareholders; this measure is similar to residual income, but it is applied by external investors rather than the company’s management.

297
Q

Three of the basic measurements used by the theory of constraints (TOC) are:

A.	 	
gross margin (or gross profit), return on assets, and total sales.

B.
number of constraints (or subordinates), number of non-constraints, and operating leverage.

C.
throughput (or throughput contribution), inventory (or investments), and operational expense.

Incorrect D.
fixed manufacturing overhead per unit, fixed general overhead per unit, and unit gross margin (or gross profit).

A

The theory of constraints uses three measurements: throughput contribution, investments, and operating costs.

298
Q

Which of the following indicates that the economy is in a recessionary phase?

A.
The rate of unemployment decreases.

B.
The purchasing power of money declines rapidly.

C.
Potential national income exceeds actual national income.

Incorrect D.
There is a shortage of essential raw materials and costs are rising.

A

In the recession phase of a business cycle, more resources become unemployed so that actual output falls below potential output, so the answer choice “potential national income exceeds actual national income “is correct.

The answer choice “the rate of unemployment decreases” is incorrect because unemployment increases during a recession.

“The purchasing power of money declines rapidly” is incorrect because in a recession more resources become unemployed. This may occur with either inflation or deflation.

“There is a shortage of essential raw materials and costs are rising” is incorrect because in a recession more resources become unemployed. There would be sufficient raw materials, and costs could be dropping or rising due to inflation or deflation.

299
Q

Which of the following phrases defines the internal rate of return on a project?

A.
The number of years it takes to recover the investment

Correct B.
The discount rate at which the net present value of the project equals zero

C.
The discount rate at which the net present value of the project equals one

D.
The weighted-average cost of capital used to finance the project
Which of the following phrases defines the internal rate of return on a project?

A.
The number of years it takes to recover the investment

Correct B.
The discount rate at which the net present value of the project equals zero

C.
The discount rate at which the net present value of the project equals one

D.
The weighted-average cost of capital used to finance the project

A

The internal rate of return is the interest rate that will make the present value of the future net cash flows equal to the initial cash outlay. In other words, it is the interest rate that gives a net present value of zero.

The answer choice “the number of years it takes to recover the investment” is incorrect because it is the definition of the payback period, a number of years, not a rate of return. “The discount rate at which the net present value of the project equals one” is incorrect because the internal rate of return is the rate of return where the net present value is zero, not one. “The weighted-average cost of capital used to finance the project” is incorrect because weighted-average cost of capital is the interest rate that the company is paying on its other sources of financing. It is used to determine that project rate of return that would be acceptable to the company, but it is not used to calculate the internal rate of return.

300
Q

Which of the following is the primary measure of the level of economic activity in the United States?

A.
Disposable income

Correct B.
Gross domestic product

C.
National income

D.
Personal income

A

Gross domestic product is the primary measure of the level of economic activity in the United States.

301
Q

To identify those components of a telecommunication system (i.e., network) that present the greatest risk, the internal auditor should first:

A.
review the open systems interconnect (OSI) network model.

B.
identify the network operating costs.

Correct C.
determine the business purpose of the network.

D.
map the network software and hardware products into their respective layers.

A

Determining the business purpose of the network will be the best first step to identify those components of a telecommunication system which present the greatest risk.

Reviewing the open systems interconnect (OSI) network model may be done as part of audit preparation.
Identifying the network operating costs may be an audit step.
Mapping the network software and hardware products into their respective layers may be a subsequent audit step.

302
Q

Which one of the following statements pertaining to the return on investment (ROI) as a performance measurement is incorrect?

A.
When the average age of assets differs substantially across segments of a business, the use of ROI may not be appropriate.

B.
ROI relies on financial measures that are capable of being independently verified while other forms of performance measures are subject to manipulation.

C.
The use of ROI may lead managers to reject capital investment projects that can be justified by using discounted cash flow models.

Incorrect D.
The use of ROI can make it undesirable for a skillful manager to take on trouble-shooting assignments such as those involving turning around unprofitable divisions.

A

Many financial measures such as ROI, Residual Income and Contribution Margin Percentage are capable of being independently verified.

Companies strive to find performance measures that are not subject to manipulation but also recognize that most financial measures can be subjected to some financial manipulation. Financial management usually monitors possible manipulation practices. For example, with respect to ROI, financial managers will monitor the results to ensure the appropriate use of accruals and capitalization.

303
Q

Using the variable costing method, which of the following costs are assigned to inventory?

A.
Variable selling and administrative costs

B.
Variable factory overhead costs

Incorrect C.
Both variable selling and administrative costs and variable factory overhead costs

D.
Neither variable selling and administrative costs nor variable factory overhead costs

A

Under variable costing, only variable manufacturing costs are assigned to inventory. This would include direct material and labor costs as well as variable overhead costs. All period costs (i.e., selling and administrative costs) would be treated as expenses of the period.

Thus, variable factory overhead costs are assigned to inventory while variable selling and administrative costs are not.

304
Q

Which of the following definitions best characterizes benchmarking?

A.
A technique that examines product and process attributes to identify areas for improvements

Correct B.
The comparison of existing activities with the best levels of performance in other, similar organizations

C.
The development of the most effective methods of completing tasks in a particular industry

D.
The complete redesign of a process within an organization

A

Benchmarking is a tool for improving business processes by comparison with performance leaders.

“A technique that examines product and process attributes to identify areas for improvements” is incorrect because it does not mention comparisons with similar organizations. “The development of the most effective methods of completing tasks in a particular industry” is incorrect because benchmarking is not the development of effective methods; it is emulating the best methods of others. “The complete redesign of a process within an organization” is incorrect because benchmarking does not necessarily involve complete redesign of a process but merely improvements to bring it up to the best performance of similar organizations.

305
Q

The cost approach to valuation is appropriate to use when:

Incorrect A.
relevant guideline data is available.

B.
the value of the firm is basically related to the assets held.

C.
the projected future benefit stream is expected to differ significantly from the past.

D.
a substantial amount of goodwill appears to exist.

A

The use of a cost (asset-based) approach for valuation is appropriate when:

the company is in liquidation.
the company is worth more in liquidation than as a going concern.
the company’s value is basically related to the assets held.
the company has had no income in recent years.
future benefit streams cannot be adequately predicted.

306
Q

According to COSO, an effective approach to monitoring internal control involves each of the following steps, except:

A.
establishing a foundation for monitoring.

Correct B.
increasing the reliability of financial reporting and compliance with applicable laws and regulations.

C.
designing and executing monitoring procedures that are prioritized based on risks to achieve organizational objectives.

D.
assessing and reporting the results, including following up on corrective action where necessary.

A

Monitoring is intended to insure that controls are functioning effectively as designed. The question asks for the answer choice that is not a step in monitoring the system. Increasing financial reporting reliability and compliance with laws and regulations are goals of the internal control structure, but are not steps in the monitoring of that system.

The other answer choices are all steps in the monitoring process, since they relate to evaluating whether the system is functioning as designed.

307
Q

If you know a nation’s net domestic product (NDP), you can compute national income (NI) by:

A.
subtracting the capital consumption allowance.

B.
adding corporate income taxes and transfer payments.

Correct C.
subtracting net foreign factor income earned in the United States and indirect business taxes.

D.
adding corporate income taxes and indirect business taxes.

A

Using the income approach, the relationship between GDP, NNP, NI, PI, and DI can be found as follows:

           GDP
         - Capital consumption allowance
         -------------------------------
           NDP
         - Net foreign factor income
         - Indirect business taxes
         -------------------------------
         NI
         - Social Security contribution
         - corporate income taxes
         - undistributed corporate profits
         \+ transfer payments
         -------------------------------
         PI
         - personal taxes
         -------------------------------
         DI Terms
308
Q

Under human resources policies and procedures, what is an appropriate policy or procedure for managing employees?

A.
Hire employees based only on the cover page of their resume.

B.
Do not promote employees on merit.
Correct C.
Train top management to enforce sanctions against employees violating policies.

A

Human resources policies and procedures should include the following:

Hire employees based on the written job requirements
Verify resumes and perform background checks
Promote on both merit and performance
Train members of the organization n many aspects

309
Q

It is assumed that cash flows are reinvested at the rate earned by the investment in which of the following capital budgeting techniques?

Internal rate of return
Net present value
Yes
Yes
Yes
No
No
Yes
No
No
A

This answer is correct. The internal rate of return method determines the rate of return at which the present value of the cash flows or benefits will exactly equal the investment outlay. This method assumes that cash flows received are reinvested to earn the same internal rate of return. On the other hand, the net present value method requires the selection of a discount rate which represents the minimum rate of return desired. This method assumes that all cash flows received are reinvested at this minimum rate of return and not at the rate earned on the investment.

310
Q

A company is arranging financing for the purchase of a new piece of equipment that has a five-year expected useful life. Which of the following alternative financing arrangements has the lowest effective annual percentage rate if each has a quoted nominal rate of 9.5%?
A five-year term loan with interest compounded annually.
A ten-year term loan with interest compounded semiannually.
A five-year term loan with interest compounded quarterly.
A ten-year term loan with interest compounded monthly.

A

This answer is correct. For any given quoted nominal rate, the least frequent compounding is associated with the lowest effective annual percentage cost. Annual compounding is less frequent than semiannual, quarterly, or monthly. Note that the term of the loan is not relevant to the calculation of the effective annual percentage cost of financing.

311
Q

If two companies, company X and company Y, are alike in all respects except that company X employs more debt financing and less equity financing than company Y does, which of the following statements is true?
Company X has more net earnings variability than company Y.
Company X has more operating earnings variability than company Y.
Company X has less operating earnings variability than company Y.
Company X has less financial leverage than company Y.

A

This answer is correct. Since company X has more debt financing, it has greater fixed financing charges than company Y. Interest payments are fixed financing charges while dividends are not. As a result, company X will have a more volatile net income stream than company Y, all else equal.

312
Q

Assume a company has gone bankrupt and will be liquidated. After liquidating the assets and covering tax liabilities, administration fees, and wage expenses, the following claims remain:

Notes payable $10,000,000
Unsecured bank loans 4,000,000
Subordinated debentures 6,000,000

There is only $10,000,000 available to pay these claims.  How much will be allocated to subordinated debentures?
$0
$3,000,000
$4,000,000
$6,000,000
A

This answer is correct. The subordinated debentures are unsecured and subordinated to other liabilities.

313
Q

The purchase of treasury stock with a firm’s surplus cash
Increases a firm’s assets.
Increases a firm’s financial leverage.
Increases a firm’s interest coverage ratio.
Dilutes a firm’s earnings per share.

A

This answer is correct. Purchase of treasury stock decreases the firm’s assets and stockholders’ equity. Therefore it increases the firm’s financial leverage.

314
Q
The capital structure of a firm includes bonds with a coupon rate of 12% and an effective interest rate is 14%. The corporate tax rate is 30%. What is the firm’s net cost of debt?
8.4%
9.8%
12%
14%
A

Here we use effective market rate eventhough to calculaye pmts we use stated or coupon rate because we do get a dicount if effective is less than stated or coupon, so our cost is really based on Market or

315
Q

The best reason corporations issue Eurobonds rather than domestic bonds is that
These bonds are denominated in the currency of the country in which they are issued.
These bonds are normally a less expensive form of financing because of the absence of government regulation.
Foreign buyers more readily accept the issues of both large and small US corporations than do domestic investors.
Eurobonds carry no foreign exchange risk.

A

This answer is correct. Eurobonds are subject to less stringent registration requirements making them less costly to issue.

316
Q
Which of the following is not a financial intermediary?
Commercial bank.
Mutual fund.
Investment banker.
A commercial business.
A

Close
Forward contracts and swaps are often created and exchanged by financial intermediaries, such as

a. Commercial banks
b. Insurance companies
c. Pension funds
d. Savings and loan associations
e. Mutual funds
f. Finance companies
g. Investment bankers
h. Money market funds
i. Credit unions
The other party to the contract or agreement is referred to as the counterparty.

317
Q

Residual income of an investment center is the center’s:

A.
income plus the imputed interest on its invested capital.

Correct B.
income less the imputed interest on its invested capital.

C.
contribution margin plus the imputed interest on its invested capital.

D.
contribution margin less the imputed interest on its invested capital.

A

Residual income is the amount of net income in excess of the imputed interest on its invested capital, so “income less the imputed interest on its invested capital” is correct. The imputed interest on investment is a rate determined by corporate headquarters to encourage the investment center managers to invest in projects that would return more than that rate since residual income will be increased.

“Income plus the imputed interest on its invested capital” is incorrect because the imputed interest is subtracted from income rather than added.

“Contribution margin plus [or less] the imputed interest on its invested capital” is incorrect because the contribution margin is not used in the computation of residual income.

318
Q

Lin Co. is buying machinery it expects will increase average annual operating income by $40,000. The initial increase in the required investment is $60,000, and the average increase in required investment is $30,000. To compute the accrual accounting rate of return, what amount should be used as the numerator in the ratio?

A.
$20,000

B.
$30,000

Correct C.
$40,000

D.
$60,000

A

Accounting rate of return = Increase in income ÷ Required investment

Increase in income (numerator) = $40,000

319
Q

What is the formula for calculating the profitability index of a project?

A.
Subtract actual after-tax net income from the minimum required return in dollars

Correct B.
Divide the present value of the annual after-tax cash flows by the original cash invested in the project

C.
Divide the initial investment for the project by the net annual cash inflow

D.
Multiply net profit margin by asset turnover

A

The profitability index considers the size of the original investment and the value of the discounted cash flows. It is calculated by dividing the present values of the cash flows after the initial investment by that investment.

The other answer choices are incorrect because they do not compare the present value of the investment’s cash flows with the initial investment.

320
Q

Maylar Corporation has sold $50 million of $1,000 par value, 12% coupon bonds. The bonds were sold at a discount and the corporation received $985 per bond. If the corporate tax rate is 40%, the after-tax cost of these bonds for the first year (rounded to the nearest hundredth percent) is:

Correct A.
7.31%.

B.
12.18%.

C.
12.00%.

D.
7.09%.

A

Interest payment per year ($1,000 x 12%) = $120
Before-tax cost = 120 / 985 = .1218
After-tax cost = .1218 x (1-40%) = 7.31%
Note the use of the $985 proceeds, not the par value, in this calculation.

Alternative Calculation:

   Interest payment per year ($1,000 x 12%)      =  $120
   Tax Savings per year ($120 x 40%)             =  $ 48
                                                    ----
   Effective Interest paid per year              =  $ 72
                                                    ====
   After-tax cost ($72 / $985)                   =  7.31%
321
Q

What is the calculation for the interest payment on a bond?

Correct A.
Stated rate of interest × Par value

B.
Stated rate of interest × Market value

C.
Effective rate of interest × Par value

D.
Effective rate of interest × Market value

A

The calculation for the interest payment on a bond is the Stated rate of interest × Par value. This is the calculation used by the corporation to establish the interest to be paid.

322
Q

A firm’s financial risk is a function of how it manages and maintains its debt. Which of the following sets of ratios characterizes the firm with the greatest amount of financial risk?

Incorrect A.
High debt-to-equity ratio, high interest coverage ratio, stable return on equity

B.
Low debt-to-equity ratio, low interest coverage ratio, volatile return on equity

C.
High debt-to-equity ratio, low interest coverage ratio, volatile return on equity

D.
Low debt-to-equity ratio, high interest coverage ratio, stable return on equity

A

Remember the significance of these ratios:

The debt-to-equity ratio measures the percentage of total financing provided by creditors, in the form of debt, compared to financing by owners.
The interest coverage ratio measures the firm’s ability to make required interest payments.
The return on equity reflects the net income that accrues to owners.
Financial risk increases as the debt-to-equity ratio increases, and as the volatility of returns increases. Both of these suggest difficulty in repaying the debt. A higher interest coverage ratio means greater ability to repay debt.

323
Q

Consumers in Microlandia buy only three general types of products, X, Y, and Z. Each product type is weighted in the consumers market-basket based on the quantity of the item that is purchased. Changes in the prices of these items on a year-to-year basis are given below:

        Average Price per Unit
        ----------------------
Product    Quantity    Year 1    Year 2
-------    --------    ------    ------
   x          20         $15       $18
   y          18          25        28
   z           6          40        38
Using Year 1 as the base year, the country's CPI for Year 2 is:

A.
1.05.

Correct B.
110.3.

C.
90.7.

D.
107.8.

A

In the first year it cost the consumers $990 to purchase the market-basket of goods ((20 × $15) + (18 × $25) + (6 × $40)). In year 2 it took $1,092 to purchase the same market-basket of goods ((20 × $18) + (18 × $28) + (6 × $38)). Thus, if the base year is 100 = ($990 ÷ $990) × 100, the CPI for the second year would be ($1,092 ÷ $990) × 100 = 110.3.

324
Q

ABC Co. had debt with a market value of $1 million and an after-tax cost of financing of 8%. ABC also had equity with a market value of $2 million and a cost of equity capital of 9%. ABC’s weighted average cost of capital would be:

A.
8.0%.

B.
8.5%.

Correct C.
8.7%.

D.
9.0%.

A

The weighted average cost of capital (WACC) is the weighted average of the debt and equity used to finance the assets of the company. This weighted average can be based upon either the current capital structure of the organization or on a particular target structure desired by management, both using market values of the various components.

This problem can be solved by first determining the current weights for each of the capital components.

Capital Item Market Value Weight
———— ———— ——
Debt $1,000,000 33.3%
Equity 2,000,000 66.7%
———- ——
Total $3,000,000 100.0%
Now the weighted average of the cost of capital can be calculated:

Capital Item Weight Cost Weighting Factor
———— —— —- —————-
Debt 33.3% 8% 2.7%
Equity 66.7% 9% 6.0%
—— —-
100.0% WACC 8.7%
Terms

325
Q

A firm’s dividend policy may treat dividends either as the residual part of a financing decision or as an active policy strategy.

Treating dividends as an active policy strategy assumes that:

Correct A.
dividends provide information to the market.

B.
the firm should pay dividends only after investing in all investment opportunities having an expected return greater than the cost of capital.

C.
dividend payments should be made to common shareholders first.

D.
dividends are costly, and the firm should retain earnings and issue stock dividends.

A

Treating dividends as an active policy strategy assumes that dividends provide information to the market. If a firm chooses to use dividend policy as an active policy strategy (rather than as a residual part of a financing decision), the firm is more concerned about the announcement effects of the dividend than the available investment opportunities (alternative uses of the retained earnings). This is particularly important when management wants to signal improved prospects to the outside investors.

326
Q

Variable rate loans reduce the potential interest rate risk of:

A.
borrowers.

B.
lenders.

C.
both borrowers and lenders.

Incorrect D.
neither borrowers nor lenders.

A

Variable rate loans affect both loan parties when market interest rates change. The overall impact is to reduce the risk of significant gains or losses than could affect either borrowers or lenders.

327
Q

Which of the following types of bonds is most likely to maintain a constant market value?

A.
Zero coupon

Correct B.
Floating rate

C.
Callable

D.
Convertible

A

With most bonds, interest rates are fixed, resulting in an identical interest payment for each payment period over the life of the bonds. As current market interest rates rise, the market value of the bonds will go down since that value is based upon the present value of the future cash flows related to the current market rate; therefore, most bonds expose the holder to interest rate risk.

Floating-rate bonds eliminate interest rate risk since the interest rate paid for a given payment period is based upon the prevailing rates in the current bond market; therefore, if interest rates rise, the interest payment will also increase. Since the market value of bonds is based upon the present value of future cash flows, the market value of floating-rate bonds will remain relatively constant.

328
Q

Which of the following is an advantage of net present value modeling?

A.
It is measured in time, not dollars.

B.
It uses the accrual basis, not the cash basis of accounting for a project.

C.
It uses the accounting rate of return.

Correct D.
It accounts for compounding of returns.

A

Advantages of using the net present value method for decision making include the following:

The time value of money is considered (compounding of returns).
Given a perfect market, correct decision advice will be obtained.
A correct ranking will be obtained for mutually exclusive projects given similar lives and investments.
An absolute value is obtained.
Disadvantages of using the net present value method for decision making include the following:

The discount rate is difficult to determine.
Assumptions related to cash flows have to be made that may or may not be correct.

329
Q

The marketable securities with the least amount of default risk are:

A.
federal government agency securities.

Correct B.
U.S. Treasury securities.

C.
commercial paper.

D.
bankers’ acceptances.

A

U.S. Treasury securities are considered to be the financial instrument with the least default risk, followed by securities of federal government agencies. Default risk is simply the possibility that interest or principal payments will not be made.

Bankers’ acceptances are agreements by a business to pay for a transaction (e.g., import), guaranteed by a bank. Commercial paper is unsecured short-term notes of major corporations.

330
Q

Which of the following is an accurate comparison of fair value and fair market value?

A.
Fair market value defines the buyer as hypothetical, whereas fair value defines a specific buyer.

Incorrect B.
Fair market value uses the principal market, whereas fair value uses the most advantageous market.

C.
Fair market value uses the premise that the seller and not necessarily the buyer is willing to enter into the transaction, whereas fair value uses the premise that both the buyer and seller are willing parties.

D.
Fair market value defines the seller as hypothetical, whereas fair value assumes a specific seller.

A

D.
Fair market value defines the seller as hypothetical, whereas fair value assumes a specific seller.

You answered B. The correct answer is D.

Fair market value is probably the most common standard of value used in business valuation engagements. The International Glossary of Business Terms defines fair market value as “the price, expressed in terms of the cash equivalent, at which property would change hands between a hypothetical willing and able buyer and a hypothetical willing and able seller, acting at arm’s length in an open and unrestricted market, when neither is under compulsion to buy or sell and when both have reasonable knowledge of the relevant facts.”

The International Glossary of Business Terms defines fair value as “the price that would be received to sell an asset or paid to transfer a liability in an orderly transaction between market participants at the measurement date.”

The differences between fair market value and fair value are quite substantial:

Fair market value implies a willing buyer and seller, whereas the buyer and seller under fair value are not necessarily willing.
Fair market value defines the seller as hypothetical, whereas there is a specific seller when using fair value.
Fair market value takes advantage of an unrestricted market, whereas fair value uses the principal or most advantageous market.

331
Q

Space Tech Company (STC) specializes in designing and fabricating components used in lunar and planetary landing and exploration systems. At the end of last year, STC reported the following data in its financial statements:

Total Assets        $16,000,000
Total Debt           10,000,000
Sales                22,000,000
Net income            2,000,000
The debt to equity ratio of STC was \_\_\_\_\_\_\_\_ to 1.

A.
.60

B.
.80

C.
1.25

Correct D.
1.67

A

Debt to equity is total debt divided by total stockholders’ equity. STC’s total stockholders’ equity consists of total assets of $16,000,000 less total debt of $10,000,000 for a net of $6,000,000.

Thus, STC’s ratio of debt to equity is:

Debt to Equity = $10,000,000 / $6,000,000
= 1.67

332
Q

Joe CPA has just accepted a business valuation engagement for The Charter Company, a small business. Some of the facts related to this company are as follows:

The company has been in existence for 25 years.
For the last 3 years, the company has shown little, if any, profit.
The owner has recently had some difficulty making expected loan principal repayments.
The owner met with a credit counselor last month and is in the process of making some recommended changes.
As a beginning step in the valuation process, Joe has determined the premise of value to be:

A.
liquidation.

B.
bankruptcy.

C.
reorganization.

Correct D.
a going concern.

A

As defined in the International Glossary of Business Valuation Terms, the premise of value is defined as “an assumption regarding the most likely set of transactional circumstances that may be applicable to the subject valuation.”

The premise of value will be one of two situations: a going concern or liquidation. Although The Charter Company has displayed poor performance in the past few years, the business does not appear to be in liquidation at this time. Companies can falter, regroup, and continue. It appears that The Charter Company is currently in the process of regrouping, as demonstrated by the owner’s attempts to obtain and implement expert advice; therefore, this business valuation should use a “going concern” as the premise of value.

333
Q

A small vacuum cleaner repair shop is located in a once-quiet, out-of-the way neighborhood. Recently, however, a large entertainment complex that will be used for concerts, basketball games, and other major events was just opened a block from this small business. When determining the fair value of this property, the fair value should be based upon:

Incorrect A.
the use of the property as a small vacuum cleaner repair shop.

B.
the use of the property as a small restaurant serving a variety of light meals, exotic desserts, and alcohol.

C.
the use of the property as a small restaurant serving a variety of light meals, exotic desserts, and alcohol only if the current owner plans to sell the property.

D.
the average of the property value based upon its current use and the highest and best use.

A

B.
the use of the property as a small restaurant serving a variety of light meals, exotic desserts, and alcohol.

FASB ASC 820 provides guidance when determining the fair value of an asset or business ownership. The fair value measurement assumes that the business (or item) being valued will be put to the highest and best use that is physically possible, legally permissible, and financially feasible. The result will be the maximization of value. It is important to realize that the highest and best use may not be the current use; therefore, the highest and best use is determined from the viewpoint of the purchaser and not the seller.

334
Q

A potential problem indicated by a higher than industry average inventory turnover is risk of:

A.
high storage costs.

B.
obsolescence.

Correct C.
stockouts.

D.
building up excessive funds in inventory.

A

Inventory turnover is costs of goods sold divided by average inventory. A very high inventory turnover could be the result of very low inventory levels. If this is the case, a serious risk of stockouts could exist. Management should certainly check this situation out.

Each of the other answers would result from excessive inventory holdings (i.e., a lower than average inventory turnover situation).

335
Q

Assume that each day a company writes and receives checks totaling $10,000. If it takes five days for the checks to clear and be deducted from the company’s account, and only four days for the deposits to clear, what is the float?

Correct A.
$10,000

B.
$(10,000)

C.
$50,000

D.
$25,000

A

The delay for the checks and deposits to clear the company’s account is called the “float.” During the float, the company has the use of the cash equal to the amount of checks written but does not have the use of deposits until they clear its bank.

Since it takes five days for the checks to clear (i.e., be deducted from the company’s account), the company has the use of $10,000 for five days or what is essentially a “loan” of $50,000. However, for the four days required for the deposits to clear, the company is losing the use of $10,000 × 4 or $40,000. Thus, the company enjoys a net 1-day float on the checks it writes of $10,000.

Thus, skillful management of the float requires the company to delay the coverage of checks as long as possible and to deposit checks received as quickly as possible.

336
Q

Which of the following terms represents the residual income that remains after the cost of all capital, includ­ing equity capital, has been deducted?

A.
Free cash flow

B.
Market value-added

Correct C.
Economic value-added

D.
Net operating capital

A

Economic value added (EVA) is after-tax operating income less the weighted average cost of capital.

Free cash flow is a measure of financial performance calculated as operating cash flow minus capital expenditures, not a measure of income.

Market value added is the difference between the current market value of a firm measured by the price of stock on the stock exchange and the capital contributed by investors, not the income remaining after cost of capital is deducted.

Net operating capital is a term used to describe working capital (current assets less current liabilities), not the cost of capital.

337
Q

Inflation may be measured using each of the following measures, except:

A.
consumer price index.

Incorrect B.
gross domestic product deflator.

C.
gross domestic product inflator.

D.
wholesale price index.

A

All of the items listed can be used to measure inflation except the gross domestic product inflator. It is not a technique for measurement of inflation. In contrast, the gross domestic product deflator does measure inflation.

338
Q

A firm’s dividend policy may treat dividends either as the residual part of a financing decision or as an active policy strategy.

Treating dividends as the residual part of a financing decision assumes that:

Correct A.
earnings should be retained and reinvested as long as profitable projects are available.

B.
dividends are important to shareholders, and any earnings left over after paying dividends should be invested in high-return assets.

C.
dividends are relevant to a financing decision.

D.
dividends are costly, and the firm should retain earnings and issue stock dividends.

A

Answer (A) is correct. According to the residual theory of dividends, the amount (residual) of earnings paid as dividends depends on the available investment opportunities and the debt-equity ratio at which cost of capital is minimized. The rational investor should prefer reinvestment of retained earnings when the return exceeds what the investor could earn on investments of equal risk. However, the firm may prefer to pay dividends when investment opportunities are poor and the use of internal equity financing would move the firm away from its ideal capital structure.

339
Q

An example of a preventive control activity would be:

Incorrect A.
rotation of duties.

B.
use of passwords.

C.
required vacations.

D.
internal audits.

A

Control activities are included in an organization’s policies, procedures, techniques, and mechanisms to aid management in achieving the firm’s objectives, protect the firm’s assets, and measure performance. These activities can be either preventive or detective.

Preventive activities would include:

separation of duties,
use of passwords,
required authorizations,
required approvals,
alarm systems,
use of locks,
security guards and cameras, and
education, training, and monitoring of employees.
Detective activities would include:
audits,
required vacations,
background investigations,
rotation of duties,
variance analysis,
reconciliations, and
physical inventories.Control activities are included in an organization's policies, procedures, techniques, and mechanisms to aid management in achieving the firm's objectives, protect the firm's assets, and measure performance. These activities can be either preventive or detective.
systems,
use of locks,
securi
340
Q

The capital budgeting model that is generally considered the best model for long-range decision making is the:

A.
payback model.

B.
accounting rate of return model.

C.
unadjusted rate of return model.

Correct D.
discounted cash flow model.

A

The capital budgeting model that is generally considered the best model for long-range decision making is the discounted cash flow model because the time value of money (present and future values) is considered.

The payback model and accounting rate of return model are unadjusted rate of return models which do not consider the time value of money.

341
Q

Variable rate loans can be used to reduce the risk associated with changes in interest rates during the term of the loan. The greatest level of risk relates to ________ loans.

Incorrect A.
short-term

B.
intermediate-term

C.
long-term

D.
noninterest-bearing

A

Long-term fixed interest rate loans create the greatest interest-rate risk for both borrowers and lenders.

Borrowers benefit from fixed rates when interest rates go up but lose when they decline. Lenders’ risk is in the opposite direction.

The chance for interest rates to change is much greater over a long time period than over a short time period.

342
Q

Datacomp Industries, which has no current debt and has a beta of .95 for its common stock. Management is considering a change in the capital structure to 30% debt and 70% equity. This change would increase the beta on the stock to 1.05, and the after-tax cost of debt will be 7.5%. The expected return on equity is 16%, and the risk-free rate is 6%. Should Datacomp’s management proceed with the capital structure change?

A.
No, because the cost of equity capital will increase

B.
Yes, because the cost of equity capital will decrease

Correct C.
Yes, because the weighted average cost of capital will decrease

D.
No, because the weighted average cost of capital will increase

A

in the capital structure to 30% debt and 70% equity. This change w

343
Q

A preferred stock is sold for $101 per share, has a face value of $100 per share, underwriting fees of $5 per share, and annual dividends of $10 per share. If the tax rate is 40%, the cost of funds (capital) for the preferred stock is:

A.
6.2%.

B.
10.0%.

Correct C.
10.4%.

D.
5.2%.

A

The cost of the preferred stock to the firm will be 10.4%.

The cost to the firm of selling preferred stock is equal to the annual dividend the firm must pay, divided by the net funds received when the stock was sold. Since preferred dividends are not tax deductible, there is no tax adjustment for the cost of preferred stock as there is for debt. The formula for calculating the cost of preferred stock to the firm is as follows:

ks = D1 / (PO - u - f)
Where:

D1 = Annual preferred dividends
PO = Current market price of the stock
u = Underpricing per share, if any
f = Flotation costs paid the investment banker
Here, ks = 10 / (101 - 0 - 5) = 10 / 96 = 0.104, or 10.4%.

344
Q

Market risk or systemic risk includes all the following except:

Correct A.
company risk.

B.
congressional tax reform.

C.
inflation or recession.

D.
world energy situation(s).

A

Market or systemic risk is risk that cannot be eliminated through diversification. It includes noncontrollable factors such as inflation or recession, fluctuations in world energy markets, and congressional tax reform.

Company risk is not a component of market risk. It is related to a particular company and is much more specific than market risk. Company risk can be alleviated or avoided through diversification.

345
Q

The net presenThe net present value of a proposed investment is negative; therefore, the discount rate used must be:

A.
greater than the project’s internal rate of return.

Incorrect B.
less than the project’s internal rate of return.

C.
greater than the firm’s cost of equity.

D.
less than the incremental

D

A

The internal rate of return (time adjusted rate of return) is the rate which equates the present value of the projected future net cash flows with the cost of the investment. The net present value method seeks to determine whether the present value of estimated future cash inflows at a desired rate of return will be greater or lesser than the cost of the proposed investment. When the net present value is zero, the internal rate of return equals the rate used in computing the net present value. Using the present value formula, one can see that an increase in the discount rate decreases present value. The discount rate can increase until the present value of future cash flows becomes less than the initial outflow, making net present value negative.

                              1
       Present Value  =  F --------
                                  n
                           (1 + i)
If a net present value of zero means the internal rate of return equals the discount rate, then a negative net present value would indicate that the discount rate used is higher than the internal rate of return.
346
Q

Which of the following actions is the acknowledged preventive measure for a period of deflation?

A.
Increasing interest rates

Correct B.
Increasing the money supply

C.
Decreasing interest rates

D.
Decreasing the money supply

A

Deflation, or negative inflation, occurs when prices fall because the supply of goods is higher than the demand for those goods. This is usually due to a reduction in money, credit, or consumer spending. Deflation can occur as a result of a combination of four factors: the supply of money goes down; the supply of other goods goes up; demand for money goes up; or demand for other goods goes down.

To prevent deflation, the opposite actions (i.e., inflationary actions) must take place: increase the money supply; decrease the supply of other goods; decrease the demand for money; or increase the demand for other goods.

The only correct answer choice is increasing the money supply. When the Federal Reserve (the Fed) takes action to increase the money supply, it begins a cycle whereby interest rates decrease and the demand for goods and services increases. Eventually this creates inflationary pressure on the economy, and the Fed then begins to implement deflationary policies.

347
Q

The overall cost of capital is the:

Correct A.
rate of return on assets that covers the costs associated with the funds employed.

B.
average rate of return a firm earns on its assets.

C.
minimum rate a firm must earn on high risk projects.

D.
maximum rate of return on assets.

A

The overall cost of capital is the rate of return on assets that covers the costs associated with the funds employed.

A firm’s overall cost of capital is a weighted average of the costs of the different sources of funds that the firm uses to finance its assets. These sources are usually some combination of debt and equity. Cost of capital usually refers to the cost of long-term sources of funds, such as long-term debt, preferred stock, and common stock. A firm must pay a return to the suppliers of all of these sources of funds. Theoretically, the minimum return that a firm must earn to keep the value of its stock from declining is equal to the weighted average cost of capital. If a firm earns less than this on its total assets, then there are not enough earnings to pay the suppliers of funds what they expect to receive. Since interest costs are fixed and must be paid first, common stockholders are the ones who will usually suffer a shortfall in returns. If common stockholders do not receive at least the minimum return they require for the risk they are taking by holding common stock, then the value of their investment decreases and the value of their stock falls.

348
Q

A top-down risk assessment (TDRA) is done in order for a company to be compliance with SOX 404. The purpose of a TDRA is to do all of the following except:

A.
identify and assess financial reporting elements.

Correct B.
identify acts of fraud and embezzlement and assess the effect these items have had on company performance.

C.
identify and assess the internal control procedures meant to limit the identified risks.

D.
identify and assess the risks related to the financial reporting elements.

A

A top-down risk assessment (TDRA) is used to identify and assess:

financial reporting items.
the risks related to financial reporting.
the internal control procedures meant to limit the identified risks.
The internal control procedures are meant to prevent fraud and embezzlement—not to find evidence of such actions.

349
Q

Kore Industries is analyzing a capital investment proposal for new equipment to produce a product over the next eight years. The analyst is attempting to determine the appropriate “end-of-life” cash flows for the analysis. At the end of eight years, the equipment must be removed from the plant and will have a net book value of zero, a tax basis of $75,000, a cost to remove of $40,000, and scrap salvage value of $10,000. Kore’s effective tax rate is 40%. What is the appropriate “end-of-life” cash flow related to these items that should be used in the analysis?

A.
$27,000

B.
$12,000

Incorrect C.
$(18,000)

D.
$(30,000)

A

The key to solving this problem is separating the cash flow items from the noncash items. The $40,000 cost to remove the asset is a cash outflow. The scrap salvage value of $10,000 is a cash inflow. Both of these items are also part of the net income upon which tax must be computed. The $75,000 loss that will result from the disposal is also part of the net income upon which tax must be computed. However, the loss is not a cash outflow. What is a cash flow is the tax or tax savings in the net income or loss. The “end-of-life” cash flow may be calculated as follows:

Outflow: Cost to remove ($ 40,000)
Inflow: Salvage value $ 10,000
Inflow: Tax savings from
net loss $ 42,000 *
———-
Net cash inflow $ 12,000
* The tax savings is calculated on a net loss of $105,000. The loss is a result of the $65,000 tax loss on the asset disposal ($75,000 tax basis offset by $10,000 scrap value) and the $40,000 cost to remove the asset.

350
Q

Which of the following statements is correct regarding corporate debt and equity securities?

Both debt and equity security holders have an ownership interest in the corporation.
Both debt and equity securities have an obligation to pay income.
A.
I only

B.
II only

C.
Both I and II

Correct D.
Neither I nor II

A

Debt is a form of financing that increases the liabilities of a corporation, whereas equity securities are an ownership interest.

Debt holders receive interest that is a deductible expense to the payer and recognized as income by the debt holder. Due to the contractual nature of debt, the debt holder is obligated to pay the agreed-upon interest.

Equity holders may receive dividends that are a distribution of income earned by the subject company; however, a company is not obligated to pay dividends until the board of directors has declared them.

351
Q

The market approach is one of the three basic methodologies available to the valuator. When determining whether this approach can be employed in a particular engagement (whether guideline companies are available), the valuator must keep all of the following in mind except:

A.
the normalized statements of the subject of the valuation need to use similar GAAP choices, such as LIFO and FIFO, as the guideline companies.

B.
one company does not make a comparable.

Correct C.
the guideline companies will need to be identical to the subject of the valuation.

D.
a guideline company needs to produce (supply) similar products, serve similar markets, and be within a similar size range as the subject company.

A

Finding guideline companies when performing a business valuation is often difficult. When developing a list of potential guideline companies, the business valuator assesses items such as whether the potential guideline company:

uses similar GAAP choices as the subject company.
has a similar product diversification as the subject company.
serves similar markets as the subject company.
has a similar geographic diversification as the subject company.
is of a similar size as the subject company.
has similar financial and operating leverage as the subject company.
has similar liquidity, solvency, growth, and profitability as the subject company.
It is important to note that a business valuation cannot be based upon one guideline organization, nor is a comparable expected to be identical to the subject of the valuation in any manner.

352
Q

To assist in an investment decision, Gift Co. selected the most likely sales volume from several possible outcomes. Which of the following attributes would that selected sales volume reflect?

A.
The mid-point of the range

B.
The median

Correct C.
The greatest probability

D.
The expected value

A

“Most likely” means “greatest probability” (or likelihood) of occurrence.

Mid-point and median are simply the central numbers in an array. They do not convey a specific message as to the probability or likelihood of occurrence.
Expected value goes beyond probability to get an expected dollar value outcome.

353
Q

Maylar Corporation has sold $50 million of $1,000 par value, 12% coupon bonds. The bonds were sold at a discount and the corporation received $985 per bond. If the corporate tax rate is 40%, the after-tax cost of these bonds for the first year (rounded to the nearest hundredth percent) is:

Correct A.
7.31%.

B.
12.18%.

C.
12.00%.

D.
7.09%.

A

Interest payment per year ($1,000 x 12%) = $120
Before-tax cost = 120 / 985 = .1218
After-tax cost = .1218 x (1-40%) = 7.31%
Note the use of the $985 proceeds, not the par value, in this calculation.

Alternative Calculation:

   Interest payment per year ($1,000 x 12%)      =  $120
   Tax Savings per year ($120 x 40%)             =  $ 48
                                                    ----
   Effective Interest paid per year              =  $ 72
                                                    ====
   After-tax cost ($72 / $985)                   =  7.31%
354
Q

When calculating a company’s cost of common stock, an analyst evaluates the following four components: risk-free rate, stock’s beta coefficient, rate of return on the market portfolio, and required rate of return on the company’s stock. Which of the following measurement models is being used?

A.
Constant growth

B.
Weighted marginal cost of capital

Correct C.
Capital asset pricing

D.
Overall cost of capital

A

Capital asset pricing model is the risk of price change due to the unique circumstances of a specific security, as opposed to the overall market that can be virtually eliminated from a portfolio through diversification.

355
Q

A project’s net present value, ignoring income tax considerations, is normally affected by the:

Correct A.
proceeds from the sale of the asset to be replaced.

B.
carrying amount of the asset to be replaced by the project.

C.
amount of annual depreciation on the asset to be replaced.

D.
amount of annual depreciation on fixed assets used directly on the project.

A

Calculation of a project’s net present value is accomplished by:

computing the discounted (present) value of all future cash inflows and outflows of the proposed project.
subtracting the present value of outflows from inflows to arrive at the net present value.
Past costs used to calculate the carrying value and depreciation on the asset(s) to be replaced or used on the project are not used in the present value computation. In addition, depreciation is not a cash flow. Expected proceeds from the sale of the asset to be replaced are a future cash inflow, which will affect the proposed project’s net present value.

356
Q

The cost approach to valuation is appropriate to use when:

A.
relevant guideline data is available.

Correct B.
the value of the firm is basically related to the assets held.

C.
the projected future benefit stream is expected to differ significantly from the past.

D.
a substantial amount of goodwill appears to exist.

A

he use of a cost (asset-based) approach for valuation is appropriate when:

the company is in liquidation.
the company is worth more in liquidation than as a going concern.
the company’s value is basically related to the assets held.
the company has had no income in recent years.
future benefit streams cannot be adequately predicted.

357
Q

In general, it is more expensive for a company to finance with equity capital than with debt capital because:

Correct A.
investors are exposed to greater risk with equity capital.

B.
the interest on debt is a legal obligation.

C.
equity capital is in greater demand than debt capital.

D.
dividends fluctuate to a greater extent than interest rates.

A

Stockholders are the last to be paid, making their risk in the company greater. The greater degree of risk requires a greater reward. Bondholders, on the other hand, can expect a fixed return, known in advance, and therefore have a lower degree of risk.

358
Q

Variable rate loans can be used to reduce the risk associated with changes in interest rates during the term of the loan. The greatest level of risk relates to ________ loans.

A.
short-term

B.
intermediate-term

Correct C.
long-term

D.
noninterest-bearing

A

Long-term fixed interest rate loans create the greatest interest-rate risk for both borrowers and lenders.

Borrowers benefit from fixed rates when interest rates go up but lose when they decline. Lenders’ risk is in the opposite direction.

The chance for interest rates to change is much greater over a long time period than over a short time period.

359
Q

Assume that you borrow $2,000 from a bank and the loan has an 8% annual percentage rate. The loan is to be paid back at the end of 12 months. If the inflation rate during the year was 10%, then:

Correct A.
the dollars that you repay will have less purchasing power than those you borrowed from the bank.

B.
the real rate of return the bank receives on the loan will be greater than was originally expected.

C.
you will actually be paying the bank back fewer dollars than you borrowed.

D.
the inflation will cause income to be redistributed from you to the bank.

A

You have borrowed $2,000 from the bank and the purchasing power of those dollars at the end of the year with a 10% rate of inflation would be $1,800. In this specific instance, even if you add the interest the bank would earn ($160), the purchasing power of the money they receive would be less than the amount lent.

360
Q

A company is investing in a machine costing $365,000. The following table shows selected financial data for the company for the next five years:

Year	Annual Cash Flows	Annual Net Income
1	$50,000	$45,000
2	125,000	120,000
3	150,000	145,000
4	  50,000	  45,000
5	  30,000	  25,000
What is the payback period on this machine?

Correct A.
3.8 years

B.
4.0 years

C.
4.4 years

D.
5.0 years

A

The payback period is the time required to recover the $365,000 initial investment. At the end of the third year cash flows total $325.000, which is $40,000 short of the investment of $365,000. Cash flows in the fourth year are $50,000. If we assume cash flows occur evenly during the year, it takes $40/$50 or 80% of the fourth year cash flows for cash to equal the investment cost. The payback period is three years plus 80% of the fourth year.

361
Q
Question 61:
FINM-0002
Need a hint?See Reference...
Spotech Co.’s budgeted sales and budgeted cost of sales for the coming year are $212,000,000 and $132,500,000, respectively.  Short-term interest rates are expected to average 5%.  If Spotech could increase inventory turnover from its current 8.0 times per year to 10.0 times per year, its expected cost savings in the current year would be
$ 165,625
$0
$3,312,500
$ 828,125
A

This answer is correct. If cost of sales is $132,500,000 and inventory turnover is 8 times per year, average inventory is $16,562,500 ($132,500,000 ÷ 8) If turnover increases to 10 times, average inventory would decrease to $13,250,000 ($132,500,000 ÷ 10). Average inventory would decrease by $3,312,500 ($16,562,500 – $13,250,000), which would save Spotech $165,625 ($3,312,500 × 5%) in interest.

362
Q

A financial lease:

A.
may normally be canceled by the lessee on 30 days’ notice.

Correct B.
has a duration that corresponds to the useful life of the asset and payments that amortize the cost of the asset while providing the lessor an interest return.

C.
is only available through a bank.

D.
is only available on assets whose economic life exceeds 20 years.

A

Financial leases are financing-type leases. They typically are noncancelable and extend over the life of the leased asset with title transferring to lessee at the end of the lease term. Lease payments “pay for” the asset while providing the lessor with interest income.

363
Q

When estimating cash flow for use in capital budgeting, depreciation is:

A.
included as a cash or other cost.

B.
excluded for all purposes in the computation.

C.
utilized to estimate the salvage value of an investment.

Correct D.
utilized in determining the tax costs or benefit.

A

The only effect depreciation expense has on cash flows is the determination of income tax. Depreciation expense is subtracted from income to find taxable income. Therefore, while depreciation is not a cash flow directly, it does affect income tax cash flows.

“Included as a cash or other cost” is incorrect because depreciation is not a cash flow. “Excluded for all purposes in the computation” is incorrect because depreciation is considered in determining cash flows for income taxes. “Utilized to estimate the salvage value of an investment” is incorrect because the salvage value is the cash value of the asset at the end of its useful life, which is not related to the book value remaining after deducting annual depreciation expense.

364
Q

All of the following are valid reasons for a business to hold cash and marketable securities except to:

A.
satisfy compensating balance requirements.

B.
maintain adequate cash needed for transactions.

C.
maintain a precautionary balance.

Correct D.
earn maximum returns on investment assets.

A

Cash and marketable securities are liquid assets that are held to meet short-term financial needs. Valid reasons for a business to hold cash and marketable securities include all of the following:

To satisfy compensating balance requirements
To maintain adequate cash needed for transactions
To meet future needs
To maintain a precautionary balance
The goal of earning maximum returns on investment assets is a long-term goal achieved by investing cash in long-term investments. This is not a valid reason for a business to hold cash and marketable securities.

365
Q

A lower than average inventory turnover ratio could be the result of any or all of the following except:

A.
past outages of popular products has led to overstocking.

B.
sales in the last half of the year were less than expected.

C.
several inventory categories contain obsolete goods.

Correct D.
stronger than expected demand for most products.

A

Stronger than expected sales would produce higher cost of goods sold and lower end-of-year inventories than planned.

Since inventory turnover is computed as cost of goods sold divided by average inventory, the overall effect is a higher turnover ratio.

Each of the other conditions mentioned affects cost of goods sold and/or average inventory in a negative manner, causing a lower than expected inventory turnover.

366
Q

Janet Taylor Casual Wear has $75,000 in a bank account as of December 31, 2007. If the company plans on depositing $4,000 in the account at the end of each of the next three years (2008, 2009, and 2010) and all amounts in the account earn 8% per year, what will the account balance be at December 31, 2010? Ignore the effect of income taxes.

                   8% Interest Rate Factors

Period Future Value of $1 Future Value of an Annuity of $1
—— —————— ——————————–
1 1.0800 1.0000
2 1.1664 2.0800
3 1.2597 3.2464
4 1.3605 4.5061
A.
$87,000

Incorrect B.
$88,001

C.
$96,070

D.
$107,464

A

The problem is set up to enable you to use both the “Future value of $1” factors and the “Future value of an annuity of $1” factors. The initial investment will earn interest for three years (December 31, 2007, until December 31, 2010) and will be factored with a “future value” factor of 1.2597. The three deposits of $4,000 are an annuity of $4,000 for three years. This annuity will use a “future value of an annuity” factor of 3.2464.

Initial Deposit: $75,000 x 1.2597 = $ 94,478
Annuity of $4,000 for 3 years at 8%: $ 4,000 x 3.2464 = $ 12,986
——–
Total account balance at 12/31/10 $107,464

367
Q

The CPA reviewed the minutes of a board of directors meeting of LQR Corp., an audit client. An order for widget handles was outsourced to SDT Corp. because LQR could not fill the order. By having SDT produce the order, LQR was able to realize $100,000 in sales profits that otherwise would have been lost. The outsourcing added a cost of $10,000, but LQR was ahead by $90,000 when the order was completed. Which of the following statements is correct regarding LQR’s action?

Correct A.
The use of resource markets outside of LQR involves opportunity cost.

B.
Accounting profit is total revenue minus explicit costs and implicit costs.

C.
Implicit costs are not opportunity costs because they are internal costs.

D.
Explicit costs are opportunity costs from purchasing widget handles from the resource market.

A

Each decision made by a company involves opportunity cost. Opportunity cost refers to the benefits given up by making one choice over another. Even though LQR Corp. seems to have made the right decision by outsourcing, intangible opportunity costs are always present when one plan of action is chosen over another.

Accounting profit equals revenue minus explicit costs (cash expenditures). Accounting profit does not take into account implicit costs (earnings that could have been received had the resources been used in an alternative use).

Opportunity costs are a form of implicit cost. Explicit costs are not opportunity costs.

368
Q

For the next two years, a lease is estimated to have an operating net cash inflow of $7,500 per annum, before adjusting for $5,000 per annum tax basis lease amortization and a 40% tax rate. The present value of an ordinary annuity of $1 per year at 10% for two years is 1.7355. What is the lease’s after-tax present value using a 10% discount factor?

A.
$2,611

B.
$4,350

C.
$9,569

Correct D.
$11,281

A

Net annual cash inflows = Cash inflow - income taxes
= $7,500 - (.40 x ($7,500 - $5,000))
= $7,500 - (.40 x ($2,500))
= $7,500 - $1,000 = $6,500

Present value of cash inflow = Net annual cash inflow x PV factor
= $6,500 x 1.7355
= $11,280.75, or 11,281 rounded

369
Q

A niche (focus) strategy based on differentiation can be attractive if:

Correct A.
the market has distinctive buyer groups who have specific needs in product attributes or have different uses for the product.

B.
the firm has superior vertical integration and can perform all of the value chain activities necessary to serve the customer.

C.
the buyers are price sensitive and will quickly switch brands in order to obtain the highest perceived value.

D.
the industry is quickly expanding into new markets.

A

A successful niche market requires that the buyers need very specific product attributes. These attributes must differ from those offered on the general market product so much so that the general product does not meet the needs of the niche market. Another possibility would be for the niche market buyers to have different uses for the product, and with its package of attributes, the general product cannot meet the needs for the niche market.

370
Q

FASB ASC 820 discusses the disclosures that need to be presented in financial statements related to fair value measurements. A basic goal of these disclosures is to provide:

Correct A.
enough information to the user of the financial statements so that the inputs used in the fair value measurement can be assessed.

B.
evidence that the fair value measurement is accurate.

C.
a reconciliation of the beginning and ending balances of the various fair value measurements for all nonrecurring fair value measurements.

D.
information on the effect of the fair value measurements on the retained earnings balance.

A

Per FASB ASC 820, the disclosures related to fair value measurements contained in the financial statements need to:

allow for enough information to be provided so that the user of that information can do a reasonable assessment of the inputs used to develop the fair value.
provide for enough information so that the use of significant unobservable inputs (Level 3 inputs) can be assessed.
Although a reconciliation of the beginning and ending balances of balance sheet items using a fair value measurement needs to be provided for all recurring measurements, a reconciliation only needs to be done for nonrecurring fair value measurements when significant unobservable inputs are used (Level 3).

371
Q

A significant decline in the exchange rate of the U.S. dollar generally will have which of the following effects?

A.
It will hurt all U.S. business.

B.
It will benefit U.S. importers.

Correct C.
It will benefit U.S. exporters.

D.
It will make foreign goods cheaper for U.S. consumers.

A

Answer A is incorrect because some U.S. businesses will be helped and others will be hurt. U.S. importers will have to pay more U.S. dollars for goods priced in foreign currencies, increasing costs to the U.S. importers.

Answer B is incorrect because U.S. importers will have to pay more U.S. dollars for goods priced in foreign currencies, increasing costs to the U.S. importers.

Answer C is correct because a decline in the exchange rate of the U.S. dollar will make goods produced in the U.S. less expensive in foreign currencies, improving the competitiveness of U.S. exporters.

Answer D is incorrect because U.S. consumers will have to pay more dollars for goods priced in foreign currencies, making those goods more expensive for those consumers.

372
Q

Variations between business cycles most likely are attributable to which of the following factors?

A.
The law of diminishing returns

B.
Comparative advantage

Correct C.
Duration and intensity

D.
Opportunity costs

A

Variations, or differences, between business cycles in the economy as a whole are due to many factors that result in fluctuations in business activity over time. These are measured by duration (from peak to peak) and intensity (the peaks and troughs in the cycle).

The law of diminishing returns applies to an individual firm in the intensity of its use of fixed costs, not to the business cycle of the economy as a whole. Comparative advantage describes the internal factors within one company that give that firm strengths in comparison to other firms; it does not apply to the economy as a whole. Opportunity costs represent the lost income when a firm chooses to use a resource in one area instead of another; again, this does not apply to the economy as a whole.

373
Q

A firm that often factors its accounts receivable has an agreement with its finance company that requires the firm to maintain a 6% reserve and charges 1% commission on the amount of receivables. The net proceeds would be further reduced by an annual interest charge of 10%. Assuming a 360-day year, what amount of cash (rounded to the nearest dollar) will the firm receive from the finance company at the time a $100,000 account that is due in 90 days is turned over to the finance company?

A.
$93,000

B.
$90,000

C.
$83,000

Correct D.
$90,675

A

Factoring involves the sale of accounts receivable as a way for the seller to obtain financing. However, the seller receives an amount less than the face amount of the accounts sold to compensate the factor for assuming the risk and the costs of collection. In this case, the amount of cash the seller will receive from the finance company is $90,675, computed as follows:

Face amount of accounts receivable factored  =          $100,000
LESS:   6% reserve  =  .06 x $100,000  =  $6,000
     1% commission  =  .01 x $100,000  =  $1,000           7,000
                                                        --------
Net amount available                                    $ 93,000
LESS:  10% interest  =  .10 x $93,000 x (90 / 360) =       2,325
                                                        --------
Cash proceeds                                           $ 90,675
374
Q

If Brewer Corporation’s bonds are currently yielding 8% in the marketplace, why would the firm’s cost of debt be lower?

Incorrect A.
Market interest rates have increased.

B.
Additional debt can be issued more cheaply than the original debt.

C.
Interest is deductible for tax purposes.

D.
There is a mixture of old and new debt.

A

C.

Interest is deductible for tax purposes.

375
Q

Which of the following is the primary reason that many auditors hesitate to use embedded audit modules?

A.
Embedded audit modules cannot be protected from computer viruses.

B.
Auditors are required to monitor embedded audit modules continuously to obtain valid results.

C.
Embedded audit modules can easily be modified through management tampering.

Correct D.
Auditors are required to be involved in the system design of the application to be monitored.

A

An embedded audit module is coded into the information processing software, allowing the auditor to access real data. Often the routines search for unusual items as transactions are processed and report those items to the auditor. There is some danger of data contamination since the routines are working with real data. Also, the auditors must work closely with management in the design of the embedded audit module, which may affect their independence.

“Embedded audit modules cannot be protected from computer viruses” is incorrect because embedded audit modules can be protected from viruses in the same ways as other software, such as with effective firewalls. “Auditors are required to monitor embedded audit modules continuously to obtain valid results” is incorrect because embedded audit modules can accumulate information to be reviewed by the auditor periodically. “Embedded audit modules can easily be modified through management tampering” is incorrect because embedded audit modules can be protected with passwords and biometric controls in the same manner as other software components.

376
Q

The economy appears to be poised to enter into the recovery phase of the business cycle. For firms in the capital goods sector, in terms of the inventory cycle, you would expect that:

Incorrect A.
inventory levels are high as the firms have intentionally increased inventory to meet the increased demand that would be expected to occur when the recovery begins.

B.
inventory levels are high as firms saw unintended inventory accumulate throughout the economic contraction.

C.
inventory levels are low as there was an unanticipated decline in inventory during the preceding contraction phase of the business cycle.

D.
inventory levels are low as firms have intentionally sold off inventories as the economic contraction continued to bring inventories to their desired level.

A

Inventory levels tend to be high as the economy begins the contraction phase of the business cycle and firms cut orders and use their unanticipated inventory to meet demand, attempting to bring inventory levels back to their desired level as contraction continues. Thus, inventory levels tend to be low at the end of the contraction phase due to deliberate management actions.

377
Q

The primary purpose for the use of depository transfer checks, or official bank checks, is to ________ within the banking system.

A.
increase customer perception of companies

Correct B.
move funds from one account to another

C.
reduce ATM usage

D.
spread a company’s funds out

A

Depositor transfer checks are non-negotiable. They are used to move funds from one account to another (i.e., from a local bank to a concentration bank). This allows companies to have flexibility and control over their cash accounts but it can be somewhat time consuming.

378
Q

Mutual interdependence means that:

A.
each firm is an oligopolistic industry produces a product that is a close substitute for those produced by rival firms.

B.
when a monopolist chooses a price for its product, the quantity it will produce is dependent on the demand curve the firm faces.

Correct C.
each firm in an oligopolistic industry must consider the reactions of its rivals when it makes decision concerning how to price its product.

D.
when a monopolistic competitive firm chooses the type of product differentiation to pursue, it is dependent on the desires and whims of the consumer.

A

The term “mutual interdependence” relates to the fact that the outcome of pricing decisions in an oligopoly is dependent upon the reactions of organization’s rivals.

379
Q

Mutual interdependence means that:

A.
each firm is an oligopolistic industry produces a product that is a close substitute for those produced by rival firms.

B.
when a monopolist chooses a price for its product, the quantity it will produce is dependent on the demand curve the firm faces.

Correct C.
each firm in an oligopolistic industry must consider the reactions of its rivals when it makes decision concerning how to price its product.

D.
when a monopolistic competitive firm chooses the type of product differentiation to pursue, it is dependent on the desires and whims of the consumer.

A

The term “mutual interdependence” relates to the fact that the outcome of pricing decisions in an oligopoly is dependent upon the reactions of organization’s rivals.

380
Q

A written policy and procedure manual should contain:

A.
a formal job description.

Incorrect B.
an employee training program.

C.
corporation budgets.

D.
proper business practices.

A

Policies and procedures help the employee understand the organization’s policies for operation and the procedures that are followed to meet the policies. The policies and procedures include such things as the proper business practices, the purpose of the organization, responsibilities, and definitions.

381
Q

In which of the following situations would there be inelastic demand?

Correct A.
A 5% price increase results in a 3% decrease in the quantity demanded

B.
A 4% price increase results in a 6% decrease in the quantity demanded

C.
A 4% price increase results in a 4% decrease in the quantity demanded

D.
A 3% price decrease results in a 5% increase in the quantity demanded

A

The price elasticity of demand is the absolute value of the percentage change in quantity demanded divided by the percentage change in price. If the elasticity is less than 1.0, the elasticity is inelastic. Since a 3% decrease in quantity demanded results from a 5% price increase, the elasticity of demand is:

0.3 ÷ 0.5 = 0.6
This is a number less than 1.0, indicating inelastic demand.

A 4% price increase that results in a 6% decrease in the quantity demanded, and a 3% price decrease that results in a 5% increase in the quantity demanded, represent elastic demand since the change in quantity is greater than the change in price. A change of 4% in quantity demanded due to a 4% price increase results in an elasticity of exactly 1.0, neither elastic nor inelastic.

382
Q

The Committee of Sponsoring Organizations of the Treadway Commission (COSO) has developed a widely accepted and used framework for internal control that was designed to provide reasonable assurance for a company’s objectives related to all items except:

A.
effectiveness and efficiency of operations.

B.
reliability of financial reporting.

Correct C.
expansion of markets.

D.
compliance with laws and regulations.

A

COSO has developed a widely accepted and used framework for internal control that was designed to provide reasonable assurance for a firm’s objects related to:

effectiveness and efficiency of operations,
reliability of financial reporting, and
compliance with laws and regulations.
Although the expansion of markets may be in the strategic plan for a company and may come about due to the effectiveness and efficiency of operations, it is not a focus of the COSO Framework designed for internal control.

383
Q

Short-term interest rates are:

A.
generally lower than long-term rates.

Incorrect B.
generally higher than long-term rates.

C.
lower than long-term rates during periods of high inflation only.

D.
not significantly related to long-term rates.

A

Short-term interest rates are usually lower than long-term rates, because there is less risk involved in a shorter time period, and lenders require less compensation since their money is tied up for a shorter time. Short-term rates are definitely related to long-term rates, but tend to be more volatile, in part due to the exercise of monetary policy actions by the Federal Reserve.

384
Q

According to COSO, the position or internal entity that is best suited, as part of the enterprise risk management process, to devise and execute risk procedures for a particular department is:

A.
the internal audit department.

B.
the chief executive officer.

Correct C.
a manager within the department.

D.
the audit committee.

A

Answer A is incorrect because the internal audit department evaluates risk procedures and should not be in the position of evaluating procedures it has developed.

Answer B is incorrect because the CEO in a large organization does not have the time or knowledge to devise risk procedures for every department.

Answer C is correct because a manager within the department has the most detailed knowledge of risks in that department.

Answer D is incorrect because the audit committee of the board has overall responsibility for the selection of the auditor and receipt of audit results. They do not have detailed knowledge of any one department.

385
Q

The manager of a production line has the authority to order and receive replacement parts for all machinery that require periodic maintenance. The manager typically pays for the parts using a corporate credit card (that bills to the company). The internal auditor received an anonymous tip that the manager ordered substantially more parts than were necessary from a family member in the parts supply business. The unneeded parts were never delivered. Instead, the manager processed receiving documents and charged the parts to machinery maintenance accounts. The manager processed payments for the undelivered parts through the company’s credit card and those payments were sent to the family-member supplier. After the supplier received the money, it was divided between the manager and the family member.

An internal auditor is conducting an audit of the use of corporate credit cards by employees and the supplies ordering process. Which of the following are major audit concerns regarding these issues?

Segregation of duties is insufficient.
The purchasing function is impaired.
Cards may be used for personal benefit.
The company is required to make one large payment instead of many small ones.
A.	 	
II and IV only

B.
III only

C.
I, II, III, and IV

Correct D.
I and III only

A

The segregation of duties is insufficient as there should be another person to process the receiving documents. In the absence of effective monitoring, credit cards could easily be used for personal benefit.

386
Q

In respect to the roles and responsibilities within an internal control framework:

A.
the goals of internal controls are to provide close to absolute assurance that the objectives of the company will be met.

B.
the CEO of an organization is expected to allow his senior staff to set the ethical tone for the organization so as not to micromanage and stifle the organization.

C.
since the board of directors do not devote themselves to the day-to-day operations, they have little influences on the internal control environment.

Correct D.
the internal and external auditors are responsible for the assessment of internal controls in relation to design, implementation, and effectiveness.

A

The internal and external auditors are responsible for the following:

The assessment of whether internal controls are correctly designed
The assessment of whether internal controls are properly implemented
The assessment of whether internal controls are effective
Making recommendations for improvements in the internal control procedures
The CEO of an organization is expected to set an ethical tone and provide direction and leadership in this area for senior staff. The board of directors provides additional guidance to help assure ethical behavior within the organization.

Even though a company has excellent internal controls, there are still unexpected events outside the company’s control, such as economic conditions and the competition that can prevent an organization from meeting its goals.

387
Q

The capital budgeting model that is generally considered the best model for long-range decision making is the:

A.
payback model.

B.
accounting rate of return model.

C.
unadjusted rate of return model.

Correct D.
discounted cash flow model.

A

The capital budgeting model that is generally considered the best model for long-range decision making is the discounted cash flow model because the time value of money (present and future values) is considered.

The payback model and accounting rate of return model are unadjusted rate of return models which do not consider the time value of money.

388
Q

Aggregate demand is defined as:

A.
net investment in plant and equipment designed to move the economy out of a recession.

B.
total expenditure on consumption, investment, government spending, and net exports during a given year.

C.
a schedule or curve that shows the amount of real GDP or output that buyers collectively desire to buy at every price level.

Incorrect D.
the schedule or curve that shows consumers’ willingness and ability to purchase a particular product at various alternative prices at a given moment in time

A

Aggregate demand is the amount of goods and services—the amount of real national income—that will be purchased at each possible price level. There is an inverse relationship between the price level and real GDP (gross domestic product). There are three different price effects that explain this inverse relationship. The real balance effect reduces the purchasing power effectiveness of accumulated public savings balances. Since consumers are now poorer in real terms, they will need to reduce their spending. The interest-rate effect causes consumers to need more money for their purchases as prices increase. The increase in demand for money will drive up interest rates which would reduce business investment and interest-related consumption spending, thus reducing demand. Finally, we have the foreign purchases effect. When prices of domestic goods rise relative to foreign prices, foreign consumers would buy fewer of our goods, and our consumers would buy more foreign goods.

389
Q

When evaluating capital budgeting analysis techniques, the payback period emphasizes:

Correct A.
liquidity.

B.
profitability.

C.
cost of capital.

D.
net income.

A

Payback period is the length of time in years required to recover the cash invested in a project. Payback is computed as net investment divided by average expected annual cash inflow.

Payback focuses on rapid recovery of cash investment (i.e., liquidity).

390
Q

All of the following are procedures of a change control process, except:

A.
the change control board approves the change.

Correct B.
once the work is done, the process is released without testing.

C.
schedules are set up.

D.
the project manager keeps things running smoothly.

A

The change control process should never be released without testing. The procedures for a well-defined change control process would include the following:

The change control board approves the change and assigns a project manager.
The project manager makes sure all paperwork has been received and approved.
The project manager sets up schedules for all personnel involved.
The projects are completed.
Changes are tested and approved before release.

391
Q

Everything else being equal, a noncallable bond will be priced in comparison to a callable bond so that the noncallable bond will provide:

A.
a higher yield.

B.
a lower yield.

Incorrect C.
the same yield.

D.
a yield 1% less.

A

Callable bonds reduce issuer risk by allowing the bonds to be called in if interest rates decline. The holder of callable bonds, however, is exposed to greater risk (i.e., loss of relatively high interest in a declining interest rate period).

In contrast, a noncallable bond is less risky for a bondholder, so it should sell at a lower yield.

392
Q

Financial statements were being prepared for the ABC Company, and among its long-term investments were 1,000 shares of XYZ common stock, a publicly held company traded on a major market. At the close of the day related to the date of the financial statements, the XYZ stock had a quoted market price of $65 per share; however, at approximately 4:30 p.m., after the close of the market, it was announced that a major fire had destroyed the only production plant held by XYZ Company. As a result of this new information, the quoted market price for the XYZ common stock fell to $25 per share on the following day. When considering all of this information, ABC Company should value the investment of the XYZ common stock at:

A.
$65 per share since there is valid Level 1 input available for the date of the valuation.

B.
$25 per share due to the new information; however, the input level would be dropped to Level 2.

Incorrect C.
$65 per share since there is valid Level 1 input available for the date of the valuation; however, the slide to $25 per share should be disclosed in the notes to the financial statements.

D.
$45 (an average of $65 and $25) and drop the input level to Level 2.

A

As a general rule, if observable inputs such as quoted market prices for identical assets are available, the subject asset should be used in determining value, since this would represent a Level 1 input in the fair value hierarchy. However, if a company announces negative information that has a negative impact on the market price shortly after the close of the market, the fair value valuation should employ the use of the new information and list the value of the subject asset at the lower price. This would also drop the input level to Level 2.

393
Q

All of the following are the rates used in net present value analysis except for the:

A.
cost of capital.

B.
hurdle rate.

Correct C.
accounting rate of return.

D.
required rate of return.

A

The accounting rate of return is a capital budgeting method or technique which disregards the time value of money. It is not a rate used in a net present value analysis. Each of the other terms—cost of capital, hurdle rate, and required rate of return—describes a rate used in net present value analysis.

394
Q

Processing data through the use of simulated files provides an auditor with information about the operat­ing effectiveness of control policies and procedures. One of the techniques involved in this approach makes use of:

A.
controlled reprocessing.

Correct B.
an integrated test facility.

C.
input validation.

D.
program code checking.

A

An integrated test facility allows an auditor to introduce test data (simulated files) into an actual processing run to test the processing of that data. This provides evidence about operating effectiveness of the software.

“Controlled reprocessing” is incorrect because reprocessing the same data again with the same software provides no new information. “Input validation” is incorrect because input validation is a control that improves the accuracy of data entry, but does not provide information about control effectiveness. “Program code checking” is incorrect because manual program code checking in a complex system is a difficult task, sometimes impossible, which is more efficiently done by using test data in an integrated test facility.

395
Q

Maxgo Company is considering replacing its current computer system. The new system would cost Maxgo $60,000 to have it installed and operational. It would have an expected useful life of four years and an estimated salvage value of $12,000. The system would be depreciated on a straight-line basis for financial statement reporting purposes and use the modified accelerated cost recovery system (MACRS) depreciation method for income tax reporting purposes. Assume that the percentages of depreciation for MACRS are 25%, 40%, 20%, and 15% for the 4-year life of the new computer.

Maxgo’s current computer system has been fully depreciated for both financial statement and income tax reporting purposes. It could be used for four more years, but not as effectively as the new computer system. The old system currently has an estimated salvage value of $8,000 and will have an estimated salvage value of $1,000 in four years. It is estimated that the new system will save $15,000 per year in operating costs. Also, because of features of the new software, working capital could immediately be reduced by $3,000 if the new system is purchased. Maxgo expects to have an effective income tax rate of 30% for the next four years.

Assume that Maxgo Company purchases the new system. How would the capital budgeting decision differ if the company chooses to use straight-line depreciation for both financial statement and income tax reporting purposes?

Correct A.
The net present value of the new system would be lower, and the investment would be judged less desirable.

B.
This should have no effect on the decision because depreciation is a noncash expense.

C.
The net present value of the new system would be higher, and the investment would be judged more desirable.

D.
The effect could be either positive or negative, but there is not enough information given to make an informed decision.

A

The use of MACRS depreciation would result in a greater deduction in Year 2 (40% MACRS versus 25% straight line). This will result in tax dollars saved in the early life of the project. The use of straight line (SL) would, of course, not provide this saving until the third and fourth years. The result would be:

a slightly lower net cash flow from the use of SL in year 2.
lower net present value using SL.
less desirable investment.

396
Q

What is the formula for calculating the profitability index of a project?

A.
Subtract actual after-tax net income from the minimum required return in dollars

Correct B.
Divide the present value of the annual after-tax cash flows by the original cash invested in the project

C.
Divide the initial investment for the project by the net annual cash inflow

D.
Multiply net profit margin by asset turnover

A

The profitability index considers the size of the original investment and the value of the discounted cash flows. It is calculated by dividing the present values of the cash flows after the initial investment by that investment.

The other answer choices are incorrect because they do not compare the present value of the investment’s cash flows with the initial investment.

397
Q

Given a 10% discount rate with cash inflows of $3,000 at the end of each year for five years and an initial investment of $11,000, what is the net present value?

A.
$(9,500)

Correct B.
$370

C.
$4,000

D.
$11,370

A

The net present value is the excess of the discounted present value of future cash returns less the investment cost.

The formula to calculate present value for any single future payment is PV = Payment ÷ (1 + r)n, where r is the interest rate and n is the number of periods.

The present value of the payment in the first year is $3,000 ÷ 1.1, or $2,727.
The present value of the payment in the second year is $3,000 ÷ (1.1 × 1.1), or $2,479.
The present value of the payment in the third year is $3,000 ÷ (1.1 × 1.1 × 1.1), or $2,254.
The present value of the payment in the fourth year is $3,000 ÷ (1.1 × 1.1 × 1.1 × 1.1), or $2,049.
The present value of the payment in the fifth year is $3,000 ÷ (1.1 × 1.1 × 1.1 × 1.1 × 1.1), or $1,863.
The sum of the present value of the five future payments is $11,372. The cost of the investment is $11,000, so the net present value is $11,372 - $11,000, or $372, rounded to $370.

398
Q

Joe CPA has just accepted a business valuation engagement for The Charter Company, a small business. Some of the facts related to this company are as follows:

The company has been in existence for 25 years.
For the last 3 years, the company has shown little, if any, profit.
The owner has recently had some difficulty making expected loan principal repayments.
The owner met with a credit counselor last month and is in the process of making some recommended changes.
As a beginning step in the valuation process, Joe has determined the premise of value to be:

A.
liquidation.

B.
bankruptcy.

C.
reorganization.

Correct D.
a going concern.

A

As defined in the International Glossary of Business Valuation Terms, the premise of value is defined as “an assumption regarding the most likely set of transactional circumstances that may be applicable to the subject valuation.”

The premise of value will be one of two situations: a going concern or liquidation. Although The Charter Company has displayed poor performance in the past few years, the business does not appear to be in liquidation at this time. Companies can falter, regroup, and continue. It appears that The Charter Company is currently in the process of regrouping, as demonstrated by the owner’s attempts to obtain and implement expert advice; therefore, this business valuation should use a “going concern” as the premise of value.

399
Q

If a retailer’s terms of trade are 3/10, net 45 with a particular supplier, what is the cost on an annual basis of not taking the discount? Assume a 360-day year.

A.
37.11%

B.
36.00%

C.
24.74%

Correct D.
31.81%

A

Terms of trade credit of 3/10, net 45 means a 3% discount may be taken if the bill is paid in 10 days or the full amount must be paid in 45 days. If a retailer’s payment terms of trade are 3/10, net 45 with a particular supplier, for every $100 the retailer pays in 10 days, he only has to pay $97. If the retailer waits 45 days to pay, he must pay the full $100. This 3% discount works out to be a higher effective interest rate because the three additional dollars the retailer pays in 45 days is in addition to the $97, not the $100.

The natural inclination when presented with a problem of this type is to simply consider the 3% to be 3% of 100%. However, it must be noted that the retailer will only pay 97% if he pays in 10 days. It is this 97% that will be penalized with the additional 3%.

Therefore, the actual interest rate for the period is derived by calculating .03 ÷ .97 or 3.093%. This interest rate is charged over a time period of 35 days (45 days minus 10 days). There are 10.286 35-day periods in 360 (360 ÷ 35). Therefore, the cost of not taking the discount on an annual basis is 10.286 × 3.093% or 31.81%.

400
Q

Edwards Manufacturing Corporation uses the standard economic order quantity (EOQ) model. If the EOQ for Product A is 200 units and Edwards maintains a 50-unit safety stock for the item, what is the average inventory of Product A?

A.
250 units

Correct B.
150 units

C.
100 units

D.
50 units

A

The average inventory level when the standard economic order quantity model is used is one-half of the EOQ. The EOQ for Product A is 200 units. One-half of 200 is 100. Add the 50-unit safety stock to arrive at 150 units as the average inventory of Product A.

401
Q

Allo Foundation, a tax-exempt organization, invested $200,000 in a 5-year project at the beginning of 2006. Allo estimates that the annual cash savings from this project will amount to $65,000. The $200,000 of assets will be depreciated over their 5-year life on the straight-line basis. On investments of this type, Allo’s desired rate of return is 12%. Information on present value factors is as follows:

                                  AT 12%     AT 14%     AT 16%
                                  ------     ------     ------ Present value of 1 for 5 periods      0.5674     0.5194     0.4761 Present value of an annuity of   1 for 5 periods                     3.6048     3.4331     3.2743 Allo's internal rate of return on this project is:

A.
less than 12%.

B.
less than 14%, but more than 12%.

C.
less than 16%, but more than 14%.

Correct D.
more than 16%.

A

nternal rate of return is the interest rate which will result in a net present value (PV) of zero. Therefore:

  PV of Cash Savings = PV of Investment Outlay
 $65,000 x PV factor = $200,000 x 1.0
 $65,000(PV factor)  = $200,000 Then dividing by $65,000:

           PV factor = $200,000 / $65,000
                     = 3.08 Referring to the present value factors in the question, the percentage rates for present value of an annuity indicates that a present value factor of 3.08 would relate to an interest rate of more than 16%.
402
Q

Which of the following statements is true regarding opportunity cost?

A.
Opportunity cost is recorded in the accounts of an organization that has a full costing system.

Incorrect B.
The potential benefit is not sacrificed when selecting an alternative.

C.
Idle space that has no alternative use has an opportunity cost of zero.

D.
Opportunity cost is representative of actual dollar outlay.

A

An opportunity cost is the lost inflow of the next best alternative that is forgone as the result of a decision. Although opportunity costs are important in the decision-making process, they are not included as actual costs in financial statements.

For example, a company has idle space that could be used for the production of a new product or could be rented to a third party. If the production of a new product is considered, then the lost rental income would be an opportunity cost that would have to be considered during the decision-making process. If idle space has no alternative use, it has an opportunity cost of zero.

403
Q

Which of the following formulas should be used to calculate the economic rate of return on common stock?

A.
(Dividends + Change in price) ÷ Beginning price

B.
(Net income - Preferred dividend) ÷ Common shares outstanding

C.
Market price per share ÷ Earnings per share

Incorrect D.
Dividends per share ÷ Market price per share

A

The economic rate of return is a percentage measure of the total return received by the investor. This is determined by the following formula:

[Dividends received + (Ending price - Beginning price)] ÷ Beginning price

404
Q

The following information pertains to Quest Co.’s Gold Division:

  Sales                       $311,000
  Variable cost                250,000
  Traceable fixed costs         50,000
  Average invested capital      40,000
  Imputed interest rate            10%

Quest’s return on investment was:

A.
10.00%.

B.
13.33%.

Correct C.
27.50%.

D.
30.00%.

A

Division net income equals $311,000 sales less variable costs of $250,000 and traceable fixed costs of $50,000, or $11,000.

Return on investment is net income ($11,000) divided by investment ($40,000), or 27.5%.

405
Q

Assume that each day a company writes and receives checks totaling $10,000. If it takes five days for the checks to clear and be deducted from the company’s account, and only four days for the deposits to clear, what is the float?

Correct A.
$10,000

B.
$(10,000)

C.
$50,000

D.
$25,000

A

Ijust have one extra day of 10k since my deposits only take 4 days.

The delay for the checks and deposits to clear the company’s account is called the “float.” During the float, the company has the use of the cash equal to the amount of checks written but does not have the use of deposits until they clear its bank.

Since it takes five days for the checks to clear (i.e., be deducted from the company’s account), the company has the use of $10,000 for five days or what is essentially a “loan” of $50,000. However, for the four days required for the deposits to clear, the company is losing the use of $10,000 × 4 or $40,000. Thus, the company enjoys a net 1-day float on the checks it writes of $10,000.

Thus, skillful management of the float requires the company to delay the coverage of checks as long as possible and to deposit checks received as quickly as possible.

406
Q

A corporation is considering purchasing a machine that costs $100,000 and has a $20,000 salvage value. The machine will provide net annual cash inflows of $25,000 per year and has a 6-year life. The corporation uses a discount rate of 10%. The discount factor for the present value of a single sum 6 years in the future is 0.564. The discount factor for the present value of an annuity for 6 years is 4.355. What is the net present value of the machine?

Incorrect A.
$(2,405)

B.
$8,875

C.
$20,155

D.
$28,875

A

The net present value is the excess of the discounted present value of future cash returns above the investment cost.

The present value of the future cash returns is the annual cash flow of $25,000 multiplied by the present value of an annuity of $1 a year for six years at 10%, which is 4.355. Thus, the present value of the future cash flows is $108,875 ($25,000 × 4.355).

The salvage value of $20,000 will be received in cash in six years, so its present value is $20,000 multiplied by the present value of a single payment of $1 in six years at 10%, or 0.564, giving a present value of the salvage value of $11,280.

Summing the present value of the annual payments ($108,875) and the present value of the salvage value ($11,280) gives a total present value of future cash flows of $120,155.

The cost of the machine is $100,000, so the net present value is $120,155 less $100,000, or $20,155.

407
Q

Many organizations are critically dependent on information systems to support daily business operations. Consequently, an organization may incur significant loss of revenues or incur significant expenses if a disaster such as a hurricane or power outage causes information systems processing to be delayed or interrupted.

Which of the following activities is necessary to determine what would constitute a disaster for an organization?

A.
Risk analysis

B.
File and equipment backup requirements analysis

C.
Vendor supply agreement analysis

Incorrect D.
Contingent facility contract analysis

A

Risk analysis is necessary to determine an organization’s definition of a disaster and evaluate the effect of that disaster.

System backup analysis, vendor supply agreement analysis, and contingent facility contract analysis are all contingency planning strategies to react to a disaster.

408
Q

Variable rate loans can be used to reduce the risk associated with changes in interest rates during the term of the loan. The greatest level of risk relates to ________ loans.

A.
short-term

B.
intermediate-term

Correct C.
long-term

D.
noninterest-bearing

A

Long-term fixed interest rate loans create the greatest interest-rate risk for both borrowers and lenders.

Borrowers benefit from fixed rates when interest rates go up but lose when they decline. Lenders’ risk is in the opposite direction.

The chance for interest rates to change is much greater over a long time period than over a short time period.

409
Q

Selected financial statement data for company Aye is shown as follows:

Additional paid-in capital $ 20,000
Accounts payable (current) 40,000
Accounts receivable (net) 100,000
Cash and marketable securities 70,000
Common stock 150,000
Notes payable (long-term) 50,000
Property, plant, and equipment (net) 150,000
Retained earnings 60,000
What is Aye’s net working capital?

A.
$170,000

Correct B.
$130,000

C.
$80,000

D.
$30,000

A

Net working capital is current assets minus current liabilities.

Aye’s net working
capital = Current assets - Current liabilities
= Cash + Accounts receivable - Accounts payable
= $70,000 + $100,000 - $40,000
= $130,000

410
Q

According to the hedging approach to financing, seasonal variations in current assets should be financed with:

A.
common stock.

B.
long-term debt.

C.
retained earnings.

Correct D.
short-term debt.

A

Under the hedging approach, the length of financing term is matched to the life or duration of assets financed. Long-term assets are financed with long-term debt and short-term assets (such as current assets) are financed with short-term debt.

411
Q

Sylvan Corporation has the following capital structure.

Debenture bonds $10,000,000
Preferred equity 1,000,000
Common equity 39,000,000

The financial leverage of Sylvan Corporation would increase as a result of:

A.
issuing common stock and using the proceeds to retire preferred stock.

B.
issuing common stock and using the proceeds to retire debenture bonds.

Correct C.
financing its future investments with a higher percentage of bonds.

D.
financing its future investments with a higher percentage of equity funds.

A

Leverage refers to the amount of debt in the firm’s capital structure. Of the financial instruments mentioned, only debenture bonds are considered to be debt. Both common and preferred stock are considered equity, even though preferred stock may pay a fixed dividend.

412
Q

In order to increase production capacity, Gunning Industries is considering replacing an existing production machine with a new technologically improved machine effective January 1, 2007. The following information is being considered by Gunning Industries:

The new machine would be purchased for $160,000 in cash. Shipping, installation, and testing would cost an additional $30,000.
The new machine is expected to increase annual sales by 20,000 units at a sales price of $40 per unit. Incremental operating costs are comprised of $30 per unit in variable costs and total fixed costs of $40,000 per year.
The investment in the new machine will require an immediate increase in working capital of $35,000 that will not be needed at the end of the useful life of the new machine.
Gunning uses straight-line depreciation for financial reporting and tax reporting purposes. The new machine has an estimated useful life of five years and zero salvage value.
Gunning is subject to a 40% corporate income tax rate.
Gunning uses the net present value method to analyze investments and will employ the following factors and rates.

                                 Present Value of an
         Present Value of        Ordinary Annuity of Period          $1 at 10%                $1 at 10% ------       ----------------        -------------------   1              0.9091                    0.9091   2              0.8264                    1.7355   3              0.7513                    2.4869   4              0.6830                    3.1699   5              0.6209                    3.7908

The overall discounted cash flow impact of Gunning Industries’ working capital investment for the new production machine would be:

A.
$(7,959).

B.
$(10,680).

Correct C.
$(13,268).

D.
$(35,000).

You are correct, the answer is C.

A

The overall discounted cash flow impact of Gunning Industries’ Working Capital investment for the new production machine is calculated as follows:

– Initial working capital investment at
beginning of Year 1: $(35,000)
– Discounted return of initial WC investment
at end of Year 5:
$35,000 x Present value of $1 at 10% for
5 periods (.6209) 21,732
———
Overall discounted cash flow of Working Capital
Investment: $(13,268)

413
Q

Which risk response reflects a change from acceptance to sharing?
A. After employees stole numerous inventory items, management implemented mandatory background checks on all employees.
B. An insurance policy on a manufacturing plant was not renewed.
C. Management sold a manufacturing plant.
D. Management purchased insurance on previously uninsured property.

A

Answer (D) is correct.
The categories of risk responses under the COSO ERM model are avoidance, acceptance, reduction, and sharing. If management does not insure a building, the response is acceptance. Ordinarily, acceptance is based on a judgment that the cost of another response is excessive. However, once management purchases insurance, the risk is shared with an outside party.

414
Q

According to COSO, the four categories of entity objectives in the enterprise risk management framework include each of the following, except
A. Reliability of reporting.
B. Implementation of internal controls.
Answer (B) is correct.
Implementation of internal control is not an objective of enterprise risk management.
C. Effective and efficient use of the entity’s resources.
D. Compliance with applicable laws and regulations.

A

Answer (B) is correct.

Implementation of internal control is not an objective of enterprise risk management

415
Q

Inherent risk is
A. Risk response risk.
B. The risk after management takes action to reduce the impact or likelihood of an adverse event.
C. The risk when management has not taken action to reduce the impact or likelihood of an adverse event.
Answer (C) is correct.
Inherent risk is the risk when management has not taken action to reduce the effect or likelihood of an adverse event. Thus, it is risk in the absence of a risk response.
D. A potential event that will adversely affect the organization.

A

Answer (C) is correct.
Inherent risk is the risk when management has not taken action to reduce the effect or likelihood of an adverse event. Thus, it is risk in the absence of a risk response.

416
Q

Which of the following actions is required to ensure the validity of a contract between a corporation and a director of the corporation?
A. The director must disclose the interest to the independent members of the board and refrain from voting.

B. The shareholders must review and ratify the contract.
C. The director must resign from the board of directors.
D. An independent appraiser must render to the board of directors a fairness opinion on the contract.

A

Answer (A) is correct.
To protect the corporation against self-dealing, a director is required to make full disclosure of any financial interest (s)he may have in any transaction to which both the director and the corporation may be a party. Under modern corporate law, a transaction is not voidable merely on the grounds of a director’s conflict of interest if the transaction is fair to the corporation or has been approved by a majority of (1) informed, disinterested qualified directors or (2) holders of qualified shares. This rule applies even if the director was counted for the quorum and voted to approve the transaction. A qualified director does not have (1) a conflict of interest regarding the transaction or (2) a special relationship (familial, professional, financial, etc.) with another director who has a conflict of interest. Shares are qualified if they are not controlled by a person with (1) a conflict of interest or (2) a close relationship with someone who has a conflict. Thus, the director who contracts with the corporation cannot provide the vote that approves the contract.

417
Q

The COSO model for internal control lists seven specific areas encompassed by the control environment component. Which of the following are elements of the control environment?
A. Organizational structure.
B. All of the answers are correct.
es, (2) integrity and ethical values, (3) organizational structure, (4) commitment to competence, (5) philosophy and opAnswer (B) is correct.
The elements of the control environment are: (1) human resource policies and practicerating style of management, (6) board of directors or audit committee participation, and (7) assignment of authority and responsibility.
C. Integrity and ethical values.
D. Assignment of authority and responsibility.

A

control environment are: (1) human resource policies and practices, (2) integrity and ethical values, (3) organizational structure, (4) commitment to competence, (5) philosophy and operating style of management, (6) board of directors or audit committee participation, and (7) assignment of authority and responsibility.
C. Integrity and ethical values.

418
Q

Question: 64 Internal controls are likely to fail for any of the following reasons, except
A. They are not designed and implemented properly at the outset.
B. They are designed and implemented properly as static controls, but the environment in which they operate changes.
C. They are designed and implemented properly, but their operation changes in some way.
D. They are designed and implemented properly, and their design changes as processes change

A

Answer (D) is correct.
After the internal control is designed and implemented, the inherent design of the control will not change. Therefore, internal controls are not likely to fail because their design changes. However, internal controls may fail due to (1) established objectives not suitable for internal control, (2) failures due to human judgment and errors, (3) breakdowns and employee misunderstanding, (4) management override, (5) collusion, and (6) external events.

419
Q

ERM allows management greater capabilities. Which of the following is not a capability of ERM?
A. Reduced operational surprises and losses.
B. Better deployment of capital.
C. Increased productivity.
Answer (C) is correct.
The following are the categories of the capabilities of ERM:
Risk appetite and strategy
Risk response decisions
Operational surprises and losses
Multiple and cross-enterprise risks
Opportunities
Deployment of capital
Although increased productivity may result from ERM, it is not directly a capability provided by ERM.
D. Improved risk response decisions.

A
nswer (C) is correct. 
The following are the categories of the capabilities of ERM:
Risk appetite and strategy
Risk response decisions
Operational surprises and losses
Multiple and cross-enterprise risks
Opportunities
Deployment of capital
Although increased productivity may result from ERM, it is not directly a capability provided by ERM.
420
Q
The components of ERM should be present and functioning effectively. What does “present and functioning effectively” mean?
No material weaknesses exist.
Risk is within the risk appetite.
A.	I only.
B.	II only.
C.	Both I and II.
Answer (C) is correct. 
A component is present and functioning effectively if (1) no material weaknesses exist, and (2) risk is within the risk appetite.
D.	Neither I nor II.
A

Answer (C) is correct.
A component is present and functioning effectively if (1) no material weaknesses exist, and (2) risk is within the risk appetite.

421
Q

Enterprise risk management
A. Guarantees achievement of organizational objectives.
B. Requires establishment of risk and control activities by internal auditors.
C. Involves the identification of events with negative impacts on organizational objectives.
D. Includes selection of the best risk response for the organization.
Answer (D) is incorrect.
Enterprise risk management is concerned with selecting not the best risk response but the risk response that falls within the enterprise’s risk tolerances and appetite.

A

Answer (D) is incorrect.
Enterprise risk management is concerned with selecting not the best risk response but the risk response that falls within the enterprise’s risk tolerances and appetite.

422
Q
Which of the following members of an organization has ultimate ownership responsibility of the enterprise risk management, provides leadership and direction to senior managers, and monitors the entity’s overall risk activities in relation to its risk appetite?
A.	Chief risk officer.
B.	Chief executive officer.
C.	Internal auditors.
D.	Chief financial officer.
Answer (D) is incorrect. 
The CFO is subordinate to the CEO.
A

Answer (B) is correct.
The chief executive officer (CEO) sets the tone at the top of the organization and has ultimate responsibility for ownership of the ERM. The CEO will influence the composition and conduct of the board, provide leadership and direction to senior managers, and monitor the entity’s overall risk activities in relation to its risk appetite. If any problems arise with the organization’s risk appetite, the CEO will also take any measures to adjust the alignment to better suit the organization.

423
Q
Question: 44	The policies and procedures helping to ensure that management directives are executed and actions are taken to address risks to achievement of objectives are best described as
A.	Risk assessments.
B.	Control environments.
C.	Control activities.
D.	Monitoring activities.
A

Answer (C) is correct.
The COSO model for internal control describes control activities as the policies and procedures helping to ensure that management directives are executed and actions are taken to address risks to achievement of objectives.